You are on page 1of 133

lOMoARcPSD|16636219

QT Marketing (trắc nghiệm)

marketing management (Trường Đại học Ngân hàng Thành phố Hồ Chí Minh)

Studocu is not sponsored or endorsed by any college or university


Downloaded by Lan Anh Tr?nh (lananh30303@gmail.com)
lOMoARcPSD|16636219

Chapter 1: Defining Marketing for the New Realities


1) Which of the following statements about marketing is true?
A. It is of little importance when products are standardized.
B. It can help create jobs in the economy by increasing demand for goods and services.
C. It helps to build a loyal customer base but has no impact on a firm's intangible assets.
D. It is more important for bigger organizations than smaller ones.
E. It is seldom used by nonprofit organizations.
2) _________ management is the art and science of choosing target markets and getting, keeping, and growing
customers through creating, delivering, and communicating superior customer value.
A. Marketing
B. Knowledge
C. Operations
D. Strategic
E. Distribution
3) Identify the correct statement about marketing management.
A. It is primarily concerned with the systematic gathering, recording, and analysis of data about issues related to
marketing products and services.
B. It focuses mostly on monitoring the profitability of a company's products and services.
C. It focuses solely on attaining an organization's sales goals in an efficient manner.
D. It is defined as the field that deals with planning and managing a business at the highest level of corporate hierarchy.
E. It occurs when at least one party to a potential exchange thinks about the means of
achieving desired responses from other parties.
4) A social definition of marketing says _________.
A. effective marketing requires companies to remove intermediaries to achieve a closer connection with direct consumers
B. a company should focus exclusively on achieving high production efficiency, low costs, and mass distribution to
facilitate the broadest possible access to the company's products
C. marketing is the process by which individuals and groups obtain what they need and want through creating, offering,
and freely exchanging products and services of value with others
D. marketing is the process of extracting the maximum value from consumers to facilitate corporate growth
E. marketing is the process of aggressive selling and promotion to encourage the purchase of products that might
otherwise be unsought by the consumer
5) _________ goods constitute the bulk of most countries' production and marketing efforts.
A. Durable
B. Impulse
C. Physical
D. Luxury
E. Intangible
6) As economies advance, a growing proportion of the economy's activities focuses on the production of
_________ .
A. products
B. events
C. experiences
D. luxury goods
E. services
7) Car rental firms, hair dressers, and management consultants provide_________ .
A. goods
B. experiences
C. events
D. services
E. information
8) Soccer's World Cup is promoted aggressively to both companies and fans. This is an example of marketing
a(n) _________ .
A. idea
B. place
C. luxury item
D. event
E. service
9) The "Malaysia, Truly Asia" ad campaign that showcased Malaysia's beautiful landscape and its multicultural
society in order to attract tourists is an example of _________ marketing.
A. event
B. property
C. service

Downloaded by Lan Anh Tr?nh (lananh30303@gmail.com)


lOMoARcPSD|16636219

D. place
E. idea
10) In Walt Disney's Magic Kingdom, customers can visit a fairy kingdom, a pirate ship, or even a haunted house.
Disney is marketing a(n) _________.
A. experience
B. service
C. event
D. organization
E. good
11) Janet is very upset that she can't get tickets to the Rolling Stones concert because they are sold out. Which
of the following demand states applies to Janet's situation?
A. nonexistent demand
B. latent demand
C. full demand
D. unwholesome demand
E. overfull demand
12) Sales of woolen clothing usually increase during the winter season and decline thereafter. This is an example
of _________ demand.
A. irregular
B. declining
C. impulse
D. latent
E. negative
13) Julia is worried about the rising pollution levels in her city. She doesn't mind paying extra for goods and
services that use sustainable processes to help control pollution. This is an example of _________ for pollution.
A. declining demand
B. nonexistent demand
C. latent demand
D. negative demand
E. unwholesome demand
14) People in emerging countries today are becoming increasingly health conscious and are seeking healthy
food choices. As a result, demand for health foods is rising steadily, creating an opportunity for marketers to
exploit this _________ market.
A. demographic
B. business
C. need
D. geographic
E. service
15) When consumers share a strong need that cannot be satisfied by an existing product, it is called _________
demand.
A. negative
B. latent
C. declining
D. irregular
E. nonexistent
16) When demand is _________ , it implies that more customers would like to buy the product than can be
satisfied.
A. latent
B. irregular
C. overfull
D. full
E. negative
17) In the case of _________ demand, consumers dislike the product and may even pay a price to avoid it.
A. nonexistent
B. overfull
C. irregular
D. negative
E. declining
18) Organizations catering to the _________ market need to price their offerings carefully because these markets
usually have limited purchasing power.
A. business
B. global

Downloaded by Lan Anh Tr?nh (lananh30303@gmail.com)


lOMoARcPSD|16636219

C. nonprofit
D. consumer
E. exclusive
19) Which of the following is true of business markets?
A. Buyers are usually not skilled at comparing competitive product offerings.
B. Buyers have limited purchasing power.
C. Property rights, language, culture, and local laws are the most important concerns.
D. Products sold in such markets are usually highly standardized.
E. Business buyers purchase goods to make or resell a product to others at a profit.
20) The responses marketers seek from prospects include all of the following EXCEPT _________.
A. decampment
B. attention
C. a purchase
D. a vote
E. a donation
21) Jorge drinks two liters of soda each day. He is aware of the negative impacts of sugary drinks on his health
and the potential for the empty bottles to add to pollution. His demand state is best characterized as _________
demand.
A. overfull
B. latent
C. irregular
D. unwholesome
E. nonexistent
22) P&G sells cosmetics, laundry detergent, and products like Pringles to the _________ market by establishing
strong brand images, developing superior products, ensuring their availability, and backing them with engaging
communications and reliable performance.
A. business
B. governmental
C. global
D. nonprofit
E. consumer
23) _________ are basic human requirements, while _________ are the ways in which those requirements are
satisfied.
A. Wants; needs
B. Demands; wants
C. Needs; wants
D. Needs; demands
E. Demands; needs
24) When Frank buys his own house, he would like to have a home theater system and a jacuzzi. He plans to save
enough money in the next three years so that he can fulfill his wish. Frank's desire for the home theater and the
jacuzzi is an example of a(n) _________.
A. need
B. want
C. demand
D. unstated need
E. latent demand
25) When companies measure the number of people who are willing and able to buy their products, they are
measuring _________.
A. demand
B. price elasticity
C. real needs
D. standard of living
E. disposable income
26) A(n) _________ need is one that the consumer is reluctant or unwilling to explicitly verbalize.
A. secondary
B. unstated
C. delight
D. secret
E. stated
27) A(n) __________ need is a need that the consumer explicitly verbalizes.
A. stated
B. affirmative

Downloaded by Lan Anh Tr?nh (lananh30303@gmail.com)


lOMoARcPSD|16636219

C. unsought
D. delight
E. secret
28) The identification and profiling of distinct groups of buyers who might prefer or require varying product and
service mixes is known as __________.
A. segmentation
B. integration
C. disintermediation
D. cross-selling
E. customization
29) Companies address needs by putting forth a _________, a set of benefits that they offer to customers to
satisfy their needs.
A. brand
B. value proposition
C. deal
D. marketing plan
E. demand
30) During market segmentation analysis, the marketer identifies which segments present the greatest
opportunity. These segments are called _________ markets.
A. target
B. capital
C. tertiary
D. demographic
E. developing
31) _________ reflects a customer's judgment of a product's performance in relation to his or her expectations.
A. Brand equity
B. Satisfaction
C. Value
D. Perception
E. Brand image
32) The value of an offering is described as the _________.
A. price consumers are charged for a product
B. cost of manufacturing a product
C. degree to which consumer demand for a product is positive
D. sum of the tangible and intangible benefits and costs to customers
E. intangible benefits gained from a product
33) When Volvo runs ads suggesting that its cars are the safest that money can buy, it is trying to _________.
A. segment the market
B. provide a service
C. enter a new market
D. develop brand loyalty
E. position its product
34) If a marketer uses warehouses, transportation companies, banks, and insurance companies to facilitate
transactions with potential buyers, the marketer is using a _________ channel.
A. service
B. distribution
C. communication
D. relationship
E. standardized
35) The _________ is the channel from raw materials to components to final products that are carried to final
buyers.
A. communication channel
B. distribution channel
C. supply chain
D. service chain
E. marketing chain
36) The actual and potential rival offerings and substitutes that a buyer might consider are referred to as the
_________.
A. supply chain
B. global market
C. value proposition
D. competition

Downloaded by Lan Anh Tr?nh (lananh30303@gmail.com)


lOMoARcPSD|16636219

E. marketing environment
37) The _________ environment includes the actors involved in producing, distributing, and promoting an
offering. The main actors are the company, suppliers, distributors, dealers, and the target customers.
A. economic
B. management
C. strategic
D. task
E. tactical
38) Chipotle engages customers through Facebook, Twitter, and other social media via its grassroots "Food with
Integrity" digital strategy, which puts the focus on what it sells and where it comes from. These are examples of
_________ media.
A. owned
B. earned
C. paid
D. service
E. marketing
39) Which of the following is NOT true regarding impressions?
A. They are useful for tracking the scope of a communication.
B. They are useful for tracking the breadth of a communication's reach.
C. They can be compared across all communication types.
D. They represent a less active response than engagement.
E. They provide insight into the results of viewing the communication.
40) In an attempt to create greater competition and growth opportunities, countries often _________.
A. increase trade barriers
B. protect industries
C. deregulate industries
D. encourage intermediation
E. reduce privatization
41) Which of the three transformative forces mentioned in the chapter is associated with the number of mobile
phones in India recently exceeding 500 million and Boston Consulting Group believing brand marketers must
enhance their "digital balance sheets"?
A. demographics
B. accountability
C. social responsibility
D. globalization
E. technology
42) By 2025, annual consumption in emerging markets will total $30 trillion and contribute more than _________
percent of global GDP growth.
A. 65
B. 70
C. 78
D. 81
E. 85
43) The three central trends in Marketing 3.0 are collaborative marketing, globalization, and ________.
A. cultural relevance
B. horizontal marketing
C. consumer well-being
D. the rise of a creative society
E. sustained technological development
44) The Body Shop, Timberland, and Patagonia incorporate _________ as a way to differentiate themselves from
competitors, build consumer preference, and achieve notable sales and profit gains.
A. social responsibility
B. technology
C. globalization
D. collaboration
E. goodwill
45) Tracy's is a chain of hair dressing salons for women. They use television, magazines, radio, and newspapers
to advertise their services. The owners ensure that all communication channels deliver a common message to
prospective customers. Tracy's believes in _________ marketing.
A. internal
B. integrated
C. socially responsible

Downloaded by Lan Anh Tr?nh (lananh30303@gmail.com)


lOMoARcPSD|16636219

D. global
E. relationship
46) The ultimate outcome of relationship marketing is a unique company asset called the _________, consisting
of the company and its supporting stakeholders.
A. brand
B. supply chain
C. marketing network
D. value proposition
E. service channel
47) Which of the following is NOT one of the 4 As customers most value?
A. acceptability
B. affordability
C. affability
D. accessibility
E. awareness
48) Which of the following reflects the "people" component of the marketing mix?
A. the creativity, discipline, and structure brought to marketing management
B. the development of new products by the marketers
C. the firm's consumer-directed activities
D. the right set of processes to guide activities and programs within the firm
E. the internal marketing of the firm
49) Which of the new four Ps encompasses the old four Ps as well as a range of other marketing activities that
might not fit well into the old view of marketing?
A. programs
B. processes
C. promotion
D. people
E. performance
50) Showrooming is associated with which of the following new consumer capabilities?
A. Consumers can tap into social media to share opinions and express loyalty.
B. Consumers can actively interact with companies.
C. Consumers can use the Internet as a powerful information and purchasing aid.
D. Consumers can reject marketing they find inappropriate.
E. Consumers can communicate on the move.
51) The drugstore chain CVS uses loyalty-card data to better understand what
consumers purchase, the frequency of store visits, and other buying preferences,
which is associated with which technology-enhanced company capability?
A. Companies can reach consumers quickly and efficiently via social media and mobile marketing, sending targeted ads,
coupons, and information.
B. Companies can improve purchasing, recruiting, and internal and external communications.
C. Companies can improve cost efficiency.
D. Companies can collect fuller and richer information about markets, customers, prospects, and competitors.
E. Companies can use the Internet as a powerful sales channel.
52) The success of online purchasing resulted in _________ in the delivery of products and services by
intervening in the traditional flow of goods through distribution channels.
A. disintermediation
B. diversification
C. reduced competition
D. deregulation
E. privatization
53) In response to threats from such companies as Amazon.com, E*TRADE, and dozens of others, established
manufacturers and retailers became "brick-andclick" oriented by adding online services to their existing
offerings. This is known as _________.
A. reintermediation
B. disintermediation
C. retail transformation
D. e-collaboration
E. new-market synchronization
54) When eBay and Amazon.com cut out the majority of middlemen that normally would participate in the
exchange process, it is an example of _________.
A. deregulation
B. reverse auctioning

Downloaded by Lan Anh Tr?nh (lananh30303@gmail.com)


lOMoARcPSD|16636219

C. reintermediation
D. disintermediation
E. diversification
55) Each of the following is true about the Internet's impact on the way business is conducted today, EXCEPT
one. Identify the exception.
A. It has facilitated high-speed communication among employees.
B. It has empowered consumers with easy access to information.
C. It can be used as a powerful sales channel.
D. It has facilitated mass marketing but not the sale of customized products.
E. It enables marketers to use social media to advertise their products.
56) Which of the following is NOT a consumer touch point?
A. store layouts
B. package designs
C. product functions
D. shipping and logistics
E. All of these are touch points.
57) Which of the following holds that consumers prefer products that are widely available and inexpensive?
A. the product concept
B. the production concept
C. the selling concept
D. the performance concept
E. the marketing concept
58) Managers of _________ businesses concentrate on achieving high manufacturing efficiency, low costs, and
mass distribution.
A. selling-oriented
B. product-oriented
C. production-oriented
D. marketing-oriented
E. consumer-oriented
59) The _________ concept holds that consumers will favor offerings with the best quality, performance, or
innovative features.
A. product
B. marketing
C. production
D. selling
E. holistic marketing
60) Which of the following categories of goods and services is most likely to require an aggressive use of the
selling concept?
A. shopping goods
B. unsought goods
C. necessary goods
D. luxury goods
E. complementary goods
61) As a major steel manufacturer, SteelMakers Inc. focuses on having the most efficient manufacturing
processes in place. The company believes that its competitive edge lies in its ability to offer the best prices. They
also maintain an excellent distribution network that ensures wide availability of their products. SteelMakers has a
_________ orientation.
A. selling
B. production
C. product
D. marketing
E. social
62) Webmax Inc. produced and marketed cameras. After considerable research and development, they developed
a new digital camera that had an array of new features. Webmax was so sure about the new offering that they
even reduced their marketing budget. What sort of orientation does Webmax have toward the marketplace?
A. production orientation
B. product orientation
C. selling orientation
D. marketing orientation
E. holistic marketing orientation
63) Rick Johnson trains his company's sales force to target the consumer. He repeatedly asks his team to bear in
mind the fact that it is the sales team's responsibility to rouse the consumer's interest and make the consumer

Downloaded by Lan Anh Tr?nh (lananh30303@gmail.com)


lOMoARcPSD|16636219

feel that he or she needs the product. A true salesman is one who can convert an indifferent consumer walking
into the store into a new customer. Johnson believes in the _________ concept.
A. product
B. production
C. selling
D. marketing
E. social responsibility
64) Marketers at Johnny Inc. believe in putting their customers ahead of
everything else. Their products are carefully designed to meet customer
requirements and the entire focus is on achieving customer satisfaction. Johnny
Inc. follows the _________ concept in doing business.
A. production
B. product
C. selling
D. marketing
E. social responsibility
65) The _________ concept holds that consumers and businesses, if left alone, will ordinarily not buy enough of
the organization's products.
A. production
B. selling
C. marketing
D. product
E. holistic marketing
66) Identify the concept that emerged in the mid-1950s as a customer-centered, sense-and- respond philosophy.
A. the product concept
B. the production concept
C. the selling concept
D. the marketing concept
E. the holistic marketing concept
67) The marketing concept holds that _________.
A. a firm should find the right products for its customers, and not the right customers for its products
B. customers who are coaxed into buying a product will most likely buy it again
C. a new product will not be successful unless it is priced, distributed, and sold properly
D. consumers and businesses, if left alone, won't buy enough of the organization's products
E. a better product will by itself lead people to buy it without much effort from the sellers
68) _________ marketing is based on the development, design, and implementation of marketing programs,
processes, and activities that recognize their breadth and interdependencies.
A. Niche
B. Holistic
C. Relationship
D. Supply chain
E. Demand-centered
69) What are the four broad components of holistic marketing?
A. relationship, internal, position, and performance marketing
B. integrated, internal, position, and performance marketing
C. relationship, integrated, internal, and performance marketing
D. integrated, relationship, social responsibility, and position marketing
E. relationship, social responsibility, internal, and performance marketing
70) _________ marketing aims to build mutually satisfying long-term collaboration with key constituents, such as
customers, employees, suppliers, distributors, and other marketing partners, in order to earn and retain their
business.
A. Integrated
B. Demand-based
C. Direct
D. Relationship
E. Internal
71) Which of the following is most consistent with the integrated marketing approach?
A. A good product will sell itself.
B. If left alone, consumers are inclined to purchase only inexpensive products.
C. All communication to consumers must deliver a consistent message irrespective of the medium.
D. In order to succeed, the main focus should be on having an efficient production process in place.
E. Online marketing is less important than traditional marketing efforts.

Downloaded by Lan Anh Tr?nh (lananh30303@gmail.com)


lOMoARcPSD|16636219

72) Which aspect of holistic marketing motivates employees and ensures that everyone in the organization
embraces appropriate marketing principles, especially senior management?
A. relationship marketing
B. integrated marketing
C. internal marketing
D. network marketing
E. performance marketing
73) Financial accountability and social responsibility marketing are elements of _________ marketing.
A. performance
B. relationship
C. internal
D. social
E. mass
74) Companies are recognizing that much of their market value comes from _________, particularly their brands,
customer base, employees, distributor and supplier relations, and intellectual capital.
A. variable assets
B. value propositions
C. intangible assets
D. market offerings
E. industry convergence
75) Holistic marketing incorporates _________ marketing, an understanding of broader concerns in the ethical,
environmental, legal, and social context of marketing activities.
A. internal
B. cultural
C. social responsibility
D. relationship
E. integrated
76) Design is at the root of _________.
A. acceptability
B. affordability
C. affability
D. accessibility
E. awareness
77) McCarthy classified marketing activities into the four Ps of the marketing mix. These four Ps stand for
_________.
A. product, positioning, place, and price
B. product, production, price, and place
C. promotion, place, positioning, and price
D. place, promotion, production, and positioning
E. product, price, promotion, and place
78) Incorporating the holistic view of marketing, the four Ps of the marketing mix can be updated to _________.
A. product, positioning, people, and price
B. people, processes, place, and promotion
C. product, processes, price, and people
D. people, processes, programs, and performance
E. product, price, promotion, and people
79) __________ combined with affordability determines the product's value proposition.
A. Acceptability
B. Affluence
C. Affability
D. Accessibility
E. Awareness
80) Sheth and Sisodia say _________ is ripest for improvement because most companies are either ineffectual or
inefficient at developing it.
A. acceptability
B. affordability
C. affability
D. accessibility
E. awareness
81) At the heart of any marketing program is the firm's _________, its tangible offering to the market.
A. strategy
B. product

Downloaded by Lan Anh Tr?nh (lananh30303@gmail.com)


lOMoARcPSD|16636219

C. brand
D. value
E. people
82) _________ activities include those the company undertakes to make the product accessible and available to
target customers.
A. Line extension
B. Segmentation
C. Marketing research
D. Channel
E. New-product development

Downloaded by Lan Anh Tr?nh (lananh30303@gmail.com)


lOMoARcPSD|16636219

Chapter 2: Developing Marketing Strategies and Plans


1) The task of any business is to _________.
A. create customer needs
B. differentiate in terms of cost of production
C. deliver customer value at a profit
D. reduce competition
E. communicate similar value as provided by competitors
2) What is the traditional view of marketing?
A. Firms should just focus on production because if the products are good then they will automatically sell.
B. Firms should just focus on production and selling because marketing occurs as a part of the selling process.
C. Firms should have a proper marketing team that can increase consumers' awareness of their products and rouse their
interest in them.
D. Firms should price their products as low as possible so that marketing them becomes easy.
E. Firms should remember that promotion is the most important of the four Ps.
3) The value delivery process can be divided into three phases, out of which "choosing the value" implies
_________, which is the essence of strategic marketing.
A. segmentation, developing, and delivering
B. targeting, positioning, and communicating
C. targeting, positioning, and delivering
D. segmentation, targeting, and positioning
E. researching, developing, and delivering
4) Apex Corporation is one of the best in its industry in terms of costs and performance. Many companies in its
industry will probably consider Apex as a
_________.
A. pioneer
B. benchmark
C. target for acquisition
D. future supplier
E. sounding board for ideas
5) James Frank has been put in charge of gathering marketing intelligence, disseminating it within his
organization, and eventually directing action on the information. Mr. Frank's task is best described as part of the
_________ process.
A. market-sensing
B. new-offering realization
C. customer acquisition
D. customer relationship management
E. fulfillment management
6) When a customer places an order at BookBox.com, the company processes the customer's payment
information, sends the order to the nearest warehouse, and ships the order via FedEx. This is best described as
the _________ process.
A. market-sensing
B. customer acquisition
C. customer relationship management
D. fulfillment management
E. new-offering realization
7) Today, the "mass market" is actually splintering into numerous segments,
each with its own wants, perceptions, preferences, and buying criteria. This
implies that _________.
A. the traditional marketing approach would be the best approach to follow
B. the producers must consider themselves as a part of the value-chain process
C. the producers should focus on niche markets
D. all market segments are equally profitable
E. target market strategies are no longer effective
8) The first phase of the value creation and delivery sequence is _________ the value that represents the
"homework" marketing must do before any product exists.
A. choosing
B. providing
C. communicating
D. considering
E. acquiring
9) What is the second phase of the value creation and delivery sequence?
A. choosing the value

Downloaded by Lan Anh Tr?nh (lananh30303@gmail.com)


lOMoARcPSD|16636219

B. providing the value


C. communicating the value
D. calculating the value
E. calibrating the value
10) The last step in the value creation and delivery sequence is _________ the value where the sales force, sales
promotion, advertising, and other communication tools announce and promote the product.
A. developing
B. positioning
C. communicating
D. reversing
E. researching
11) Angelo made the decision to outsource the software components of his consulting company so he could
focus on the company's _________, which are sources of competitive advantage, make a contribution to
perceived customer benefits, have application in a wide variety of markets, and are difficult to imitate.
A. core competencies
B. infrastructure
C. market-sensing processes
D. fulfillment management processes
E. outbound logistics
12) The _________ in the value chain cover the sequence of bringing materials into the business (inbound
logistics), converting them into final products (operations), shipping out final products (outbound logistics),
marketing them (marketing and sales), and servicing them (service).
A. operations processes
B. manufacturing processes
C. primary activities
D. secondary activities
E. tertiary activities
13) Michael Porter's value chain would identify which of the following as a
support activity?
A. shipping out final products
B. marketing products
C. procurement
D. servicing products
E. operations
14) Which of the following can be considered a primary activity in the value chain process?
A. procurement
B. human resource management
C. technology development
D. inbound logistics
E. firm infrastructure
15) The fulfillment management process includes all the activities in _________.
A. gathering and acting upon information about the market
B. researching, developing, and launching new high-quality offerings quickly and within budget standards
C. defining target markets and prospecting for new customers
D. building deeper understanding, relationships, and offerings to individual customers
E. receiving and approving orders, shipping the goods on time, and collecting payment
16) With respect to core business processes, the _________ process includes all the activities involved in
gathering external information, disseminating it within the organization, and acting on the information.
A. market-sensing
B. market research
C. target marketing
D. market pulse
E. deployment
17) With respect to core business processes, the _________ process includes all the activities involved in
developing and launching high-quality products quickly and within budget.
A. market-sensing
B. new-offering realization
C. fulfillment management
D. customer acquisition
E. customer relationship management
18) Activities involved in building deeper understanding of existing consumers and what the product offerings
mean to them would be a part of the _________ process.

Downloaded by Lan Anh Tr?nh (lananh30303@gmail.com)


lOMoARcPSD|16636219

A. customer acquisition
B. customer relationship management
C. customer prospecting
D. customer integrating management
E. customer equity
19) _________ management allows the company to discover who its target markets are, how they behave, and
what they need or want. It also enables the company to respond appropriately, coherently, and quickly to different
customer opportunities.
A. Network
B. Supply chain
C. Marketing
D. Customer relationship
E. Total quality
20) Another way to describe a supply chain, where companies partner with specific suppliers and distributors to
ensure a smooth functioning of the system, is to call it a _________.
A. teamwork group
B. horizontal integration
C. domestic power center
D. value exploration
E. value delivery network
21) The key to utilizing organizational core competencies is to _________ that make up the essence of the
business.
A. force organizational departments to justify the budgetary components
B. vertically integrate and own all intermediaries
C. own and nurture the resources and competencies
D. emphasize global promotions
E. segment the workforces
22) A _________ has three characteristics: (1) a source of competitive advantage making a significant
contribution to perceived customer benefits; (2) has applications in a wide variety of markets; and (3) is difficult
to imitate.
A. core competency
B. market sensing process
C. corporate social responsibility effort
D. strategic business unit
E. philanthropic endeavor
23) Core competencies tend to refer to areas of special technical and production expertise, whereas _________
tend to describe excellence in broader business processes.
A. process benchmarks
B. distinctive capabilities
C. distributive capabilities
D. facultative benchmarks
E. concentric capabilities
24) Amber oversees the activities at a consumer packaged goods firm that include researching, developing, and
launching new, high-quality shampoos, quickly and within budget. With which process is her job most closely
aligned?
A. the fulfillment management process
B. the market-sensing process
C. the customer acquisition process
D. the customer relationship management process
E. the new-offering realization process
25) As Kodak addresses the digital revolution taking over the photographic industry, it wants customers to see it
as a leader in digital photography. Thus, it is moving away from the production of film roll cameras. This would
be an example of which of the following value creation steps?
A. abandoning current product lines
B. changing the corporate vision
C. repositioning the company's brand identity
D. redoing the corporate logo
E. realigning core competencies
26) In the cycle of complete strategic planning, taking corrective action is a part of _________.
A. planning
B. implementation
C. controlling

Downloaded by Lan Anh Tr?nh (lananh30303@gmail.com)


lOMoARcPSD|16636219

D. organizing
E. analyzing
27) Of the four organizational levels, the corporate level is likely to make which of the following decisions?
A. entering a new market
B. resource allocation for each product
C. strategic plan for individual business units
D. choosing specific suppliers for each business unit
E. marketing plan for each product
28) Most large companies consist of four organizational levels: the corporate level, the _________ level, the
business unit level, and the product level.
A. board of director
B. major stakeholder
C. management team
D. division
E. strategic
29) During Jill's market research study, many customers indicated that traditional oven mitts made it very difficult
to hold baking dishes, resulting in frequent spills. Jill brought her findings to the research department, and her
company leveraged its engineering and design competencies to develop a new handheld hotpad that allows for
significantly greater dexterity in handling hot cooking implements, while protecting the cook from burns. This is
an example of the _________ element of the holistic marketing framework.
A. integrated marketing
B. value creation
C. negative demand
D. value networking
E. value focus
30) The marketing plan, the central instrument for directing and coordinating the
marketing effort of a company, operates at the _________ levels.
A. functional and operational
B. strategic and tactical
C. corporate and operational
D. customer and expenditure
E. corporate and division
31) The _________ lays out the target markets and the value proposition that
will be offered, based on an analysis of the best market opportunities.
A. organizational plan
B. strategic marketing plan
C. corporate tactical plan
D. corporate mission
E. customer-value statement
32) "Enriching Women's Lives" is a three-word mantra that is used by Mary Kay. This is an example of which of
the following characteristics of good mission statements?
A. It is focused on a limited number of goals.
B. It is short, memorable, and meaningful.
C. It defines the major competitive spheres in which the company will operate.
D. It takes a long-term view.
E. It stresses the company's major policies and values.
33) Which of the following plans would most likely include directions for implementing and addressing daily
challenges and opportunities in product features, promotion, merchandising, pricing, sales channels, and service
areas?
A. the tactical marketing plan
B. the target marketing plan
C. the deployment plan
D. the product-launch plan
E. the product-development plan
34) Which of the following is NOT one of the four planning activities undertaken by all corporate headquarters?
A. defining the corporate mission
B. establishing strategic business units
C. assigning resources to each SBU
D. assessing growth opportunities
E. deciding sales channels
35) Juan Garcia plans the daily promotional releases about his company's products and services. He can be
described as a(n) _________ planner.

Downloaded by Lan Anh Tr?nh (lananh30303@gmail.com)


lOMoARcPSD|16636219

A. strategic
B. selective
C. tactical
D. niche
E. operational
36) A clear, thoughtful mission statement provides employees with a shared sense of purpose, direction, and
_________.
A. profitability
B. target market feasibility
C. opportunity
D. continuous improvement
E. quality products
37) Mission statements are at their best when they reflect a _________.
A. market
B. strength
C. competency
D. vision
E. value
38) Which of the following is one of the five major characteristics of good mission statements?
A. They focus on a large number of goals.
B. They expand the range of individual employee discretion.
C. They define the major competitive spheres within which the company will operate.
D. They take a short-term view.
E. They are long and comprehensive to ensure that all critical concepts are included.
39) The _________ sphere is the number of channel levels, from raw materials to final product and distribution, in
which a company will participate.
A. industry
B. vertical
C. product/application
D. competence
E. market segment
40) While viewing businesses in terms of customer needs can suggest additional growth opportunities, a
_________ definition tends to focus on selling a product or service to a current market.
A. strategic market
B. target market
C. cognitive
D. product
E. tactical
41) What are hollow corporations?
A. companies that market their products through franchisees
B. companies that have liabilities exceeding their assets
C. companies that outsource all production to suppliers
D. companies that do not have any physical presence and only operate online
E. companies that are horizontally integrated
42) A(n) _________ market definition of a business focuses on selling a product or service to an existing market.
A. target
B. strategic
C. mass
D. differentiated
E. integrated
43) Which of the following is best described as a market-oriented business definition?
A. Missouri-Pacific Railroad: We run a railroad.
B. Xerox: We make copying equipment.
C. Standard Oil: We sell gasoline.
D. Encyclopedia Britannica: We distribute information.
E. Columbia Pictures: We make movies.
44) A characteristic of a _________ unit is that it can be a single business or collection of related businesses that
can be planned separately from the rest of the company.
A. strategic business
B. subsidiary
C. merged
D. niche market

Downloaded by Lan Anh Tr?nh (lananh30303@gmail.com)


lOMoARcPSD|16636219

E. specialized business
45) A strategic-planning gap can be corrected by _________.
A. changing the company's mission
B. adjusting its core values
C. choosing integrative growth strategies
D. redefining organizational culture
E. increasing resource use
46) Market-penetration, product-development, and market-development strategies would all be examples of
_________ strategies.
A. concentric growth
B. conglomerate
C. horizontal
D. intensive growth
E. integrative growth
47) Which of the following is an example of growth by diversification?
A. a company introducing its existing products in a new market
B. a company introducing new product category in a new market
C. a company increases its product line in an existing market
D. a company introducing a new product category in an existing market
E. a company integrates backward to cut costs
48) A company that seeks to increase its sales and profits through backward, forward, or horizontal integration
within the industry is said to be employing a(n) _________ growth strategy.
A. diversification
B. intensive
C. target
D. integrative
E. conglomerate
49) Which of the following is an example of intensive growth?
A. a company improves its market share with its current products in the current markets
B. a company enters a new market with current products
C. a company develops new products for its existing markets
D. a company improving sales by integrating backward
E. a company developing new products for new markets
50) If you were the CEO of a company that was looking to implement strategies to fill a perceived strategic-
planning gap, you would most likely explore _________ strategy first because it is easier to improve an existing
business than to build a new one.
A. market-penetration
B. market-development
C. diversification
D. product-development
E. exclusive
51) Once Starbucks established its presence in thousands of cities internationally, the company sought to
increase the number of purchases by existing customers with a _________ strategy that led to new in-store
merchandise, including compilation CDs and high-speed wireless access.
A. product-development
B. market-penetration
C. diversification
D. market-development
E. conglomerate
52) A(n) _________ is when a company might seek new businesses to purchase
that have no relationship to its current technology, products, or markets.
A. concentric strategy
B. conglomerate strategy
C. horizontal strategy
D. intensive growth strategy
E. integrative strategy
53) Which of the following terms can be defined as "the shared experiences, stories, beliefs, and norms that
characterize an organization"?
A. organizational dynamics
B. a business mission
C. an ethical/value statement
D. customer relationships

Downloaded by Lan Anh Tr?nh (lananh30303@gmail.com)


lOMoARcPSD|16636219

E. corporate culture
54) Qantas, the Australian national airline carrier, introduces flights to the Middle East. This is an example of
_________.
A. product development
B. diversification
C. market development
D. market penetration
E. differentiation
55) The first step in the business unit strategic-planning process deals with which of the following?
A. formulate goals
B. define business mission
C. implement strategies
D. execute programs
E. conduct SWOT analysis
56) Through its cutting-edge point-of-sale inventory management technology and highly efficient shipping
practices, Walmart is able to keep its inventory expenditure extremely low and to pass these savings on to
consumers in the form of low prices. Walmart's strategy is best described as _________.
A. a focused approach
B. integrative growth
C. differentiation
D. market development
E. overall cost leadership
57) When a firm aims to underprice competitors and win market share, it is using a(n) _________ strategy that
requires relatively less marketing skills as compared to other strategies.
A. product differentiation
B. overall cost leadership
C. focus
D. domestic customer relationship
E. price skimming
58) After analyzing their company's strengths and weaknesses, top managers at Loan Bright decided that they
would serve individual loan officers who typically only wanted to purchase small sets of homebuyer data, rather
than bigger institutional clients. To meet the needs of this segment, Loan Bright simplified its sales contract,
restructured its advertising efforts to focus on Google ads, and created a separate customer-service department.
Loan Bright's strategy is best described as a(n) _________ strategy.
A. overall cost leadership
B. focus
C. differentiation
D. diversification
E. promotional
59) Unlike its competitors in the online air travel industry, Travelocity provides its customers with a greater
variety of services such as cruise reservations, package tours, hotel bookings, and car rentals. This is an
example of a(n) _________ strategy.
A. overall cost leadership
B. focus
C. differentiation
D. diversification
E. promotional
60) McDonald's has often teamed up with Disney to offer products related to current Disney films as part of its
meals for children. The best description of this form of alliance would be a ________.
A. product alliance
B. logistics alliance
C. pricing collaboration
D. network alliance
E. promotional alliance
61) Abbot Laboratories warehouses and delivers 3M's medical and surgical products to hospitals across the
United States. The best description of this form of alliance would be a(n)
A. product alliance
B. logistics alliance
C. pricing collaboration
D. indirect collaboration
E. promotional alliance

Downloaded by Lan Anh Tr?nh (lananh30303@gmail.com)


lOMoARcPSD|16636219

62) MasterCard and Visa may team up with university alumni associations to offer affinity credit cards that
typically display an iconic image associated with the university on the card itself and may include an incremental
donation program associated with purchases made using the card. The best description of this form of alliance
would be a _________.
A. product alliance
B. logistics alliance
C. pricing collaboration
D. network alliance
E. promotional alliance
63) When a business gets to know market segments intimately and pursues either cost leadership or
differentiation within the target segment, it is employing a _________ strategy.
A. defined
B. focused
C. value-added
D. competitive advantage
E. customer-focused
64) When one company licenses another to produce its offerings, or two companies jointly market their
complementary offerings, it is called a _________.
A. pricing collaboration
B. product or service alliance
C. promotional alliance
D. logistics collaboration
E. total quality management
65) To keep their strategic alliances thriving, corporations have begun to develop organizational structures to
support them and have come to view the ability to form and manage strategic alliances as core skills. This is
called _________.
A. value managed partnership
B. decentralized partnership
C. centralized partnership
D. partner relationship management
E. intensive growth management
66) Which of the following statements is true of marketing plans?
A. They can be independently developed without worrying about other functional areas.
B. They provide direction and focus for a brand, product, or company.
C. They are usually profit-oriented.
D. They are of limited use to nonprofit organizations.
E. They are typically five-year plans and they lay out the strategies required to achieve
targets in those five years.
67) Which of the following elements of a marketing plan permits senior management to grasp the plan's major
thrust?
A. the situation analysis
B. the marketing strategy
C. the executive summary
D. the financial projections
E. the short-term targets
68) The most frequently cited shortcomings of current marketing plans, according to marketing executives, are
lack of realism, insufficient competitive analysis, and a _________ focus.
A. long-term
B. profit
C. short-run
D. product
E. price
ANSWER: C
69) The _________ is the last section of the marketing plan. It spells out the goals and budget for each month or
quarter so management can review each period's results and take action as needed.
A. executive summary
B. situation analysis
C. marketing strategy
D. financial projections
E. implementation controls
70) How does a marketing plan affect relationships?
A. It influences how marketing staff work with each other.

Downloaded by Lan Anh Tr?nh (lananh30303@gmail.com)


lOMoARcPSD|16636219

B. It influences how marketing staff work with other departments.


C. It affects how the company works with suppliers.
D. It influences the company's dealings with other stakeholders.
E. All of the above
71) Which financial projections include three estimates (optimistic, pessimistic, and most likely)?
A. sales forecasts
B. risk analyses
C. expense forecasts
D. break-even analyses
E. per-unit variable costs
72) Pegasus writes about its target market and demographics in the _________section of its marketing plan.
A. executive summary
B. implementation
C. situation analysis
D. strategy
E. tactics

Downloaded by Lan Anh Tr?nh (lananh30303@gmail.com)


lOMoARcPSD|16636219

Chapter 10 Crafting the Brand Positioning


1) All marketing strategy is built on STP: segmentation, targeting, and __________.
A. positioning
B. product
C. planning
D. promotion
E. performance
ANSWER: A
2) ___________ is the act of designing the company's offering and image to occupy a distinctive place in the minds of the
target market.
A. Positioning
B. Valuation
C. Pricing
D. Commercialization
E. Launching
ANSWER: A
3) The goal of positioning is to __________.
A. locate the brand in the minds of consumers to maximize the potential benefit to the firm
B. discover the different needs and groups existing in the marketplace
C. target those customers marketers can satisfy in a superior way
D. collect information about competitors that will directly influence the firms' strategy
E. help the firm anticipate what the actions of its competitors will be
ANSWER: A
4) The result of positioning is the successful creation of _________, which provides a cogent reason why the target
market should buy the product.
A. an award-winning promotional campaign
B. a customer-focused value proposition
C. a demand channel
D. everyday low pricing
E. employee value proposition
ANSWER: B
5) Which of the following best describes a car company's value proposition?
A. We charge a 20 percent premium on our cars.
B. We target safety-conscious upscale families.
C. We sell the safest, most durable wagon.
D. We are the market leader in the small car category.
E. We focus on expanding in faster-growing markets.
ANSWER: C
6) Perdue's cogent reason why a target market should buy its chicken is "More tender golden chicken at a moderate
premium price," also known as its __________.
A. customer-focused value proposition
B. competitive frame of reference
C. points-of-parity
D. straddle positioning
E. perceptual map
ANSWER: A
7) Which of the following best describes BR Chicken's value proposition?
A. We sell chicken at most major malls.
B. We undertake home delivery services.
C. We target quality-conscious consumers of chicken.
D. We sell tender golden chicken at a moderate price.
E. We charge a 10 percent premium on our chicken.
ANSWER: D
8) The ___________ defines which other brands a brand competes with and therefore which brands should be the focus
of competitive analysis.
A. consumer profitability analysis
B. competitor indexing
C. service blueprint
D. competitive frame of reference

Downloaded by Lan Anh Tr?nh (lananh30303@gmail.com)


lOMoARcPSD|16636219

E. cluster analysis
ANSWER: D
9) ___________ are defined as companies that satisfy the same customer need.
A. Communities
B. Competitors
C. Trendsetters
D. Industries
E. Task groups
ANSWER: B
10) A(n) ___________ is a group of firms offering a product or class of products that are close substitutes for one
another.
A. community
B. task force
C. industry
D. focus group
E. umbrella brand
ANSWER: C
11) ___________ refers to the products or sets of products with which a brand competes and which function as close
substitutes.
A. Consumer profitability analysis
B. Competitive frame of reference
C. Category membership
D. Value membership
E. Demand field
ANSWER: C
12) When Coca-Cola determines the bottled-water competitors for its Dasani brand by identifying the products or sets of
products with which a brand competes and which function as close substitutes, it is determining Dasani's __________.
A. customer-focused value proposition
B. points-of-parity
C. points-of-difference
D. category membership
E. brand mantra
ANSWER: D
13) When Coca-Cola focused on developing its soft drink business but missed seeing the market for coffee bars and
fresh-fruit-juice bars that eventually impinged on its soft-drink business, it was suffering from ___________ because it
defined competition in traditional category and industry terms.
A. factor elimination
B. marketing myopia
C. factor reduction
D. category points-of-parity
E. reliance on product description
ANSWER: B
14) Which of the following terms is most closely associated with the statement: "attributes or benefits consumers strongly
associate with a brand, positively evaluate, and believe that they could not find to the same extent with a competitive
brand"?
A. points-of-inflection
B. points-of-difference
C. points-of-parity
D. points-of-value
E. points-of-presence
ANSWER: B
15) Points-of- ___________ are product associations that are not necessarily unique to the brand but may in fact be
shared with other brands.
A. parity
B. difference
C. inflection
D. presence
E. divergence
ANSWER: A

Downloaded by Lan Anh Tr?nh (lananh30303@gmail.com)


lOMoARcPSD|16636219

16) The three criteria that determine whether a brand association can truly function as a point-of- difference are
__________.
A. comparability, authenticity, deliverability
B. desirability, peculiarity, deliverability
C. deviance, peculiarity, deformity
D. desirability, deliverability, differentiability
E. differentiability, authenticity, desirability
ANSWER: D
17) The three criteria that determine whether a brand association can truly function as a point-of- difference include
desirability, _________, and differentiability.
A. discrimination
B. customerization
C. implementation
D. deliverability
E. demand
ANSWER: D
18) Which of the following criteria relates to consumers seeing the brand association as personally relevant to them?
A. deliverability
B. authenticity
C. desirability
D. differentiability
E. feasibility
ANSWER: C
19) Which of the following criteria relates to the company having the internal resources and commitment to feasibly and
profitably create and maintain the brand association in the minds of consumers?
A. differentiability
B. peculiarity
C. desirability
D. believability
E. deliverability
ANSWER: E
20) Which of the following criteria relates to consumers seeing the brand association as distinctive and superior to
relevant competitors?
A. desirability
B. differentiability
C. believability
D. deliverability
E. deviance
ANSWER: B
21) A brand must demonstrate ________, for it to function as a true point-of-difference.
A. clear superiority of an attribute or benefit
B. clear profitability to the company
C. clear similarity to the attributes of other brands
D. technological advances for an attribute or benefit
E. exploitation of competitors' weakness
ANSWER: A
22) The two basic forms of points-of-parity are ___________ points-of-parity and ___________ points- of-parity.
A. conceptual; competitive
B. strategic; conceptual
C. category; deliverable
D. competitive; peculiar
E. category; competitive
ANSWER: E
23) ___________ are attributes or benefits that consumers view as essential to a legitimate and credible offering within a
certain product or service class.
A. Category points-of-difference
B. Conceptual points-of-parity
C. Competitive points-of-parity
D. Category points-of-parity

Downloaded by Lan Anh Tr?nh (lananh30303@gmail.com)


lOMoARcPSD|16636219

E. Competitive points-of-difference
ANSWER: D
24) Philip Morris bought Miller Brewing and launched low-calorie beer, at a time when consumers had the impression that
low-calorie beer does not taste as good as normal beer. What does the company assure by stating that the beer tastes
good?
A. points-of-difference
B. points-of-presence
C. points-of-parity
D. points-of-conflict
E. points-of-inflection
ANSWER: C
25) When BMW first made its strong competitive push into the US market in the early 1980s, it positioned the brand as
the only automobile that offered both luxury (competing with Cadillac) and performance (competing with the Corvette),
which is known as ___________ because it uses points-of-difference and points-of-parity across categories.
A. a competitive frame of reference
B. zone of tolerance positioning
C. straddle positioning
D. red-ocean thinking
E. perceptual mapping
ANSWER: C
26) Philip Morris bought Miller Brewing and launched low-calorie beer, at a time when consumers had the impression that
low-calorie beer does not taste as good as normal beer. What did the company try to build when they conveyed the fact
that the beer contained one third less calories and hence it is less filling?
A. points-of-difference
B. points-of-conflict
C. points-of-parity
D. points-of-presence
E. points-of-inflection
ANSWER: A
27) Consumers might not consider a hand sanitizer truly a hand sanitizer unless they are gels designed to apply topically,
contain alcohol that kills the germs present on the skin, and developed for use after washing hands or for those times
when soap and water are not available. These service elements are considered __________.
A. competitive points-of-difference
B. competitive points-of-parity
C. category points-of-difference
D. category points-of-parity
E. conceptual points-of-parity
ANSWER: D
28) Nivea became the leader in the skin cream class on the "gentle," "protective," and "caring" platform. The company
further moved into classes such as deodorants, shampoos, and cosmetics. Attributes like gentle and caring were of no
value unless consumers believed that its deodorant was strong enough, its shampoo would cleanse, and its cosmetics
would be colorful enough. This is an example of __________.
A. competitive points-of-parity
B. competitive points-of-difference
C. category points-of-parity
D. category points-of-difference
E. competitive points-of-presence
ANSWER: C
29) ___________ are associations designed to overcome perceived weaknesses of the brand.
A. Conceptual points-of-parity
B. Category points-of-difference
C. Competitive points-of-parity
D. Competitive points-of-difference
E. Category points-of-parity
ANSWER: C
30) As a marketing manager, which of the following would be the best purpose for your organization's competitive points-
of-parity?
A. to point out competitors' points-of-difference
B. to emphasize competitors' points-of-difference

Downloaded by Lan Anh Tr?nh (lananh30303@gmail.com)


lOMoARcPSD|16636219

C. to rationalize competitors' perceived points-of-difference


D. to globalize competitors' perceived points-of-difference
E. to negate competitors' perceived points-of-difference
ANSWER: E
31) A marketer that wants to anchor a point-of-difference for Dove soap on brand benefits might emphasize which of the
following?
A. The soap is one-quarter cleansing cream.
B. Dove products include bar soaps and shampoos.
C. Dove soap helps users have softer skin.
D. The soap brand has global presence.
E. The brand has recently launched soap for men.
ANSWER: C
32) Marketers typically focus on brand ___________ in choosing the points-of-parity and points-of-difference that make
up their brand positioning.
A. equity
B. awareness
C. benefits
D. architecture
E. extensions
ANSWER: C
33) ___________ are visual representations of consumer perceptions and preferences.
A. Brand narratives
B. Share of mind variables
C. Perceptual maps
D. Exemplars
E. Points-of-parity
ANSWER: C
34) Straddle positions __________.
A. help firms to analyze who their competitors are
B. allow brands to expand their market coverage and potential customer base
C. are a necessity while creating a firm's vision and mission statement
D. assist firms in collecting information on competitors that will directly influence their strategy
E. are ambiguous moral principles behind the operation and regulation of marketing
ANSWER: B
35) Which of the following statements about brand mantras is true?
A. They guide only major decisions, they have no influence on mundane decisions.
B. Their influence does not extend beyond tactical concerns.
C. They must economically communicate what the brand is and avoid communicating what it is not.
D. They can provide guidance about what ad campaigns to run and where and how to sell the brand.
E. They leverage the values of the brand to take the brand into new markets/sectors.
ANSWER: D
36) American Express' "World-Class Service, Personal Recognition," Mary Kay's "Enriching women's lives," Hallmark's
"Caring Shared," and Starbucks' "Rewarding Everyday Moments" are examples of brand __________.
A. mantras
B. parity
C. identity
D. architecture
E. extension
ANSWER: A
37) Brand ___________ are short, three- to five-word phrases that capture the irrefutable essence or spirit of the brand
positioning and ensure that the company's own employees understand what the brand represents.
A. mantras
B. symbols
C. logos
D. alliances
E. extensions
ANSWER: A
38) A brand ___________ is a translation of the brand mantra that attempts to creatively engage consumers and others
external to the company.

Downloaded by Lan Anh Tr?nh (lananh30303@gmail.com)


lOMoARcPSD|16636219

A. vision
B. extension
C. architecture
D. slogan
E. alliance
ANSWER: D
39) BMW's "The ultimate driving machine," American Express' "Don't leave home without it," New York Times' "All the
news that's fit to print," and AT&T's "Reach out and touch someone" are all examples of brand __________.
A. slogan
B. personality
C. mission
D. architecture
E. vision
ANSWER: A
40) A brand mantra should be __________.
A. original, ambiguous, and straightforward
B. unique, complex, and inspirational
C. communicative, simple, and inspirational
D. competitive, sensitive, and simple
E. unique, sensitive, and explanatory
ANSWER: C
41) Brand mantras typically are designed to capture the brand's points-of-___________ , that is, what is unique about the
brand.
A. conflict
B. parity
C. inflection
D. difference
E. presence
ANSWER: D
42) Points-of-parity are important while designing brand mantras for brands facing __________.
A. rapid growth
B. market saturation
C. slow and steady growth
D. rapid decline
E. stability in sales volume
ANSWER: A
43) For brands in more stable categories where extensions into more distinct categories are less likely to occur, the brand
mantra may focus more exclusively on points-of- __________.
A. difference
B. presence
C. inflection
D. parity
E. conflict
ANSWER: A
44) One common difficulty in creating a strong, competitive brand positioning is that many of the attributes or benefits
that make up the points-of-parity and points-of-difference are __________.
A. negatively correlated
B. always correlated
C. directly proportional
D. never correlated
E. positively correlated
ANSWER: A
45) ___________ is a company's ability to perform in one or more ways that competitors cannot or will not match.
A. Brand positioning
B. Market research
C. Competitive advantage
D. Competitor analysis
E. Competitive intelligence
ANSWER: C

Downloaded by Lan Anh Tr?nh (lananh30303@gmail.com)


lOMoARcPSD|16636219

46) A ___________ advantage is one that a company can use as a springboard to new advantages.
A. sustainable
B. leverageable
C. realistic
D. rational
E. distinct
ANSWER: B
47) Which of the following types of differentiation relates to companies having better-trained personnel who provide
superior customer service?
A. channel differentiation
B. services differentiation
C. employee differentiation
D. image differentiation
E. product differentiation
ANSWER: C
48) Singapore Airlines is well regarded in large part because of the excellence of its flight attendants. This is an example
of ___________ differentiation.
A. image
B. services
C. product
D. employee
E. channel
ANSWER: D
49) Which of the following types of differentiation refers to companies effectively designing their distribution medium's
coverage, expertise, and performance to make buying the product easier and more enjoyable and rewarding?
A. services differentiation
B. channel differentiation
C. image differentiation
D. product differentiation
E. employee differentiation
ANSWER: B
50) Dayton, Ohio-based Iams found success selling premium pet food through regional veterinarians, breeders, and pet
stores. This is an example of ___________ differentiation.
A. service
B. employee
C. image
D. product
E. channel
ANSWER: E
51) Which of the following is an example of channel differentiation?
A. Berry's has an intensive training program for its customer-facing employees, to ensure a consistent service standard.
B. The Swan Hotels use a distinctive signature fragrance in all outlets so that customers can associate the fragrance with
the hotel.
C. JEK's sophisticated customer database allows the company to handle queries and product returns much faster than
competitors.
D. RTZ shifted its products from supermarket aisles to exclusive stores as it realized that customers were willing to pay
more in stores.
E. Hayley's found success by allowing buyers to customize the color and some features of its appliances before buying
them.
ANSWER: D
52) Which of the following is an example of image differentiation?
A. Berry's has an intensive training program for its customer-facing staff, to ensure a consistent service standard.
B. The Swan Hotels use a distinctive signature fragrance in all outlets so that customers can associate the fragrance with
the hotel.
C. JEK's sophisticated customer database allows the company to handle queries and product returns much faster than
competitors.
D. RTZ shifted its products from supermarket aisles to exclusive stores as it realized that customers were willing to pay
more in stores.

Downloaded by Lan Anh Tr?nh (lananh30303@gmail.com)


lOMoARcPSD|16636219

E. Hayley's found success by allowing buyers to customize the color and some features of its appliances before buying
them.
ANSWER: B
53) If Starbucks considers quick-serve restaurants and convenience shops in its competitive frame of reference, then
intended ___________ might be quality, image, experience and variety, while intended ___________ might be
convenience and value.
A. PODs; POPs
B. POPs; PODs
C. substantiators; image variables
D. design variables; personality variables
E. image variables; personality variables
ANSWER: A
54) If a brand is developing an offering with multiple frames of reference, which of the following is NOT advisable?
A. create a combined positioning that addresses all competitors
B. prioritize competitors
C. choose the most important set of competitors to serve as the competitive frame
D. adopt lowest common denominator positioning
E. develop the best possible positioning for each type or class of competitors
ANSWER: D
55) Subway restaurants are positioned as offering healthy, good-tasting sandwiches. When the competitive frame of
reference is quick-serve restaurants like McDonald's, what is the POD?
A. health
B. taste
C. convenience
D. price
E. brand
ANSWER: A
56) Suppliers who are dependable in their on-time delivery, order completeness, and order-cycle time are most likely to
be differentiated based on __________.
A. resilience
B. innovativeness
C. insensitivity
D. reliability
E. expertise
ANSWER: D
57) Suppliers who are better at handling emergencies, product recalls, and inquiries are most likely to be differentiated
based on their __________.
A. innovativeness
B. thoroughness
C. resilience
D. insensitivity
E. reliability
ANSWER: C
58) A supplier creates better information systems, and introduces barcoding, mixed pallets, and other methods of helping
the consumer. The supplier is most likely to be differentiated on its __________.
A. innovativeness
B. reliability
C. insensitivity
D. resilience
E. accuracy
ANSWER: A
59) Subway restaurants are positioned as offering healthy, great-tasting sandwiches. ___________ positioning allows the
brand to create a point-of-parity (POP) on taste and a point-of-difference (POD) on health with respect to quick-serve
restaurants such as McDonald's and Burger King and, at the same time, a POP on health and a POD on taste with respect
to health food restaurants and cafés.
A. Category-based
B. Need-based
C. Noncomparitive
D. Straddle

Downloaded by Lan Anh Tr?nh (lananh30303@gmail.com)


lOMoARcPSD|16636219

E. Price-quality
ANSWER: D
60) Tums claims to have the most acid-reducing components of any antacid. In what way is the brand's category
membership being conveyed?
A. comparing to exemplars
B. relying on the product descriptor
C. announcing category benefits
D. focusing on reliability
E. persuasion based on believability
ANSWER: C
61) The typical approach to positioning is to inform consumers of a brand's category membership before stating its point-
of- __________.
A. parity
B. difference
C. conflict
D. weakness
E. presence
ANSWER: B
62) In which of the following examples is a company communicating category membership
using a product descriptor?
A. Use Zipex for quick and thorough cleaning.
B. Barry's Oats, when you want nutrition and flavor.
C. Clarity offers you the best prices for the best quality.
D. Choose Grissom's for an unparalleled shopping experience.
E. Chloe: All you need for a beautiful you.
ANSWER: B
63) Industrial tools claiming to have durability, and antacids announcing their efficacy convey a brand's category
membership by __________.
A. relying on the product descriptor
B. comparing to exemplars
C. announcing category benefits
D. communicating deliverability variables
E. identifying counter examples
ANSWER: C
64) A well-known car manufacturing company introduces a new hatchback model by describing its distinctive features and
then stressing the speed and safety qualities of the car. Which of the following is the company using to convey its
membership in the hatchback segment?
A. announcing category benefits
B. comparing to exemplars
C. relying on the product descriptor
D. using channel differentiation
E. maximizing negatively correlated attributes
ANSWER: A
65) Which of the following ways to convey a brand's category membership relates to well- known, noteworthy brands in a
category helping a brand specify its category membership?
A. comparing to exemplars
B. communicating deliverability variables
C. identifying counter examples
D. announcing category benefits
E. relying on the product descriptor
ANSWER: A
66) SJC is a new retailer that targets the youth market. SJC needs to make an impression using advertising, and decides
to use funny or irreverent ads to get its point across. Each ad features one of SJC's competitors and conveys an
advantage SJC has over that competitor. Which of the following is the company using to convey its membership in the
retail segment?
A. announcing category benefits
B. comparing to exemplars
C. relying on the product descriptor
D. using channel differentiation

Downloaded by Lan Anh Tr?nh (lananh30303@gmail.com)


lOMoARcPSD|16636219

E. maximizing negatively correlated attributes


ANSWER: B
67) When Tommy Hilfiger was an unknown brand, advertising announced his membership as a great US designer by
associating him with Geoffrey Beene, Stanley Blacker, Calvin Klein, and Perry Ellis, who were recognized members of that
category. Tommy Hilfiger conveyed the brand's category membership by __________.
A. relying on the product descriptor
B. focusing on reliability
C. comparing to exemplars
D. announcing category benefits
E. identifying counter examples
ANSWER: C
68) Ford Motor Co. invested more than $1 billion on a radical new 2004 model called the X- Trainer, which combined the
attributes of an SUV, a minivan, and a station wagon. To communicate its unique position — and to avoid association with
its Explorer and Country Squire models — the vehicle, eventually called Freestyle, was designated a "sports wagon."
According to the given scenario, Ford Motor Co. conveyed their brand's category membership by __________.
A. announcing category benefits
B. identifying counter examples
C. relying on the product descriptor
D. focusing on reliability
E. comparing to exemplars
ANSWER: C
69) Randall Ringer and Michael Thibodeau see ___________ as based on deep metaphors that connect to people's
memories, associations, and stories.
A. cultural branding
B. narrative branding
C. brand journalism
D. emotional branding
E. personal branding
ANSWER: B
70) Which element of a brand story framework do Randall Ringer and Michael Thibodeau relate to the time, place, and
context of the brand story?
A. cast
B. pitch
C. narrative arc
D. setting
E. language
ANSWER: D
71) Which element of a brand story framework do Randall Ringer and Michael Thibodeau relate to the way the narrative
logic unfolds over time, including actions, desired experiences, defining events, and the moment of epiphany?
A. language
B. pitch
C. cast
D. setting
E. narrative arc
ANSWER: E
72) Which element of a brand story framework do Randall Ringer and Michael Thibodeau relate to the authenticating
voice, metaphors, symbols, themes, and leitmotifs?
A. narrative arc
B. context
C. language
D. setting
E. cast
ANSWER: C
73) Which of the following statements about the branding guidelines for a small business is true?
A. A small business must creatively conduct low-cost marketing research.
B. A small business must avoid leveraging secondary brand associations.
C. A small business must separate the well-integrated brand elements to enhance both brand awareness and brand
image.

Downloaded by Lan Anh Tr?nh (lananh30303@gmail.com)


lOMoARcPSD|16636219

D. A small business must disintegrate the brand elements to maximize the contribution of each of the three main sets of
brand equity drivers.
E. A small business must focus on building more than two strong brands based on a number of associations.
ANSWER: A
74) Which of the following is NOT one of the specific branding guidelines for small businesses?
A. Creatively conduct low-cost marketing research.
B. Leverage as many secondary associations as possible.
C. Focus on building one or two strong brands based on one or two key associations.
D. Create buzz and a loyal brand community.
E. Use a trial-and-error approach.
ANSWER: E

Downloaded by Lan Anh Tr?nh (lananh30303@gmail.com)


lOMoARcPSD|16636219

Chapter 12 Addressing Competition and Driving Growth


1) WD40 offers a Smart Straw version of its popular multipurpose lubricant with a built-in straw that pops up for use. This
is an example of which of the following main strategies for growing the core of the business?
A. Make the core of the brand as distinctive as possible.
B. Drive distribution through both existing and new channels.
C. Offer the core product in new formats or versions.
D. Increase costs and revenue.
E. Expand to another geographic region.
ANSWER: C
2) Galaxy chocolate has successfully competed with Cadbury by positioning itself as "your partner in chocolate
indulgence" and featuring smoother product shapes, more refined taste, and sleeker packaging, which represents which
of the following main strategies for growing the core of the business?
A. Make the core of the brand as distinctive as possible.
B. Drive distribution through both existing and new channels.
C. Offer the core product in new formats or versions.
D. Increase costs and revenue.
E. Expand to another geographic region.
ANSWER: A
3) Which of the following brands generates only 6 percent outside North America, with very little of that in fast-growing
emerging markets like India, China, and Brazil, which may represent opportunities for its growth?
A. UPS
B. Under Armour
C. FedEx
D. Nike
E. Adidas
ANSWER: B
4) The market _________ has the largest market share and usually shows the way to other firms in price changes, new-
product introductions, distribution coverage, and promotional intensity.
A. challenger
B. entrant
C. follower
D. nicher
E. leader
ANSWER: E
5) When the total market expands, the market _________ usually gains the most.
A. challenger
B. leader
C. follower
D. nicher
E. entrant
ANSWER: B
6) When a firm looks for new users in groups that might use a product but do not already use the product, the firm is
using the _________ strategy.
A. new-market segment
B. market-penetration
C. geographical-expansion
D. product development
E. diversification
ANSWER: B
7) As the marketing manager for a floor tile manufacturer, Evans Smith is given a target to achieve 500 new customers by
the end of summer. He decides to search the market for probable customers who might use the product but currently do
not. Which of the following strategies is Evans pursuing to increase the market demand for his product?
A. market-penetration strategy
B. new-market segment strategy
C. geographical-expansion strategy
D. needs-assessment strategy
E. consolidation strategy
ANSWER: A

Downloaded by Lan Anh Tr?nh (lananh30303@gmail.com)


lOMoARcPSD|16636219

8) When firms look for new users in groups that have never used the product before, the firm is using the _________
strategy.
A. new-market segment
B. market-penetration
C. geographical-expansion
D. product development
E. diversification
ANSWER: A
9) When Starbucks introduced its Tazo Tea line to bring in new customers who had never gone to Starbucks because they
don't drink coffee, Starbucks was employing a _ _________ strategy.
A. market-penetration
B. new-market segment
C. geographical-expansion
D. niche identification
E. blue-ocean
ANSWER: B
10) Trendz Inc. is a leading brand of fashion clothing and accessories based in Houston. After gaining a strong foothold in
the US, the company wants to foray into foreign markets. The management at Trendz knows that people residing in other
countries are likely to have different tastes and preferences, so they may have to redesign some of their offerings. Which
of the following strategies is Trendz using?
A. market-penetration strategy
B. outsourcing strategy
C. geographic-expansion strategy
D. product differentiation strategy
E. ethnocentric strategy
ANSWER: C
11) A market leader on the look out for more usage from existing customers should focus on increasing the frequency of
consumption and _________.
A. decreasing the product price
B. the product line
C. the amount of consumption
D. decreasing production turnover time
E. diversifying into unrelated markets
ANSWER: C
12) Which of the following marketing strategies requires either identifying additional opportunities to use the brand in the
same basic way or identifying completely new and different ways to use the brand?
A. increasing the amount of consumption
B. decreasing the level of consumption
C. increasing dedication to consumption
D. increasing product innovation
E. increasing frequency of consumption
ANSWER: E
13) When food product companies advertise recipes that use their branded products in entirely different ways, they are
increasing the _________ consumption of the brand.
A. amount of
B. level of
C. dedication to
D. frequency of
E. awareness of
ANSWER: D
14) Oliver, a company that produces different types of olive oil, launched a promotional campaign focusing on the
alternative uses of olive oil. What strategy is the company implementing to expand its total market demand?
A. using the new-market segment strategy to attract new customers
B. improving the current level of product performance
C. advertising new and different applications of the brand
D. protecting its market share
E. using the market-penetration strategy to attract new customers
ANSWER: C

Downloaded by Lan Anh Tr?nh (lananh30303@gmail.com)


lOMoARcPSD|16636219

15) Clorox runs ads stressing the many benefits of its bleach, such as how it eliminates kitchen odors, in order to
generate additional opportunities to use the brand in the same basic way. This is an attempt to _________ of
consumption.
A. increase the amount
B. increase the level
C. increase the perception
D. increase the frequency
E. increase the emotional implications
ANSWER: D
16) Which of the following is the most constructive response a market leader can make when defending its market share?
A. maintain basic cost control
B. innovate continuously
C. provide desired benefits
D. meet challengers with a swift response
E. provide expected benefits
ANSWER: B
17) To satisfy customers, a(n) _________ marketer finds a stated need and fills it.
A. creative
B. responsive
C. anticipative
D. laidback
E. inexperienced
ANSWER: B
18) A(n) _________ marketer looks ahead to needs customers may have in the near future.
A. laidback
B. creative
C. inexperienced
D. responsive
E. anticipative
ANSWER: E
19) A(n) _________ marketer discovers solutions customers did not ask for but to which they enthusiastically respond.
A. laidback
B. responsive
C. inexperienced
D. creative
E. anticipative
ANSWER: D
20) _________ marketers are not just market-driven, they are proactive market-driving firms.
A. Creative
B. Responsive
C. Inexperienced
D. Laidback
E. Anticipative
ANSWER: A
21) Sony is an unusual market leader. It gives its customers new products that they have never asked for (e.g.,
Walkmans, VCRs, video cameras, CDs). This makes Sony a(n) _________ firm.
A. market-driven
B. market-driving
C. operations-driven
D. vision-driven
E. virtually-driven
ANSWER: B
22) Which of the following is true about proactive marketing?
A. A company needs creative anticipation to see the writing on the wall.
B. Proactive companies create new offers to serve unmet and unknown consumer needs.
C. Proactive companies refrain from practicing uncertainty management.
D. A company needs responsive anticipation to devise innovative solutions.
E. Companies are winners when they are extremely risk-averse.
ANSWER: B

Downloaded by Lan Anh Tr?nh (lananh30303@gmail.com)


lOMoARcPSD|16636219

23) _________ defense involves occupying the most desirable market space in the minds of the consumers and making
the brand almost impregnable.
A. Position
B. Flank
C. Preemptive
D. Mobile
E. Contraction
ANSWER: A
24) A marketing manager has planned a strategy that will require the organization to erect outposts to protect its weak
front-running brands. In this _________ defense, the outposts will be central to the organization's new competitive
strategy.
A. position
B. flank
C. preemptive
D. counteroffensive
E. mobile
ANSWER: B
25) _________ defense is an aggressive maneuver where the firm attacks first, perhaps with guerrilla action across the
market and keeps everyone off balance.
A. Position
B. Flank
C. Contraction
D. Preemptive
E. Lateral
ANSWER: D
26) The defensive marketing strategy wherein a stream of new products are introduced by a firm and are announced in
advance are referred to as _________ defense.
A. mobile
B. counteroffensive
C. preemptive
D. contraction
E. flank
ANSWER: C
27) If Microsoft announces plans for a new-product development, smaller firms may choose to concentrate their
development efforts in other directions to avoid head-to-head competition. In this example, Microsoft is employing a
_________ defense strategy.
A. preemptive
B. counteroffensive
C. mobile
D. flank
E. contraction
ANSWER: A
28) In a _________ defense strategy, the market leader can meet the attacker frontally and hit its flank or launch a
pincer movement so that the attacker is forced to pull back to defend itself.
A. position
B. counteroffensive
C. preemptive
D. contraction
E. flank
ANSWER: B
29) After Olay entered and took over the Indian market with its anti-aging lotion, Ponds invested heavily in a similar
product which used only natural ingredients. This is an example of a _________ defense.
A. position
B. flank
C. counteroffensive
D. contraction
E. mobile
ANSWER: C

Downloaded by Lan Anh Tr?nh (lananh30303@gmail.com)


lOMoARcPSD|16636219

30) In a _________ defense, the market leader stretches its domain over new territories that can serve as future centers
for defense and offense.
A. counteroffensive e
B. flank
C. contraction
D. mobile
E. position
ANSWER: D
31) Market broadening and market diversification are likely tactics employed in _________ defense strategies.
A. position
B. flank
C. preemptive
D. counteroffensive
E. mobile
ANSWER: E
32) When "petroleum" companies such as BP sought to recast themselves as "energy" companies, increasing their
research into the oil, coal, nuclear, hydroelectric, and chemical industries, these companies were employing a _________
defense strategy.
A. preemptive
B. counteroffensive
C. mobile
D. flank
E. contraction
ANSWER: C
33) When large companies can no longer defend their full territory, they can launch a _________ defense where they
give up weaker markets and reassign resources to stronger ones.
A. contraction
B. preemptive
C. flank
D. mobile
E. position
ANSWER: A
34) In 2006, Sara Lee spun off products that accounted for a large percentage of the company's revenues, including its
strong Hanes hosiery brand, so it could concentrate on its well-known food brands. In this example, Sara Lee is
employing a _________ defense strategy.
A. preemptive
B. counteroffensive
C. mobile
D. flank
E. contraction
ANSWER: E
35) Market _________ generally have a smaller market share than other players and they are willing to maintain their
share and not rock the boat.
A. challengers
B. leaders
C. nichers
D. rivals
E. followers
ANSWER: E
36) When Starbucks extended its brand into coffee drinks and ice creams to reach customers who did not use its brand, it
was using a(n) _________ strategy.
A. geographical-expansion
B. new-market segment
C. market-penetration
D. larger package size
E. increased frequency
ANSWER: B
37) When Procter & Gamble used Gain and Cheer laundry detergents as outposts to protect a weak front or support a
possible counterattack, they were using a _________ defense.

Downloaded by Lan Anh Tr?nh (lananh30303@gmail.com)


lOMoARcPSD|16636219

A. flank
B. position
C. preemptive
D. counteroffensive
E. mobile
ANSWER: A
38) A firm that is willing to maintain its market share, and not attack the leader and other competitors in an aggressive
bid for further market share, is known as a market _________.
A. challenger
B. leader
C. follower
D. nicher
E. entrant
ANSWER: C
39) For a market challenger, attacking _________ is a high risk but potentially high payoff strategy, which also allows it to
distance itself from other challengers.
A. a firm of its own size
B. the market leader
C. a regional firm
D. an underfinanced firm
E. a poorly performing firm
ANSWER: B
40) In a pure _________ attack, the attacker matches its opponent's product, advertising, price, and distribution.
A. guerilla
B. bypass
C. encirclement
D. frontal
E. flank
ANSWER: D
41) Epic Inc., a firm that produces chairs for offices, uses comparative advertising to inform consumers that its products
offer the same features and quality as the competitor's. This is an example of a(n) _________ attack.
A. guerilla
B. frontal
C. encirclement
D. bypass
E. flank
ANSWER: B
42) The _________ attack can be used when the challenger spots areas where the opponent is underperforming.
A. encirclement
B. frontal
C. flank-geographic
D. backwards-flank
E. guerilla
ANSWER: C
43) Pepsodent launched a new product that could whiten teeth, fight decay, and maintain fresh breath. Observing that
Pepsodent did not focus on the dental sensitivity aspect, Colgate introduced a toothpaste which did all of the above and
also protected sensitive teeth. This is an example of a(n) _________ attack.
A. frontal
B. flank
C. guerrilla
D. encirclement
E. bypass
ANSWER: B
44) A(n) _________ strategy is another name for identifying shifts in market segments that are causing gaps to develop,
then rushing in to fill the gaps and developing them into strong segments.
A. frontal attack
B. flanking
C. encircling
D. counteroffensive

Downloaded by Lan Anh Tr?nh (lananh30303@gmail.com)


lOMoARcPSD|16636219

E. bypass
ANSWER: B
45) The _________ attack attempts to capture a significant share of the consumer market by launching a grand offensive
on several fronts.
A. frontal
B. bypass
C. flank
D. encirclement
E. guerrilla
ANSWER: D
46) Sally Seabrook is an up-and-coming marketing manager for a large department store chain. Ms. Seabrook has
distinguished herself with bold strategies such as launching attacks on her primary competitor from several fronts,
including advertising, new store openings, and new distributor alliances. Which of the following market challenger attack
strategies is Ms. Seabrook using to attack her competition?
A. frontal attack
B. bypass attack
C. guerrilla attack
D. flank attack
E. encirclement attack
ANSWER: E
47) The _________ attack offers the firm an opportunity to diversify into unrelated products, into new geographical
markets, and leapfrogging into new technologies.
A. bypass
B. flank
C. frontal
D. guerrilla
E. encirclement
ANSWER: A
48) Appy Juices, a company that manufactures bottled water, diversified into soft drinks before any of its opponents could
launch a diversified product. This would be an example of a(n) _________ attack.
A. flank
B. bypass
C. guerrilla
D. encirclement
E. frontal
ANSWER: B
49) A marketing manager has decided to "leapfrog" the competition by moving into cutting-edge technologies. This
indirect approach to attacking competition can be characterized as a(n) _________ attack.
A. flank
B. encirclement
C. bypass
D. guerrilla
E. frontal
ANSWER: C
50) Selective price cuts, intense promotional blitzes, and occasional legal action are commonplace in the strategic design
of a(n) _________ attack.
A. frontal
B. flank
C. bypass
D. encirclement
E. guerilla
ANSWER: E
51) When Canon grabbed a large chunk of Xerox's market by introducing desk copiers, it distanced itself from other
challengers by _________.
A. attacking a regional firm
B. attacking a firm of its own size that was underfinanced
C. attacking the market leader
D. attacking a firm of its own size that was not doing the job well
E. acquiring guppies

Downloaded by Lan Anh Tr?nh (lananh30303@gmail.com)


lOMoARcPSD|16636219

ANSWER: C
52) In his article, "Innovative Imitation," Theodore Levitt argues that _________.
A. imitation is wrong and should be punished
B. product imitation might be as profitable as product innovation
C. innovation is not possible without substantial imitation
D. innovation cannot begin unless dissatisfaction with imitation occurs
E. imitation should be against the law because of the intellectual property decision involved
ANSWER: B
53) As a market follower strategy, the _________ duplicates the leader's product and packages and sells it on the black
market or through disreputable dealers.
A. innovator
B. cloner
C. imitator
D. adapter
E. counterfeiter
ANSWER: E
54) Aron, a company manufacturing snack food and soft drinks, replicates its product taste and packaging from Lay's, a
market leader in snack food industry. Later, it sells these imitated products on the black market. This is an example of
_________.
A. cloning
B. imitating
C. counterfeiting
D. adapting
E. innovating
ANSWER: C
55) As a market follower strategy, the _________ emulates the leader's products, name, and packaging, with slight
variations.
A. counterfeiter
B. cloner
C. imitator
D. adapter
E. innovator
ANSWER: B
56) Amani, a company that manufactures cloth for suits, strives to be equal to Armani in product design, brand name,
and product packaging. This is an example of _________.
A. innovating
B. adapting
C. imitating
D. cloning
E. counterfeiting
ANSWER: D
57) Tasteeos, Fruit Rings, and Corn Flakes sell for nearly $1 a box less than leading cereal brands. Which market follower
strategy is being employed by the cereal manufacturer?
A. counterfeiter
B. cloner
C. imitator
D. adapter
E. reverse innovator
ANSWER: B
58) The _________ copies some things from the leader but differentiates on packaging, advertising, pricing, or location.
A. innovate
B. adapt
C. cloner
D. imitator
E. counterfeiter
ANSWER: D
59) As a(n) _________, BurgerJacks, a fast food chain selling low-price burgers, has differentiated itself by setting up
base in a market not yet exploited by McDonald's.
A. cloner

Downloaded by Lan Anh Tr?nh (lananh30303@gmail.com)


lOMoARcPSD|16636219

B. imitator
C. counterfeiter
D. adapter
E. innovator
ANSWER: B
60) The TelePizza chain, which operates in Europe and Latin America, has copied Domino's service model but maintains
differentiation in terms of location. TelePizza is an example of a(n) _________.
A. counterfeiter
B. adopter
C. cloner
D. imitator
E. adapter
ANSWER: D
61) As a market follower, the _________ may choose to sell to different markets, but often it grows into a future
challenger.
A. imitator
B. adapter
C. cloner
D. counterfeiter
E. innovator
ANSWER: B
62) Redo is a market follower in the wristwatch industry. After noticing a shortfall in the market leader's product, Redo
improves its product to suit consumer needs and becomes a challenger for the market leader. This is an example of a(n)
_________.
A. counterfeiter
B. cloner
C. imitator
D. adapter
E. innovator
ANSWER: D
63) An alternative to being a market follower in a large market is to be a leader in a small market. This type of competitor
is called a _________.
A. marketing king
B. market nicher
C. segment king
D. guerilla marketer
E. strategic clone
ANSWER: B
64) A firm that serves small market segments that are not being served by bigger firms is known as a market _________.
A. challenger
B. leader
C. follower
D. nicher
E. entrant
ANSWER: D
65) The market leader strategy ensures high sales volume, whereas the market nicher strategy allows firms to achieve
_________.
A. high margin
B. low margin
C. high promotability
D. medium pricing
E. lower demand
ANSWER: A
66) Which of the following is true about market nichers?
A. They are market followers in small markets.
B. They tend to have high manufacturing costs.
C. Their return on investment exceeds that in larger markets.
D. A nicher achieves high volume as against a mass marketer that achieves high margin.
E. They usually experience long-term losses.

Downloaded by Lan Anh Tr?nh (lananh30303@gmail.com)


lOMoARcPSD|16636219

ANSWER: C
67) The key idea in successful nichemanship is specialization. Which of the following specialists would most closely be
identified with the characterization of being an organization that limits its selling to one customer?
A. end-user specialist
B. vertical-level specialist
C. customer-size specialist
D. specific-customer specialist
E. quality-price specialist
ANSWER: D
68) Newman Inc. is a company that manufactures saddles specifically for horses that race in derbies in the United States
and the United Kingdom. Within this context the firm is exhibiting the role of a _________ specialist.
A. vertical-level
B. customer-size
C. product-line
D. job-shop
E. service
ANSWER: C
69) A market nicher is considered to be a(n) _________ specialist if the firm specializes in producing a certain type of
product or product feature such as Rent-a-Wreck, that rents only "beat-up" cars.
A. end-user
B. vertical-level
C. customer-size
D. channel
E. product-feature
ANSWER: E
70) A job-shop specialist _________.
A. sells only in a certain locality, region, or area of the world
B. customizes its products for individual customers
C. operates at the low- or high-quality ends of the market
D. offers one or more services not available from other firms
E. specializes in serving only one channel of distribution
ANSWER: B
71) A firm that is based in France designs jewelry and takes custom orders from around the world. They do not design
more than 15 pieces of jewelry in a year and ensure that each design uses unique stones and is unique. Such
nichemanship is an example of _________ specialist role.
A. customer-size
B. product
C. product-feature
D. job-shop
E. quality-price
ANSWER: D
72) When a bank takes loan requests over the phone and hand-delivers the money to the customer, it becomes a
_________ specialist.
A. geographic
B. job-shop
C. quality-price
D. channel
E. service
ANSWER: E
73) A niche specialist that specializes in one type of customer, like a value-added reseller that customizes computer
hardware and software for a specific customer segment and earns a price premium in the process, is a(n) _________
specialist.
A. geographic
B. quality-price
C. product-feature
D. end-user
E. channel
ANSWER: D
74) _________ is the period of slow sales growth and nonexistent profits.

Downloaded by Lan Anh Tr?nh (lananh30303@gmail.com)


lOMoARcPSD|16636219

A. Growth
B. Decline
C. Maturity
D. Introduction
E. Stagnancy
ANSWER: D
75) Campbells is a newly established company that specializes in preparing healthy but tasty food for children under the
age of 5. It is incurring huge production costs, nonexistent profits, and slow sales growth. The company is in the
_________ phase of its life cycle.
A. stagnancy
B. introduction
C. maturity
D. decline
E. growth
ANSWER: B
76) _________ is a period of rapid market acceptance and substantial profit improvement.
A. Stagnancy
B. Introduction
C. Maturity
D. Decline
E. Growth
ANSWER: E
77) A dance school in the Bronx teaches professional hip-hop and salsa classes. It is experiencing an increase in student
admissions, which is leading to substantial improvement in profits. The school is going through the _________ phase of
its life cycle.
A. decline
B. stagnancy
C. growth
D. introduction
E. maturity
ANSWER: C
78) _________ is a slowdown in sales growth because the product has achieved acceptance by most potential buyers.
A. Obsolescence
B. Introduction
C. Growth
D. Decline
E. Maturity
ANSWER: E
79) A music school in Boyles Height, LA, specializes in teaching the guitar and the violin. After a spurt in growth and a
few successful years, the school is experiencing a slowdown in sales and stability in its profits due to an increase in
competition. The school is in the _________ stage of its life cycle.
A. introduction
B. growth
C. decline
D. maturity
E. obsolescence
ANSWER: D
80) During the _________ stage of a product's life cycle, sales show a downward drift and profits erode.
A. introduction
B. growth
C. decline
D. obsolescence
E. maturity
ANSWER: C
81) After a couple of years of successful business, an experimental theatre company based in Aurora is unable to sell
tickets for its theatre shows. They have been using profits from previous shows to run the business. The company is in
the _________ phase of its life cycle.
A. maturity
B. obsolescence

Downloaded by Lan Anh Tr?nh (lananh30303@gmail.com)


lOMoARcPSD|16636219

C. introduction
D. growth
E. decline
ANSWER: E
82) According to Peter Golder and Gerald Tellis, a(n) _________ is the first to develop a working model of the product.
A. developer
B. creative pioneer
C. market pioneer
D. product pioneer
E. inventor
ANSWER: D
83) One of the ways to change the course of a brand is to modify the product. Under product modification, _________
improvement adds size, weight, materials, supplements, and accessories that expand the product's performance,
versatility, safety, or convenience.
A. feature
B. quality
C. style
D. size
E. technological
ANSWER: A
84) An alternate way to increase sales volume is to increase the usage rate among users. This can be done by
_________.
A. converting nonusers
B. having consumers use less of the product on each occasion
C. having consumers use the product on more occasions
D. attracting competitors' customers
E. entering new market segments
ANSWER: C
85) _________ is a distribution strategy that can be effectively used during the growth stage of the product life cycle.
A. Building product awareness
B. Phasing out unprofitable outlets
C. Building selective distribution
D. Building intensive distribution
E. Stressing on brand differences
ANSWER: D
86) Which of the following strategies should be adopted by marketers during a recession?
A. increase investment on marketing existing products
B. focus on expanding the customer base and not on the retention of existing customers
C. focus primarily on price reductions and discounts
D. concentrate on communicating the brand value and product quality to consumers
E. stick to the budget allocations adopted during the preceding years
ANSWER: D
87) Benz & Frendz Corp., a manufacturer of high end consumer durables, experienced sluggish sales growth in most of its
product categories during three consecutive quarters of 2009. However, market analysis revealed that its competitors'
sales had also slackened during this period. Analysts pointed out that when all firms are losing sales, it is extremely
important to adopt strategies that are aimed at retaining customers. This led the firm to reduce operation costs while
maintaining product quality. They also revamped their marketing strategy to focus on the values created by their
products. Which of the following can be inferred from the strategies adopted by the firm?
A. The company was trying to protect its market share and continue to operate as a market leader.
B. The company was focusing on geographical expansion.
C. The company was aiming to capture a new market segment.
D. The company was marketing its products amidst an economic downturn.
E. The company was focusing on market penetration.
ANSWER: D

Downloaded by Lan Anh Tr?nh (lananh30303@gmail.com)


lOMoARcPSD|16636219

Chapter 13 Setting Product Strategy


1) A ________ is anything that can be offered to a market to satisfy a want or need, including physical goods, services,
experiences, events, persons, places, properties, organizations, information, and ideas.
A. function
B. product
C. benefit
D. process
E. structure
ANSWER: B
2) A customer judges a product offering by three basic elements: product features and quality, services mix and quality,
and ________.
A. performance
B. utility
C. tangibility
D. price
E. availability
ANSWER: D
3) The five product levels constitute a ________. At each level more customer value is added.
A. product line
B. business model
C. customer value-hierarchy
D. value grid
E. demand chain
ANSWER: C
4) When companies search for new ways to satisfy customers and distinguish their offering from others, they look at the
________ product, which encompasses all the possible augmentations and transformations of the product.
A. consumption
B. expected
C. potential
D. augmented
E. basic
ANSWER: C
5) The way the user performs the tasks of getting and using products and related services is the user's total ________.
A. consumption system
B. consumable system
C. consistent use system
D. augmented system
E. potential system
ANSWER: A
6) Marketers must see themselves as benefit providers. For example, when a shopper purchases new shoes, he or she
expects the shoes to cover his or her feet and allow him or her to walk unobstructed. This is an example of what level in
the consumer-value hierarchy?
A. pure tangible good
B. basic product
C. augmented product
D. potential product
E. generic product
ANSWER: B
7) How a consumer shops for organic foods and how he or she uses and disposes of the product is part of the consumers'
________ that is important for marketers to consider.
A. value proposition
B. consumption system
C. value system
D. quality perception
E. value chain
ANSWER: B
8) The sellers of ________ goods carry a wide assortment to satisfy individual tastes. They must have well-trained
salespeople to inform and advise customers.
A. unsought

Downloaded by Lan Anh Tr?nh (lananh30303@gmail.com)


lOMoARcPSD|16636219

B. specialty
C. convenience
D. heterogeneous shopping
E. generic
ANSWER: D
9) Marketers have traditionally classified products on the basis of three characteristics: ________, tangibility, and use.
A. availability
B. affordability
C. aesthetics
D. durability
E. necessity
ANSWER: D
10) Which of the following are tangible goods that normally survive many uses?
A. generic goods
B. durable goods
C. core benefits
D. convenience goods
E. unsought goods
ANSWER: B
11) Because ________ are purchased frequently, marketers should make them available in many locations, charge only a
small markup, and advertise heavily to induce trial and build preference.
A. nondurable goods
B. durable goods
C. services
D. unsought goods
E. specialty goods
ANSWER: A
12) What types of goods are purchased frequently, immediately, and with minimum effort by the consumers?
A. specialty goods
B. shopping goods
C. unsought goods
D. durable goods
E. convenience goods
ANSWER: E
13) It was sunny when Jenny went to class, but by the time class was over it was raining heavily, so Jenny stopped by the
student store to buy an umbrella before she walked back to her dorm. In this case, the umbrella is an example of a(n)
________ good.
A. impulse
B. specialty
C. homogeneous shopping
D. emergency
E. heterogeneous shopping
ANSWER: D
14) What goods are similar in quality but different enough in price to justify shopping comparisons?
A. emergency goods
B. homogeneous shopping goods
C. heterogeneous shopping goods
D. specialty goods
E. convenience goods
ANSWER: B
15) Products such as insurance, cemetery plots, and smoke detectors are examples of ________ goods that are products
that the consumer does not know about or does not normally think of buying.
A. specialty
B. unsought
C. heterogeneous shopping
D. homogeneous shopping
E. convenience
ANSWER: B

Downloaded by Lan Anh Tr?nh (lananh30303@gmail.com)


lOMoARcPSD|16636219

16) Capital items are long-lasting goods that facilitate developing or managing the finished product. They include two
groups: installations and ________.
A. natural products
B. component materials
C. operating supplies
D. equipment
E. processed materials
ANSWER: D
17) ________ are major purchases and are usually bought directly from the producer with the typical sale preceded by
long negotiation periods.
A. Raw materials
B. Materials and parts
C. Processed materials
D. Capital goods
E. Installations
ANSWER: E
18) Most products are established at one of four performance levels: low, average, high, or superior. For example,
mountain bikes come in a variety of sizes and physical attributes. When a consumer purchases a mountain bike costing
$1,000, he or she expects the bike to perform to specifications and to have a high ________ , meeting the promised
specifications.
A. features
B. conformance quality
C. durability
D. performance quality
E. reliability
ANSWER: D
19) Many products can be differentiated in terms of their ________ , which is its size, shape, or physical structure.
A. form
B. prototype
C. architecture
D. model
E. blueprint
ANSWER: A
20) ________ is the ability of a company to prepare on a large-scale basis individually designed products, services,
programs, and communications.
A. Mass customization
B. Reverse engineering
C. Interoperability
D. Backward compatibility
E. Benchmarking
ANSWER: A
21) Buyers expect products to have high ________ , which is the degree to which all produced units are identical and
meet promised specifications.
A. durability
B. compatibility
C. conformance quality
D. form
E. performance quality
ANSWER: C
22) If the Ford GT is designed to accelerate to 50 miles per hour within 10 seconds, and every Ford GT coming off the
assembly line does this, the model is said to have high ________.
A. reliability
B. conformance quality
C. durability
D. compatibility
E. interoperability
ANSWER: B
23) ________ describes the product's look and feel to the buyer; it has an advantage of creating distinctiveness that is
difficult to copy.

Downloaded by Lan Anh Tr?nh (lananh30303@gmail.com)


lOMoARcPSD|16636219

A. Design
B. Style
C. Durability
D. Conformance
E. Reliability
ANSWER: B
24) Ideal ________ would exist if users could fix the product themselves with little cost in money or time.
A. durability
B. reliability
C. style
D. design
E. reparability
ANSWER: E
25) For ________ to be valued for products like vehicles and kitchen appliances, it should not be associated with an
excessive price premium and the product must not be subject to rapid technological obsolescence.
A. conformance quality
B. performance quality
C. reparability
D. durability
E. style
ANSWER: D
26) When the physical product cannot be easily differentiated, the key to competitive success may lie in adding valued
services and improving their quality. The main service differentiators are ordering ease, delivery, installation, ________,
customer consulting, maintenance, and repair.
A. technology intensity
B. responsiveness
C. ease of use
D. customer training
E. adaptability
ANSWER: D
27) Delivery refers to how well a product or service is brought to the customer. It includes speed, ________, and care
throughout the delivery process.
A. expedience
B. intensity
C. tangibility
D. performance
E. accuracy
ANSWER: E
28) ________ refers to data, information systems, and advice services that the seller offers to their buyers.
A. Sales force relationships
B. Customer relationships
C. Open source technology
D. Customer training
E. Customer consulting
ANSWER: E
29) Differentiating on ________ is important for companies with complex products and becomes an especially good selling
point when targeting technology novices.
A. delivery
B. ordering ease
C. ease of installation
D. customer consulting
E. reparability
ANSWER: C
30) ________ describes the service program for helping customers keep purchased products in good working order.
A. Returns
B. Ordering ease
C. Installation
D. Maintenance and repair
E. Delivery

Downloaded by Lan Anh Tr?nh (lananh30303@gmail.com)


lOMoARcPSD|16636219

ANSWER: D
31) Which of the following actions would result in the elimination of uncontrollable returns of products in the short run?
A. improved handling
B. better packaging
C. improved transportation
D. proper storage
E. cannot be eliminated
ANSWER: E
32) Smith & Adams Poultry has recently upgraded its transactional model such that its customers (restaurants and hotels)
can communicate with its central supply system to indicate purchase volumes, dates, and receive confirmation, through
their computer terminals. This is an example of a company differentiating itself versus competition in terms of ________.
A. customer relationships
B. customer training
C. installation
D. delivery ease
E. ordering ease
ANSWER: E
33) The common denominators of luxury brands are quality and ________.
A. uniqueness
B. practicality
C. global appeal
D. contemporary
E. goodwill
ANSWER: A
34) A winning formula for many ________ brands is craftsmanship, heritage, authenticity, and history, often critical to
justifying a sometimes extravagant price.
A. design
B. nondurable
C. durable
D. ingredient
E. luxury
ANSWER: E
35) The brand promise for ________ is "the product you buy is of highest esteem, based on its timeliness, elegant design
and the high quality, which is derived from the excellence of our craftsmen."
A. Hermés
B. Montblanc
C. Patrón
D. Sub-zero Refrigerators
E. Armani
ANSWER: B
36) ________ has overtaken the United States as the world's largest luxury market; it's forecast that one-third of all high-
end goods will be sold there in the coming years.
A. China
B. India
C. South Africa
D. Brazil
E. South Korea
ANSWER: A
37) Which of the following is NOT one of the guidelines for managing luxury brands?
A. Besides brand names, other brand elements — logos, symbols, packaging, signage — can be important drivers of
brand equity for luxury products.
B. Secondary associations from linked personalities, events, countries, and other entities should be avoided.
C. Luxury brands must employ a premium pricing strategy, with strong quality cues and few discounts and markdowns.
D. All aspects of the marketing program for luxury brands must be aligned to ensure high-quality products and services
and pleasurable purchase and consumption experiences.
E. Luxury brands must carefully control distribution via a selective channel strategy.
ANSWER: B
38) Which company launched Eco-Fina packaging for its water?
A. PepsiCo

Downloaded by Lan Anh Tr?nh (lananh30303@gmail.com)


lOMoARcPSD|16636219

B. Coca-Cola
C. SodaStream
D. PURE
E. Sigg Switzerland
ANSWER: A
39) ________ found a highly creative way to address the problem of proliferating plastic bottles with its "Waste<Less" line
and helped farmers grow cotton with less water for its "Water<Less" line.
A. Calvin Klein
B. Ralph Lauren
C. Ferragamo
D. Levi-Strauss
E. Roberto Cavalli
ANSWER: D
40) A group of products within a product class that are closely related because they perform a similar function, are sold to
the same customer groups, are marketed through the same outlets or channels, or fall within given price ranges is known
as a ________.
A. product type
B. product class
C. need family
D. product variant
E. product line
ANSWER: E
41) A(n) ________ is defined as a distinct unit within a brand or product line distinguishable by size, price, appearance, or
some other attribute.
A. stock-keeping unit
B. inventory turn
C. individual brand
D. product type
E. brand line
ANSWER: A
42) A ________ is the set of all products and items a particular seller offers for sale.
A. product line
B. product mix
C. product extension
D. product system
E. product class
ANSWER: B
43) Happy Home Products produces detergents, toothpaste, bar soap, disposable diapers, and paper products. This
company has a product ________ of five lines.
A. type
B. length
C. class
D. mix
E. width
ANSWER: E
44) Using the ________ level of the product hierarchy to market its soups, Campbell Soups feature the company name
first, then the soup variety on their packaging.
A. product class
B. product-type
C. need-family
D. product-family
E. product-line
ANSWER: E
45) A consumer products firm manufactures and sells over 200 different sizes and varieties of jams and jellies. We can say
that this manufacturer's product mix has high ________.
A. consistency
B. depth
C. intensity
D. range

Downloaded by Lan Anh Tr?nh (lananh30303@gmail.com)


lOMoARcPSD|16636219

E. width
ANSWER: B
46) The ________ of the product mix refers to how closely related the various product lines are in end use, production
requirements, distribution channels, or some other way.
A. consistency
B. depth
C. width
D. length
E. composition
ANSWER: A
47) The ________ of the product mix refers to the total number of items in the mix.
A. width
B. length
C. depth
D. breadth
E. range
ANSWER: B
48) In offering a product line, companies normally develop a ________ and modules that can be added to meet different
customer requirements.
A. convenience item
B. flagship product
C. staple item
D. potential product
E. basic platform
ANSWER: E
49) The ________ of a product mix refers to how many variants are offered of each product in the line.
A. width
B. length
C. depth
D. consistency
E. height
ANSWER: C
50) Which of the following is a benefit of product mapping?
A. studying market matrices
B. integrating target markets
C. identifying market segments
D. educating consumers
E. integrating target matrices
ANSWER: C
51) Product-line analysis provides information for two key decision areas: product-line length and ________.
A. product-class composition
B. product-mix pricing
C. product pricing
D. popular pricing
E. product-need family
ANSWER: B
52) What occurs when any company lengthens its product line beyond its current range?
A. market overreach
B. brand dilution
C. product adaptations
D. cannibalization
E. line stretching
ANSWER: E
53) A company positioned in the "middle" market introduces a lower-priced product line. What type of line-stretching is
this?
A. home stretch
B. up-market stretch
C. down-market stretch
D. maintenance stretch

Downloaded by Lan Anh Tr?nh (lananh30303@gmail.com)


lOMoARcPSD|16636219

E. two-way stretch
ANSWER: C
54) Moving ________ carries risks. The new brand can cannibalize core brand sales and lower the core brand's quality
image.
A. up-market
B. two ways
C. one way
D. down-market
E. out-market
ANSWER: D
55) Companies may wish to implement a(n) ________ to achieve more growth, to realize higher margins, or simply to
position themselves as full-line manufacturers.
A. up-market stretch
B. rebranding plan
C. outsourcing strategy
D. disintermediation policy
E. vertical integration strategy
ANSWER: A
56) A manufacturer of hiking boots looks at data that indicate that their subsegment of the market called "serious hiker" is
declining and is predicted to decline into the future. The firm decides to enter the "low-price" segment with its new items.
This is an example of a firm's ________ to reach a new market.
A. down-market stretch
B. up-market stretch
C. two-way stretch
D. marketing research
E. disintermediation
ANSWER: A
57) Marriott Corporation now contains hotels and motels from the "budget" end of the consumer spectrum to the
"premium" end with their JD Marriott flagship locations. This is an example of a firm that successfully performed a(n)
________ to reach more consumers and ventures that are more profitable.
A. upstream integration
B. two-way stretch
C. up-market stretch
D. down-market stretch
E. downstream integration
ANSWER: B
58) A product line can also be lengthened by adding more items within the present range. There are several motives for
line filling. Which of the following is one of them?
A. responding to senior management wishes
B. responding to consumer wishes
C. reaching for incremental profits
D. reaching for incremental capacity
E. responding to sales-force demands
ANSWER: C
59) If line filling is overdone, it could result in ________ and customer confusion.
A. sales paralysis
B. manufacturing inefficiencies
C. cannibalization
D. disintermediation
E. ineffective management
ANSWER: C
60) Price-setting logic must be modified when the product is part of a product mix. In that case, the firm searches for a
set of prices that ________ profits on the total mix.
A. is ineffective on total
B. has no effect on total
C. maximizes
D. minimizes
E. capitalizes upon
ANSWER: C

Downloaded by Lan Anh Tr?nh (lananh30303@gmail.com)


lOMoARcPSD|16636219

61) Companies normally develop ________ rather than single products and require sellers to establish perceived quality
differences between price steps within it.
A. product mix
B. captive products
C. product lines
D. optional products
E. average products
ANSWER: C
62) When shopping for tires for your automobile, you notice that the manufacturer you have selected has tires for your
car priced low, average, and high, based upon performance and features. This is an example of what type of product-mix
pricing?
A. two-part pricing
B. product-line pricing
C. captive product pricing
D. market pricing skimming
E. price discrimination
ANSWER: B
63) Some service firms often engage in ________ , consisting of a fixed fee plus a variable usage fee.
A. pure bundling
B. pure pricing
C. mixed pricing
D. captive pricing
E. two-part pricing
ANSWER: E
64) In ________ , the seller offers goods both individually and in bundles and often charges less for the "bundle" than for
the individual products.
A. pirating pricing
B. captive pricing
C. two-part pricing
D. pure bundling
E. mixed bundling
ANSWER: E
65) Purchasers of theatre tickets receive a 20 percent discount if they purchase and pay for the full season at one time.
This is an example of what type of product-mix pricing?
A. mixed bundling
B. pure bundling
C. cross-promotion
D. captive pricing
E. two-part pricing
ANSWER: A
66) When Mercedes successfully introduced its C-Class cars at $30,000 without injuring its ability to sell other Mercedes
cars for $100,000, it was an example of a successful ________.
A. two-way stretch
B. line pruning
C. down-market stretch
D. up-market stretch
E. choice reduction
ANSWER: C
67) Gillette uses ________ pricing when it offers razor handles at a low cost, but places a high markup on the
replacement razor blades that fit in the handles.
A. optional feature
B. two-part
C. by-product
D. product line
E. captive-product
ANSWER: E
68) McDonald's restaurants inside Walmart stores and Starbucks inside Super Targets are examples of ________, whose
main advantages are that the products can or may be convincingly positioned by virtue of the associated brands.
A. cooperative marketing

Downloaded by Lan Anh Tr?nh (lananh30303@gmail.com)


lOMoARcPSD|16636219

B. cross-promotion
C. retail co-branding
D. ingredient branding
E. feature promotion
ANSWER: C
69) Betty Crocker cake mixes using Hershey syrup in its mixes and "Lunchables" lunch combinations with Taco Bell tacos
are examples of what special type of branding?
A. family branding
B. ingredient co-branding
C. co-branding
D. generic branding
E. individual branding
ANSWER: B
70) The main advantage of co-branding is that a product may be convincingly positioned by virtue of the ________
involved.
A. branding synergy
B. increased advertising dollars
C. multiple brands
D. bundled package
E. pure bundling
ANSWER: C
71) The potential disadvantages of ________ are the risks and lack of control from becoming aligned with another brand
in the consumers mind. Consumer expectations about the level of involvement and commitment are likely to be high, so
unsatisfactory performance could be very negative for the brands involved.
A. co-branding
B. cannibalization
C. vertical integration
D. disintermediation
E. brand stretching
ANSWER: A
72) ________ branding is a special case of co-branding involving creating brand equity for materials, components, or
parts that are necessarily contained within other branded products.
A. Cross
B. Ingredient
C. Equity
D. Family
E. Generic
ANSWER: B
73) We define packaging as all the activities of designing and producing the container for a product. This includes up to
three levels of material: primary package, secondary package, and ________ package.
A. retailer
B. design
C. shipping
D. consumer
E. supplier
ANSWER: C
74) Which of the following factors is one of the contributors to the growing use of packaging as a marketing tool?
A. consumption aid
B. consumer affluence
C. consumer influence
D. conformance qualities
E. brand identification
ANSWER: B
75) ________ are formal statements of expected product performance by the manufacturer.
A. Insurance
B. Warranties
C. Bonds
D. Invoices
E. Balance sheets

Downloaded by Lan Anh Tr?nh (lananh30303@gmail.com)


lOMoARcPSD|16636219

ANSWER: B
76) Many sellers offer either general or specific guarantees. Guarantees reduce the buyer's ________ risk.
A. actual
B. perceived
C. real
D. implied
E. stated
ANSWER: B
77) Guarantees are most effective in two situations. The first is when the company or products are not well known. The
second is when the product's quality is ________ to competition.
A. not known
B. different
C. inferior
D. equivalent
E. superior
ANSWER: E

Downloaded by Lan Anh Tr?nh (lananh30303@gmail.com)


lOMoARcPSD|16636219

Chapter 14 Designing and Managing Services


1) Which of the following is a characteristic of a service?
A. The service is essentially tangible.
B. The service does not result in the ownership of anything.
C. The service's production is majorly tied to a physical product.
D. Services are typically produced and consumed at different times.
E. A client's presence is not required for rendering a service.
ANSWER: B
2) Which of the following is an example of a pure tangible good?
A. massage
B. shampoo
C. e-mail
D. restaurant meal
E. air travel
ANSWER: B
3) To which of the following categories of services does a cell phone belong?
A. major service with accompanying minor services
B. major service with accompanying minor goods
C. pure service
D. pure tangible good
E. tangible good with accompanying services
ANSWER: E
4) A computer falls into the ________ category of service mix.
A. pure tangible good
B. tangible good with accompanying services
C. hybrid
D. major service with accompanying minor goods
E. pure service
ANSWER: B
5) Which of the following is an example of a hybrid service?
A. teaching
B. car
C. restaurant meal
D. soap
E. air travel
ANSWER: C
6) A flight with complementary drinks is an example of a ________.
A. major service with accompanying minor goods and services
B. pure service
C. pure tangible good
D. tangible good with accompanying services
E. hybrid
ANSWER: A
7) Which of the following is an example of a pure service?
A. air travel
B. psychotherapy
C. baby oil
D. a laptop
E. a restaurant meal
ANSWER: B
8) Which of the following is true for services?
A. All services are people-based, while goods are equipment-based.
B. Service providers can be both for-profit or nonprofit.
C. All service companies follow the same process to deliver their services.
D. The client's presence is a hindrance during the service delivery process.
E. Service providers develop similar marketing programs for personal services and business services.
ANSWER: B

Downloaded by Lan Anh Tr?nh (lananh30303@gmail.com)


lOMoARcPSD|16636219

9) When banks try to make their positioning strategies tangible through the ________ dimension, they make sure
the exterior and interior have clean lines, the layout of the desks and the traffic flow are planned carefully, and waiting lines
are not overly long.
A. people
B. symbols
C. equipment
D. place
E. communication material
ANSWER: D
10) Some services require that the client be present to conduct the service. Which of the following is an example of
such a service?
A. pest control
B. furniture polishing
C. surgery
D. car repairing
E. tax services
ANSWER: C
11) Services high in ________ qualities have characteristics that the buyers can evaluate before purchase.
A. search
B. experience
C. credence
D. privacy
E. storing
ANSWER: A
12) Services high in ________ qualities have characteristics that the buyer can evaluate after purchase.
A. privacy
B. experience
C. credence
D. search
E. stock
ANSWER: B
13) Services high in ________ qualities are those services that have characteristics the buyer normally finds hard to
evaluate even after consumption.
A. trial
B. search
C. experience
D. privacy
E. credence
ANSWER: E
14) Which one of the following would be considered high in credence qualities?
A. an interior of a house
B. a restaurant
C. a haircut
D. psychotherapy
E. a computer
ANSWER: D
15) Which one of the following is highest in search qualities?
A. a play at a theater
B. a meal at a restaurant
C. a haircut
D. psychotherapy
E. a computer
ANSWER: E
16) Unlike physical products, services cannot be seen, tasted, felt, heard, or smelled before they are bought. This is
known as the ________ aspect of services.
A. inseparability
B. intangibility
C. variability
D. perishability

Downloaded by Lan Anh Tr?nh (lananh30303@gmail.com)


lOMoARcPSD|16636219

E. heterogeneity
ANSWER: B
17) Which of the following will most help service providers overcome the limitation of intangibility of services when
positioning itself?
A. using brand symbols
B. sharing services
C. working with larger groups
D. cultivating nonpeak demand
E. creating a service blueprint
ANSWER: A
18) A brand that is action-oriented and causes consumers to engage in physical actions appeals to the ________
dimension of brand experience.
A. sensory
B. affective
C. behavioral
D. intellectual
E. social
ANSWER: C
19) BRZ Shoes targets the youth market with vibrant, visually appealing ads in modern styles. BRZ ads appeal to the
________ dimension of brand experience.
A. sensory
B. affective
C. behavioral
D. intellectual
E. social
ANSWER: A
20) BRZ Shoes positions itself as a young, adventurous brand. Its logo and brand communication try to give
consumers a feeling of excitement and bravery. These ads appeal to the ________ dimension of brand experience.
A. sensory
B. affective
C. behavioral
D. intellectual
E. social
ANSWER: B
21) ________ refer(s) to logos, symbols, characters, and slogans that service providers use in order to make the
service and its key benefits more tangible.
A. Brand engagement
B. Brand orientation
C. Brand elements
D. Brand loyalty
E. Brand equity
ANSWER: C
22) Which of the following is true regarding services?
A. Services are typically produced, stored, and then consumed.
B. Services are generally low in experience and credence qualities.
C. Service providers develop similar marketing programs for personal services and business services.
D. There is less risk associated with the purchase of services than with the purchase of goods.
E. Provider-client interaction is a special feature of services marketing.
ANSWER: E
23) The fact that services are typically produced and consumed simultaneously is known as the ________ aspect of
services.
A. perishability
B. intangibility
C. heterogeneity
D. inseparability
E. variability
ANSWER: D
24) Service quality depends on who provides them, when and where they are provided, and to whom they are
provided. Thus, services are highly ________.

Downloaded by Lan Anh Tr?nh (lananh30303@gmail.com)


lOMoARcPSD|16636219

A. inseparable
B. tangible
C. variable
D. perishable
E. intangible
ANSWER: C
25) Which of the following will help a service provider overcome the limits imposed by the inseparability of services?
A. using differential pricing
B. working with larger customer groups
C. providing complementary services
D. concentrating on physical evidence and presentation
E. standardizing the service process
ANSWER: B
26) Which of the following steps will help service firms to increase their quality control?
A. standardizing the service-performance process
B. providing complementary services to customers
C. giving personnel authority in handling situations
D. adopting differential pricing
E. cultivating nonpeak demand
ANSWER: A
27) In order to map out the service process, the points of customer contact, and the evidence of service from the
customer's point of view, service firms should develop a ________.
A. marketing plan
B. service floor plan
C. Gantt chart
D. business plan
E. service blueprint
ANSWER: E
28) Jake had an appointment at the doctor's office, but couldn't make it on time due to traffic. By the time he arrived,
the doctor had already begun with the next patient. This illustrates the ________ of services.
A. variability
B. heterogeneity
C. perishability
D. intangibility
E. homogeneity
ANSWER: C
29) When a theater sells matinee movie tickets at low prices, it aims to shift some demand from the peak to the off-
peak period. What is the strategy that the theater is said to be adopting?
A. It is providing complementary services.
B. It is increasing peak-time efficiency.
C. It is using linear pricing.
D. It is using differential pricing.
E. It is sharing services.
ANSWER: D
30) A travel agency offers weekend discounts for car rentals. What can be deduced from this?
A. It is increasing peak-time efficiency.
B. It is using differential pricing.
C. It is using linear pricing.
D. It is cultivating peak demand.
E. It is providing complementary services.
ANSWER: B
31) In which of the following cases is a service provider trying to increase nonpeak demand?
A. A fine dining restaurant is promoting a breakfast service in addition to its popular lunch and dinner options.
B. An upscale restaurant has a cocktail lounge where customers can wait until a table is ready.
C. AXA Bank set up automated teller machines so that its customers could avoid standing in line.
D. Chesterton College hired part-time teachers as enrollment increased significantly.
E. Big department stores usually hire extra staff to handle the rush during the holiday season.
ANSWER: A

Downloaded by Lan Anh Tr?nh (lananh30303@gmail.com)


lOMoARcPSD|16636219

32) The Caesar Park Hotel generally caters to business customers during the week, but has now decided to promote
mini-vacation weekends for non-business customers as well. What is the Caesar Park Hotel trying to do?
A. It is implementing premium pricing.
B. It is trying to cultivate nonpeak demand.
C. It is promoting complementary services.
D. It is putting reservation systems in place.
E. It is implementing differential pricing.
ANSWER: B
33) Cocktail lounges in restaurants are examples of ________.
A. differential pricing
B. cultivating nonpeak demand
C. complementary services
D. reservation systems
E. shared services
ANSWER: C
34) Which of the following is an example of a complementary service?
A. Big department stores usually hire extra staff to handle the rush during the holiday season.
B. The Caesar Park Hotel generally caters to business customers during the week, but has now decided to promote mini-
vacation weekends for non-business customers as well.
C. More paramedics are on hand to assist physicians during times when emergency admissions are highest.
D. AXA Bank set up automated teller machines so that its customers could avoid standing in line.
E. Chesterton College hires part-time teachers when enrollment goes up.
ANSWER: D
35) Walmart has decided to hire extra clerks during the holiday season. It is said to be ________.
A. matching its supply with the existing demand
B. generating nonpeak demand
C. increasing its customer participation
D. sharing its services
E. facilitating for its future expansion
ANSWER: A
36) Kaya, a chain of skin clinics, requests each new visitor to fill in their own details on a printed form. This is a step
in ________.
A. increasing its peak-time efficiency
B. creating nonpeak demand
C. increasing consumer participation
D. sharing its services
E. facilitating its future expansions
ANSWER: C
37) Customers today want separate prices for each service element, and they also want the right to select the
elements they want. Customers are said to be pressing for ________ services.
A. complementary
B. perishable
C. variable
D. unbundled
E. shared
ANSWER: D
38) Often, a service problem arises from a customer's lack of understanding or ineptitude. Which of the following can
help to minimize customer failures?
A. giving customers exclusive primary service packages
B. redesigning processes to simplify service encounters
C. using differential pricing and shared services
D. minimizing service intangibility
E. working with more customers at the same time
ANSWER: B
39) ________ marketing refers to the normal work of preparing, pricing, distributing, and promoting the service to
customers.
A. Interactive
B. Internal
C. External

Downloaded by Lan Anh Tr?nh (lananh30303@gmail.com)


lOMoARcPSD|16636219

D. Promotional
E. Direct
ANSWER: C
40) ________ marketing refers to training and motivating employees to serve customers well.
A. External
B. Promotional
C. Direct
D. Internal
E. Interactive
ANSWER: D
41) ________ marketing describes the employees' skill in serving the client.
A. External
B. Internal
C. Promotional
D. Direct
E. Interactive
ANSWER: E
42) Susanna wanted to check out a new salon in her locality, so she decided to go for a hair styling session. She was
very happy with the way her stylist connected and related with her concerns. She is judging the service based on its
________.
A. promotional marketing
B. functional quality
C. technical quality
D. search qualities
E. external marketing
ANSWER: B
43) In which of the following does a customer respond to the technical quality of a service?
A. Sara's preferred hair stylist is some miles away, but Sara goes to him because his styles suit her looks.
B. Kathy tries out a new restaurant every week because she likes to experience the variety.
C. Bill has gone to the same chiropractor for the past fifteen years because he is friendly and takes the time to listen to Bill.
D. Ray avoids going to the bank as far as possible because the manager is rude and unhelpful.
E. Alex has no interest in theater, but goes often because her best friend loves plays.
ANSWER: A
44) In which of the following does a customer respond to the functional quality of a service?
A. Sara's preferred hair stylist is some miles away, but Sara goes to him because his styles suit her looks.
B. Kathy tries out a new restaurant every week because she likes to experience the variety.
C. Bill has gone to the same chiropractor for the past fifteen years because he is friendly and takes the time to listen to Bill.
D. Ray's mechanic charges high prices for service, but his work is good and worth the price.
E. Alex has no interest in theater, but goes often because her best friend loves plays.
ANSWER: C
45) Firms have decided to raise fees and lower service for those customers who barely pay their way and to coddle
big spenders to retain their patronage as long as possible. This is an example of organizing customers by ________.
A. retailer convenience
B. profit tiers
C. psychographic characteristics
D. social influence
E. customer preference
ANSWER: B
46) Charles Schwab's best customers are instantly directed to customer service representatives, while other
customers have to wait longer. Charles Schwab is trying to ________.
A. monitor its service systems
B. empower the customers
C. retain the patronage of profitable customers
D. increase consumer participation
E. standardize the service-performance process
ANSWER: C
47) The ________ rates the various elements of the service bundle and identifies required actions.
A. company performance analysis
B. voice of customer measurement

Downloaded by Lan Anh Tr?nh (lananh30303@gmail.com)


lOMoARcPSD|16636219

C. customer factor measurement


D. importance-performance analysis
E. customer importance analysis
ANSWER: D
48) Customers who view a service as homogeneous ________.
A. only patronize a preferred service provider
B. judge services on the basis of the providers
C. care less about the provider than about the price
D. pick a service provider based on functional attributes
E. opt for the service with the highest price, irrespective of quality
ANSWER: C
49) Belling Hotels provides a complimentary breakfast buffet to all guests. This is an example of a ________.
A. primary service package
B. service interface
C. service support
D. service frequency
E. secondary service feature
ANSWER: E
50) When the recent recession hit, Zappos decided to stop offering complimentary overnight shipping to first-time
buyers and offer it to repeat buyers only, which is an example of the ________ best practice of top service companies.
A. strategic concept
B. profit tiers
C. top-management commitment
D. high standards
E. monitoring systems
ANSWER: B
51) Although Brenda previously used the US Postal Service because it offered better prices on package shipping, she
now uses only FedEx, because it gives her the facility of shipping from any FedEx location 24 hours a day. Which of the
following factors led to Brenda's customer switching behavior?
A. inconvenience
B. pricing
C. response to service failure
D. ethical problems
E. involuntary switching
ANSWER: A
52) Angela switched to a new hair stylist after getting a bad haircut from her previous stylist. This is an example of
which of the following factors leading to customer switching behavior?
A. service encounter failure
B. core service failure
C. response to service failure
D. involuntary switching
E. competition
ANSWER: B
53) Dean recently had a BRZ broadband connection installed. However, the connection was poor and he didn't get the
quality he required. He contacted BRZ about the problem, but the company did not solve his problem. Dean switched to
Blue Broadband. Which of the following was the cause of Dean's switching behavior?
A. involuntary switching
B. competition
C. response to service failure
D. service encounter failures
E. inconvenience
ANSWER: C
54) Which of the following represents an aspect of the Empathy dimension of SERVQUAL?
A. providing service as promised
B. readiness to respond to customers' requests
C. giving customers individual attention
D. making customers feel safe in their transactions
E. visually appealing facilities
ANSWER: C

Downloaded by Lan Anh Tr?nh (lananh30303@gmail.com)


lOMoARcPSD|16636219

55) MGC Inc. organizes a company-wide picnic once a year. The organizers arranged for the best possible menu
thinking that food would be a priority, but the staff were disappointed because the activities were very poorly planned.
What kind of a gap is apparent here?
A. gap between perceived service and expected service
B. gap between service delivery and external communications
C. gap between service-quality specifications and service delivery
D. gap between management perception and service-quality specification
E. gap between consumer expectation and management perception
ANSWER: E
56) Which of the following is an example of a gap between management perception and the service-quality
specifications?
A. The college brochure showed state-of-the-art classrooms, but when the visitors walked in, they saw peeling walls and
dull lighting.
B. A nurse visits a patient to show care, but the patient interprets this as an indication that something is very wrong.
C. The hotel administrators think that guests want better food, but guests are more concerned with the courtesy of the
waiters.
D. A service center manager has asked his subordinates to provide fast service, but has not specified a time for the service
to be performed.
E. Customer service representatives are asked to give ample time to each customer, but must serve a minimum of 50
customers a day.
ANSWER: D
57) The customer service representatives at a call center have been asked to handle each call within five minutes. A
recent customer survey by the company revealed that customers appreciate it when employees take the time to answer
their questions fully and listen to their grievances. What kind of service gap is apparent here?
A. gap between perceived service and expected service
B. gap between service delivery and external communications
C. gap between service-quality specifications and service delivery
D. gap between management perception and service-quality specification
E. gap between consumer expectation and management perception
ANSWER: D
58) The customer service representatives at a call center have been asked to handle each call within five minutes. At
the same time, they have been asked to answer all customer queries in detail and provide appropriate solutions. What kind
of service gap is apparent here?
A. gap between perceived service and expected service
B. gap between service delivery and external communications
C. gap between service-quality specifications and service delivery
D. gap between management perception and service-quality specification
E. gap between consumer expectation and management perception
ANSWER: C
59) Which of the following is an example of a gap between service delivery and external communications?
A. The employees at GBL have been asked to take time to listen to customers, but they must serve them fast as well.
B. Amanda chose to shop at Alison's Fashions because the store's Web site offered on-the-spot alterations. However, when
she did buy a dress, she had to wait a week to get it altered.
C. Customers at LUX appreciate the personalized services the salespeople offer, but do not like the store design.
D. Clearwater Spa attendants are well-trained in massage therapy and the services they offer, but customers rarely return
because they don't like the attendants' impersonal service.
E. When sales dropped, Styx modernized its stores in order to retain customers, but didn't realize that the product quality
was the main problem.
ANSWER: B
60) Ellen came across an ad for a new restaurant that promised authentic French cuisine. When she ate there,
however, she was disappointed to find that the food was mediocre and not very authentic. Which of the following gaps of
service performance does this demonstrate?
A. gap between service-quality specifications and service delivery
B. gap between perceived service and expected service
C. gap between service delivery and external communications
D. gap between consumer expectation and management perception
E. gap between management perception and service-quality specification
ANSWER: C

Downloaded by Lan Anh Tr?nh (lananh30303@gmail.com)


lOMoARcPSD|16636219

61) When Aaron went to his doctor for his annual checkup, he was asked to undergo a number of tests. Although the
doctor assured Aaron that the tests were routine, Aaron thinks that the doctor is hiding a grave problem from him. What
kind of a gap is apparent here?
A. gap between perceived service and expected service
B. gap between service delivery and external communications
C. gap between service-quality specifications and service delivery
D. gap between management perception and service-quality specification
E. gap between consumer expectation and management perception
ANSWER: A
62) The most important determinant of service quality is ________ which refers to the ability to perform the
promised service dependably and accurately.
A. empathy
B. assurance
C. responsiveness
D. reliability
E. tangibles
ANSWER: D
63) The customer service representatives at G.K.'s customer service center must know as much about the products as
possible, so that they can help customers solve their difficulties without sounding hesitant or unsure of themselves. Which
of the following determinants of service quality are they being asked to demonstrate?
A. responsiveness
B. assurance
C. empathy
D. reliability
E. tangibility
ANSWER: B
64) Josh gets his bike serviced at Dean's Garage even though there's another garage much closer to home. He
prefers Dean's because the work is usually done quickly and the staff try to solve the issues with the bike as soon as
possible. Dean's Garage excels at which of the following five determinants of service quality?
A. reliability
B. responsiveness
C. assurance
D. empathy
E. tangibles
ANSWER: B
65) The ________ determinant of service quality refers to the knowledge and courtesy of employees and their ability
to convey trust and confidence in the service they provide.
A. conscientious
B. assurance
C. empathy
D. reliability
E. responsiveness
ANSWER: B
66) Leo's manager has asked him and his teammates to demonstrate caring towards customers. They are instructed
to learn customers' names, and use names while interacting with them. Repeat customers should get special attention, and
the team members should remember their preferences and habits. The manager is asking the team to be ________.
A. assuring
B. candid
C. empathetic
D. reliable
E. responsive
ANSWER: C
67) Gloria goes to the same bagel shop every morning because the workers there remember her name and
remember her order. They always make her feel welcome. The employees of this bagel shop excel at which of the following
determinants of service quality?
A. reliability
B. responsiveness
C. assurance
D. empathy

Downloaded by Lan Anh Tr?nh (lananh30303@gmail.com)


lOMoARcPSD|16636219

E. tangibles
ANSWER: D
68) The zone of ________ is a range where a service dimension is deemed satisfactory, anchored by the minimum
level consumers are willing to accept and the level they believe can and should be delivered.
A. immunity
B. tolerance
C. reliability
D. assurance
E. flexibility
ANSWER: B
69) According to the dynamic process model, two different types of expectations have opposite effects on perceptions
of service quality. One of these is that ________ overall service quality.
A. increasing customer expectations of what the firm will deliver improve the perceptions of
B. increasing customer expectations of what the firm will deliver decrease the perceptions of
C. decreasing customer expectations of what the firm should deliver decrease the perceptions of
D. decreasing customer expectations of what the firm will deliver improve the perceptions of
E. increasing customer expectations of what the firm should deliver improve the perceptions of
ANSWER: A
70) When customers calculate the perceived economic benefits of a continuously provided service in relationship to
the economic costs, they are gauging the ________ equity.
A. private
B. brand
C. payment
D. customer-service
E. product-service
ANSWER: C
71) A(n) ________ interface refers to any place at which a company seeks to manage a relationship with a customer,
whether through people, technology, or some combination of the two.
A. customer-service
B. product-customer
C. tangible user
D. attentive user
E. crossing-based
ANSWER: A
72) Regular maintenance and repair costs are known as ________ costs.
A. service contract
B. out-of-pocket
C. fixed
D. facilitating services
E. value-augmentation
ANSWER: B
73) A product's purchase cost plus the discounted cost of maintenance and repair less the discounted salvage value
gives the ________ cost.
A. service warranty
B. out-of-pocket
C. life-cycle
D. facilitating services
E. value-augmentation
ANSWER: C
74) Expensive equipment manufacturers not only install the equipment but also train the staff and undertake the
maintenance and repair activities of the equipment. By doing so, they are providing ________.
A. payment equity
B. value-augmenting services
C. differential pricing
D. facilitating services
E. a primary service package
ANSWER: D
75) When Johnson Controls reached beyond its climate control equipment and components business to manage
integrated facilities by offering products and services that optimize energy use, it was said to be providing ________.

Downloaded by Lan Anh Tr?nh (lananh30303@gmail.com)


lOMoARcPSD|16636219

A. facilitating services
B. a primary service package
C. value-augmenting services
D. service contracts
E. service warranties
ANSWER: C
76) When sellers agree to provide free maintenance and repair services for a specified period of time at a specified
contract price, they are offering ________.
A. a complementary service
B. payment equity
C. a service blueprint
D. differential pricing
E. an extended warranty
ANSWER: E

Downloaded by Lan Anh Tr?nh (lananh30303@gmail.com)


lOMoARcPSD|16636219

Chapter 15: Introducting New Market Offerings


1) Which of the following is most closely related with the organic growth of an organization?
A. acquiring a product or service brand
B. entering new marketplaces
C. increasing the operational profitability
D. increasing productivity of employees
E. developing new products from within
ANSWER: E
2) New-to-the-world products are ________.
A. low-cost products designed to obtain an edge in highly competitive markets
B. new product enhancements that supplement established products
C. new versions of an existing product that has been less successful
D. new products that create an entirely new market
E. existing products that are targeted to new geographical markets
ANSWER: D
3) Which of the following firms is most likely to seek radical innovation?
A. a traditional publishing company
B. a small scale FMCG company
C. a firm that procures and markets wheat
D. a high-tech firm in telecommunications
E. a firm that sells mineral water
ANSWER: D
4) Most new-product activities are devoted to ________.
A. changing the target markets
B. developing new-to-the-world products
C. introducing backward integration
D. improving existing products
E. changing the existing market dynamics
ANSWER: D
5) Which of the following is the best example of a new-to-the-world product?
A. Walmart, the retail giant, opens new stores in an underdeveloped African country.
B. Pestorica, a publishing company, decides to launch a new sports magazine.
C. Tata Motors, an Indian automobile company, acquires Jaguar to extend its business.
D. An Asian company licenses a US apparel brand name though the brand is not familiar in Asia.
E. Kids-Med, a company that produces childcare products, launches a non-contact thermometer.
ANSWER: E
6) Most established companies focus on ________ innovation when they aim to enter new markets by tweaking existing
products, or they want to stay one step ahead in the market by using variations on a core product.
A. incremental
B. continuous
C. spontaneous
D. radical
E. competitive
ANSWER: A
7) Jordan's firm enters new markets by tweaking products for new customers, uses variations on a core product to stay
one step ahead of the market, and creates interim solutions for industry-wide products. In other words, it uses ________.
A. disruptive technologies
B. incremental innovation
C. complex innovations
D. discontinuous innovations
E. radical innovations
ANSWER: E
8) It has been observed that most new products have shorter product life cycles. What is the reason for this?
A. Most new products do not use technology.
B. Most new products are not backed by a marketable idea.
C. New products do not get adequate management support.
D. Social and governmental constraints lead to this failure.
E. Rivals quickly copy products that are successful.
ANSWER: E

Downloaded by Lan Anh Tr?nh (lananh30303@gmail.com)


lOMoARcPSD|16636219

9) Which of the following strategies for new-product development incorporates buyers' preferences in the final design of
the product?
A. quality function deployment
B. market leadership
C. cost leadership
D. incremental innovation
E. disruptive technology
ANSWER: A
10) Pager, a simple personal device for short messages, became famous in the 1990s. Troveron Communications launched
a pager in the early twenty-first century. Due to the introduction of mobile phones and text messaging, the pager industry
was on a decline. The company's innovations were not well received by the market and the product was a failure. Which
of the following is the most likely reason for the product's failure in this case?
A. poor launch timing of the product
B. a small and fragmented target market
C. high cost of development
D. social and economic constraints
E. hasty product development
ANSWER: A
11) After creating a product prototype, a company tests it within the firm to see how it performs in different applications.
The company refines the prototype to correct the mistakes found in in-house testing. What should be the next step?
A. commercializing the product
B. performing concept testing
C. conducting beta testing with customers
D. creating a marketing strategy for the product
E. performing business analysis
ANSWER: C
12) Identify a shortcoming of giving the responsibility to develop new products to the product managers of a company.
A. They would not be familiar with the industry standards.
B. Product managers would not have an operational focus.
C. Product managers are often busy managing existing lines.
D. They will find it difficult to gain support from employees.
E. Product managers are less likely to use participative management.
ANSWER: C
13) ________ teams are cross-functional groups charged with developing a specific product or business.
A. Virtual
B. Venture
C. Fundamental
D. Elemental
E. Transitory
ANSWER: B
14) Intercom Inc., together with its subsidiaries, primarily engages in the generation, transmission, and distribution of
electric power in the United States. The company observes that its growth has stagnated over a period of two years. In
an attempt to promote growth, it considers adding new features to the existing products and introducing a few new
products. The company forms a committee consisting of three top executives, one of the production mangers, a few
operational managers, and a representative of the HR department to generate ideas. This team is called a(n) ________
team.
A. virtual
B. venture
C. fundamental
D. elemental
E. transitory
ANSWER: B
15) A team formed at Intercom Inc. to generate ideas for new products conducts frequent meetings and engages in
activities such as mind mapping and brainstorming. Most of the meetings are conducted at informal locations away from
office. These workplaces are called ________.
A. skunkworks
B. idea funnels
C. research centers
D. stage-gate systems

Downloaded by Lan Anh Tr?nh (lananh30303@gmail.com)


lOMoARcPSD|16636219

E. contextual bases
ANSWER: A
16) A ________ divides the innovation process into stages with a checkpoint at the end of each stage.
A. reverse assumption analysis technique
B. skunkworks system
C. stage-gate system
D. spiral development process
E. new-product department technique
ANSWER: C
17) ________ are informal workplaces, sometimes garages, where intrapreneurial teams attempt to develop new
products.
A. Stage-gate systems
B. Skunkworks
C. Funnels
D. Opportunity spaces
E. Research centers
ANSWER: B
18) Which of the following questions is answered during the business analysis of ideas?
A. Will this product meet our profit goals?
B. Have we got a technically and commercially sound product?
C. Can we find a cost-effective, affordable marketing strategy?
D. Can this product meet sales expectations?
E. Can we find a good concept consumers say they would try?
ANSWER: D
19) During which of the following stages in new product development decision making do managers analyze if they can
find a good thought consumers say they would try?
A. idea generation
B. idea screening
C. business analysis
D. concept development and testing
E. product development and testing
ANSWER: D
20) A ________ process recognizes the value of returning to an earlier stage to make improvements before moving
forward.
A. spiral development
B. reactive development
C. market testing
D. proactive development
E. concept testing
ANSWER: A

21) ________ means inviting the Internet community to help create content or software, often with prize money or a
moment of glory involved.
A. Stage-gating
B. Cocreation
C. Microstocking
D. Buzzing
E. Crowdsourcing
ANSWER: E
22) Companies such as Edison Nation and the Big Idea Group have sprung up to tap into ________ possibilities, often
combining its own design, branding, engineering, and sales teams with online participants, forming a community for
devising new products.
A. stage-gating
B. cocreation
C. microstocking
D. buzzing
E. crowdsourcing
ANSWER: E
23) Surgeons and ER nurses would be considered ________ for surgical equipment.

Downloaded by Lan Anh Tr?nh (lananh30303@gmail.com)


lOMoARcPSD|16636219

A. venture agents
B. internal customers
C. buzz agents
D. lead users
E. connectors
ANSWER: D
24) Many people believe that customer focus does not help to create better, or new, products. This belief rests on which
of the following ideas?
A. Such new product developments decrease the chances of success.
B. Customers do not consider the cost of design when making suggestions.
C. Customers are at times unaware of what they really want.
D. It increases the cost of testing a product or service.
E. Having too much customer focus leads to a negative brand image.
ANSWER: C
25) The ________ technique for stimulating creativity lists all the characteristics of an object and then modifies each to
try to arrive at a new idea.
A. reverse assumption analysis
B. lateral marketing
C. attribute listing
D. forced relationship
E. morphological analysis
ANSWER: C
26) The ________ technique used for stimulating creativity identifies a problem and then considers the dimension, the
medium, and the power source.
A. attribute listing
B. reverse assumption analysis
C. mind mapping
D. lateral marketing
E. morphological analysis
ANSWER: E
27) In an attempt to improve the product, a company that manufactures screwdrivers discusses the modification of each
attribute, such as replacing the wooden handle with plastic, providing torque power, adding different screw heads, and so
on. This creativity technique is called ________.
A. attribute listing
B. mind mapping
C. morphological analysis
D. lateral analysis
E. reverse analysis
ANSWER: C
28) Electro Locomotors (EL) Inc. engages in the manufacture, distribution, and marketing of custom vehicles in Brazil.
Bono, a hybrid car brand that the company produces, is a market leader in the hybrid vehicles segment. With the
increased global focus on nature-friendly vehicles and governmental subsidies for companies that manufacture hybrid
vehicles, competition has increased for flex-fuel cars like Bono. The heads of the R&D team at EL are in a creative session
to generate ideas for improvements to Bono. They consider each part of the car and discuss the possibilities of
improvements in each of them. Identify the creativity technique used here.
A. attribute listing
B. forced relationship analysis
C. morphological analysis
D. reverse assumption analysis
E. mind mapping
ANSWER: A
29) ________ works by listing all the normal assumptions about an entity and then turning them around.
A. Lateral marketing
B. Attribute listing
C. Reverse assumption analysis
D. Forced relationships
E. Morphological analysis
ANSWER: C

Downloaded by Lan Anh Tr?nh (lananh30303@gmail.com)


lOMoARcPSD|16636219

30) The ________ creativity technique begins with a thought that is written down, then proceeds to develop other
thoughts that are linked to the first thought, and finally a new idea materializes by associating all of the thoughts.
A. mind mapping
B. contextual analysis
C. attribute listing
D. reverse assumption analysis
E. morphological analysis
ANSWER: A
31) Increasingly, new-product ideas arise from ________ that combines two product concepts or ideas to create a new
offering.
A. reverse assumption analysis
B. lateral marketing
C. attribute listing
D. forced relationships
E. morphological analysis
ANSWER: B
32) A group of college graduates decides to start a business. Though they are knowledgeable in various business
domains, they are unable to arrive at a valuable business idea. They decide to search for ideas in a structured manner.
They meet and start discussing everyone's ideas. Each idea is recorded and then the thoughts that come up in relation to
the ideas are written down and discussed. This process helps them to finalize a business plan. What technique is used
here?
A. morphological analysis
B. forced relationship analysis
C. reverse assumption analysis
D. attribute listing
E. mind mapping
ANSWER: E
33) Morboro Energy Inc. is a large chain of fuel stations in Europe. The company decided to expand its business by
incorporating restaurants and automobile service stations as a part of its fuel outlets. This is an example of ________.
A. lateral marketing
B. brand extension
C. market development
D. internal marketing
E. brand personification
ANSWER: A
34) A ________ error occurs when the company dismisses a good idea.
A. probability
B. performance
C. double counting
D. DROP
E. GO
ANSWER: D
35) The stage in the new product process that occurs first and has a pass ratio of 1:4 is the ________ stage.
A. idea screening
B. product development
C. test marketing
D. product soft launch
E. concept testing
ANSWER: A
36) A(n) ________ is a possible product the company might offer to the market.
A. test brand
B. alpha product
C. beta version product
D. product idea
E. product concept
ANSWER: D
37) A(n) ________ is an elaborated version of a product idea expressed in consumer terms.
A. test brand
B. alpha product

Downloaded by Lan Anh Tr?nh (lananh30303@gmail.com)


lOMoARcPSD|16636219

C. beta product
D. business schedule
E. product concept
ANSWER: E
38) ________ means presenting the product concept to target consumers, symbolically or physically, and getting their
reactions.
A. Perceptual mapping
B. Brand-positioning mapping
C. Brand attribute mapping
D. Concept development
E. Concept testing
ANSWER: E
39) Modern firms use the ________ tool to design products on a computer and then produce rough models to show
potential consumers for their reactions.
A. morphological analysis
B. rapid prototyping
C. concept testing
D. perceptual mapping
E. conjoint analysis
ANSWER: B
40) Brown & Smith Inc. engages in the design, development, making, and retail selling of designer jewelry in North
America. Before approving a new design, the company draws it on a computer and then produces models to show
potential consumers and get their reactions. This allows the company to analyze the possible customer reaction. Identify
the concept testing method used here.
A. conjoint analysis
B. perceptual mapping
C. virtual reality testing
D. rapid prototyping
E. digital fabrication
ANSWER: D
41) Consumer preferences for alternative product concepts can be measured through ________, a method for deriving
the utility values that consumers attach to varying levels of a product's attributes.
A. concept testing
B. perceptual mapping
C. gap level analysis
D. conjoint analysis
E. morphological analysis
ANSWER: D
42) With ________, respondents see different hypothetical offers formed by combining varying levels of the attributes,
then rank the various offers.
A. gap level analysis
B. conjoint analysis
C. perceptual mapping
D. concept testing
E. morphological analysis
ANSWER: B
43) Universal Services Inc. provides communication services to residential and business customers in rural and small
urban communities primarily in northern England. The company offers services such as local and long distance voice,
data, and Internet and broadband product offerings. The company, in an attempt to increase the attractiveness of its
offerings, decides to provide special voice and data packages to its customers. The company designs eight different packs
that offer varying voice and data benefits to customers. The company then asks a few of its customers to rank the packs
in order to choose two best packs. Which of the following testing methods is being used in this scenario?
A. virtual reality testing
B. conjoint analysis
C. perceptual mapping
D. product fabrication
E. rapid prototyping
ANSWER: B
44) The trade-off approach may be easier to use when there ________.

Downloaded by Lan Anh Tr?nh (lananh30303@gmail.com)


lOMoARcPSD|16636219

A. are many variables


B. are only a few alternatives
C. are observers that are biased
D. are possible offers that are limited
E. is only one highly desirable solution
ANSWER: A
45) ________ is a "hybrid" data collection technique that combines self-explicated importance ratings with pair-wise
trade-off tasks.
A. Sales wave analysis
B. Simulative data testing
C. Adaptive conjoint analysis
D. Business analysis
E. Exponential data collection
ANSWER: C
46) ________ exhibit replacement cycles dictated by physical wear or obsolescence associated with changing style,
features, and performance.
A. Frequently purchased products
B. High-moving goods
C. Inexpensive products
D. Commodity products
E. Infrequently purchased products
ANSWER: E
47) To estimate replacement sales, management must research the product's ________, the number of units that fail in
year one, two, three, and so on.
A. survival-age distribution
B. life-cycle ratio
C. obsolescence rate
D. business turnover ratio
E. product-performance usage
ANSWER: A
48) ________ contribution lists the changes in income to other company products caused by the introduction of a new
product.
A. Supplementary
B. Dragalong
C. Gross
D. Cumulative
E. Net
ANSWER: A
49) The highest loss a project can create is called ________.
A. rapid prototyping income
B. payback income
C. maximum investment exposure
D. incremental yearly exposure
E. cannibalized income
ANSWER: C
50) The job of translating target customer requirements into a working prototype is helped by a set of methods referred
to as ________.
A. quality function deployment
B. quality control processes
C. rapid prototyping
D. marketing control
E. control system formation
ANSWER: A
51) Which of the following methodologies takes the list of desired customer attributes (CAs) generated by market
research and turns them into a list of engineering attributes (EAs) that engineers can use?
A. quality control processes
B. quality function deployment
C. rapid prototyping
D. marketing control

Downloaded by Lan Anh Tr?nh (lananh30303@gmail.com)


lOMoARcPSD|16636219

E. control system formation


ANSWER: B
52) ________ testing is a customer test that involves placing the product within the firm to see how it performs in
different applications.
A. Beta
B. Research
C. Sales-wave
D. Alpha
E. Simulated
ANSWER: D
53) In consumer-goods market testing, the company seeks to estimate four variables. These four variables are: trial, first
repeat, adoption, and ________.
A. guaranteed response
B. price sensitivity
C. purchase frequency
D. usage convenience
E. preferential treatment
ANSWER: C
54) A large FMCG company decides to test market Kora, a new brand of face cleanser, to be launched soon. The company
initially distributes a few free samples to some prospective consumers. Later it offers the product to the customers at a
discounted price and observes that not only more than seventy percent of the customers are purchasing it but the same
number are also satisfied using it. The company keeps using this process 3-4 times to obtain a correct count of the
number of people purchasing the product repeatedly. Which of the following testing methods is being used here?
A. simulated testing
B. controlled testing
C. full test marketing
D. parallel testing
E. sales-wave research
ANSWER: E
55) Which of the following is an advantage of sales-wave research?
A. It gives surprisingly accurate results on advertising effectiveness.
B. It allows the brand to gain favorable shelf position.
C. It guarantees immediate brand recognition and high sales volume.
D. It can be implemented quickly with a fair amount of security.
E. It easily creates a buzz in the minds of customers.
ANSWER: D
56) Which of the following is usually referred to as a full-blown test market?
A. an internal focus group
B. a discussion group
C. a country
D. a city or a few cities
E. a laboratory
ANSWER: D
57) The least costly way of consumer-goods market testing is ________.
A. simulated test marketing
B. controlled test marketing
C. a few test markets
D. sales-wave research
E. test marketing in 25 percent of the country
ANSWER: D
58) Which of the following products is most likely to undergo alpha and beta testing?
A. food products
B. industrial goods
C. consumer products
D. commodities
E. FMCG products
ANSWER: B

Downloaded by Lan Anh Tr?nh (lananh30303@gmail.com)


lOMoARcPSD|16636219

59) During ________ testing, the company's technical people observe how customers use the product, a practice that
often exposes unanticipated problems of safety and servicing and alerts the company to customer training and servicing
requirements.
A. beta
B. research
C. sales-wave
D. alpha
E. simulated
ANSWER: A
60) For a frequently purchased new product, the seller estimates repeat sales as well as first-time sales. A high rate of
repeat purchasing means customers ________.
A. value price more than differentiation
B. do not support innovation on brands
C. value differentiation more than price
D. prefer personalized products rather than standard ones
E. are satisfied with the product
ANSWER: E
61) Poga International, a multinational beverage corporation identifies that one of its competitors is launching an apple
flavored drink. The company decides to launch an apple flavor brand along with its competitor. What timing strategy is
used here?
A. first entry
B. blunt entry
C. parallel entry
D. late entry
E. exchange entry
ANSWER: C
62) Some firms might delay the launch of their products until after the competitor has borne the cost of educating the
market. Such an entry is called ________ entry.
A. strategic payoff
B. parallel
C. balancing
D. late
E. compensating
ANSWER: D
63) A(n) ________ is any good, service, or idea that is perceived as new, no matter how long its history.
A. commodity
B. innovation
C. adoption
D. invention
E. novel product
ANSWER: B
64) Adopters of new products move through five stages. These stages begin with ________ and follow with interest,
evaluation, trial, and adoption.
A. awareness
B. investigation
C. trial
D. innovation
E. diffusion
ANSWER: A
65) Identify the stage in the consumer adoption process in which the customer is stimulated to seek information about
the innovation.
A. awareness
B. adoption
C. interest
D. evaluation
E. trial
ANSWER: C

Downloaded by Lan Anh Tr?nh (lananh30303@gmail.com)


lOMoARcPSD|16636219

66) Robert has heard about the latest cell phone from LG Electronics that features Wi-Fi hotspot technology. Wi-Fi hotspot
is a technology that allows users to share mobile internet with other Wi-Fi enabled devices. Robert visits the company's
Web site to read more about the phone. Robert is in the ________ stage of consumer-adoption process.
A. interest
B. awareness
C. evaluation
D. trial
E. adoption
ANSWER: A
67) Armordo is a famous vacuum cleaner brand in Africa. Clara has heard of Armordo and she knows that the product is a
success. She has started considering whether or not to buy the vacuum cleaner. What stage of the consumer-adoption
process is Clara in?
A. awareness
B. adoption
C. evaluation
D. trial
E. interest
ANSWER: C
68) Which of the following is the mental step in the consumer-adoption process where the customer starts considering
whether to try an innovation?
A. adoption
B. trial
C. awareness
D. evaluation
E. interest
ANSWER: D
69) People who are technology enthusiasts, venturesome, and who enjoy tinkering with new products and mastering their
intricacies are called ________.
A. innovators
B. early adopters
C. early majority
D. investigators
E. experimenters
ANSWER: A
70) Troma Inc. is a famous manufacturer of cookware that follows a traditional distributor- retailer system to distribute its
products. The company abstains from the use of automated supply chain management (SCM) systems mainly due to the
fear of unknown. However, rapidly escalating operational costs and inefficiencies have made it necessary for the company
to implement an SCM system. The company goes for a big-bang installation of SCM system to become more competitive
and cost effective. Identify the adopter group to which Troma belongs.
A. innovator
B. early adopter
C. early majority
D. laggard
E. late majority
ANSWER: D
71) Jason is a technology enthusiast who is happy to conduct alpha and beta testing and report on early weaknesses of
consumer electronics products. He is most likely to fall in the ________ adopter category.
A. late majority
B. early
C. early majority
D. innovator
E. laggard
ANSWER: D
72) Which of the following characteristics is closely associated with late majority adopter groups?
A. superior technological knowledge
B. low price sensitiveness
C. opinion leadership
D. deliberate pragmatism
E. high risk aversion

Downloaded by Lan Anh Tr?nh (lananh30303@gmail.com)


lOMoARcPSD|16636219

ANSWER: E
73) Joseph, a student of Columbia University, finds many of his classmates have purchased an iPad tablet from Apple. The
iPad, launched a few months before has been identified as a very useful product and many students in the US have rated
it highly. Considering all these, Joseph also decides to purchase an iPad. Which of the following is the adopter group to
which Joseph belongs?
A. early adopter
B. innovator
C. late majority
D. laggard
E. early majority
ANSWER: E
74) Five characteristics influence the rate of adoption of an innovation. One of these is ________.
A. marketing expertise
B. relative advantage
C. packaging attractiveness
D. government regulations
E. place of value exchange
ANSWER: B
75) ________ is the effect one person has on another's attitude or purchase probability.
A. Sharing power
B. Collaborative power
C. Market influence
D. Brand power
E. Personal influence
ANSWER: E
76) Relative advantage of an innovation refers to the degree to which ________.
A. it matches the values and experiences of the individuals
B. it is difficult to understand or use
C. it can be tried on a limited basis
D. it appears superior to existing products
E. the benefits of use are observable or describable to others
ANSWER: D
77) The degree to which the beneficial results of an innovation's use are observable or describable to others is called
________.
A. divisibility
B. communicability
C. compatibility
D. relative advantage
E. plainness
ANSWER: B
78) Which of the following terms refers to the degree to which the innovation can be tried on a limited basis?
A. compatibility
B. relative advantage
C. divisibility
D. communicability
E. complexity
ANSWER: C

Downloaded by Lan Anh Tr?nh (lananh30303@gmail.com)


lOMoARcPSD|16636219

Chap 16: Developing Pricing Strategies and Programs


1) When consumers examine products, they often compare an observed price to an internal price they remember. This is
known as a(n) ________ price.
A. markup
B. reference
C. market-skimming
D. accumulated
E. target
ANSWER: B
2) ________ price refers to what the consumers feel the product should cost.
A. Fair
B. Typical
C. Usual discounted
D. List
E. Maximum retail
ANSWER: A
3) While shopping at the mall, Jane was asked by one of the sales representatives at the cosmetics counter to try out a
new lipstick that her company was test marketing. The company representative asks her how much she would be willing
to pay for the lipstick. After trying it out, Jane is of the opinion that $5 is just the right price for it. What type of a
reference price is Jane using?
A. usual discounted price
B. fair price
C. maximum retail price
D. last price paid
E. historical competitor price
ANSWER: B
4) The reservation price, the maximum that most consumers will pay for a given product, is known as the ________ price.
A. expected future
B. usual discounted
C. upper-bound
D. typical
E. historical competitor
ANSWER: C
5) Which of the following is NOT one of the possible consumer reference prices?
A. typical price
B. actual future price
C. last price paid
D. expected future price
E. upper-bound price
ANSWER: B
6) A company decided to conduct a market survey for its new MP3 player that the company had priced at $150. In the
survey, 95 percent of participants said that the maximum they would pay for the MP3 player is $100. This is an example
of which of the following possible consumer reference prices?
A. historical competitor price
B. expected future price
C. usual discounted price
D. upper-bound price
E. last price paid
ANSWER: D
7) The minimum price that most consumers would pay for a given product is known as the ________ price.
A. everyday low
B. usual discounted
C. fair
D. typical
E. lower-bound
ANSWER: E
8) A company has developed the prototype of a mobile phone that it plans to launch in the next few months. The phone
comes equipped with the most advanced technological features. As part of its test marketing efforts, the company allows
customers to examine and use the prototype and also gathers feedback regarding product features and price. The results

Downloaded by Lan Anh Tr?nh (lananh30303@gmail.com)


lOMoARcPSD|16636219

of this test marketing effort show that customers are willing to pay at least $500, considering the phone's various
features. As such, the company has discovered customers' ________.
A. last paid price
B. expected future price
C. lower-bound price
D. upper-bound price
E. typical price
ANSWER: C
9) Many consumers are willing to pay $100 for a perfume that contains $10 worth of scent because the perfume is from a
well-known brand. What kind of pricing is the company depending on?
A. going-rate pricing
B. image pricing
C. market-skimming pricing
D. target pricing
E. markup pricing
ANSWER: B
10) Pricing cues such as sale signs and prices that end in 9 are more influential when ________.
A. customers have substantial knowledge about prices
B. customers purchase the particular item regularly
C. product quality is standardized
D. product designs vary over time
E. prices do not vary from time to time
ANSWER: D
11) Which of the following is the first step in setting a pricing policy?
A. selecting a pricing method
B. selecting the pricing objective
C. determining demand
D. estimating cost
E. analyzing competitors' costs, prices, and offers
ANSWER: B
12) After determining its pricing objectives, what is the next logical step a firm should take in setting its pricing policy?
A. It should analyze its competitors' costs, prices, and offers.
B. It should select its pricing method.
C. It should select its final price.
D. It should determine the demand for its product.
E. It should estimate the cost of its product.
ANSWER: D
13) A firm that is plagued with overcapacity, intense competition, or changing consumer desires would do better if it
pursues ________ as its major objective.
A. market skimming
B. product-quality leadership
C. survival
D. profit maximization
E. market penetration
ANSWER: C
14) After estimating the demand and costs associated with alternative prices, a company has chosen to price its product
in such a way that it gains the highest rate of return on its investment. The company is looking to ________.
A. maximize market share
B. skim the market
C. become a product-quality leader
D. survive in the market
E. maximize current profit
ANSWER: E
15) Companies who believe that higher sales volume leads to lower unit costs and higher long-run profits are attempting
to ________.
A. maximize their market share
B. skim the market
C. become a product-quality leader
D. merely survive in the market

Downloaded by Lan Anh Tr?nh (lananh30303@gmail.com)


lOMoARcPSD|16636219

E. maximize their current profits


ANSWER: A
16) A company that is looking to maximize its market share would do well to follow ________ pricing.
A. markup
B. market-penetration
C. market-skimming
D. survival
E. target-return
ANSWER: B
17) A market-penetration pricing strategy is most suitable when ________.
A. a low price slows down market growth
B. production and distribution costs fall with accumulated production experience
C. a high price dissuades potential competitors from entering the market
D. the market is characterized by inelastic demand
E. a low price encourages actual competition
ANSWER: B
18) When a company introduces a product at a high price and then gradually drops the price over time, it is pursuing a
________ strategy.
A. market-penetration pricing
B. market-skimming pricing
C. value-pricing
D. switching cost
E. loss-leader pricing
ANSWER: B
19) When Apple introduced its iPhone, it was priced at $599. This allowed Apple to earn the maximum amount of revenue
from the various segments of the market. Two months after the introduction, the price had come down to $399. What
kind of a pricing did Apple adopt?
A. loss-leader pricing
B. market-penetration pricing
C. market-skimming pricing
D. target-return pricing
E. value pricing
ANSWER: C
20) Market skimming pricing makes sense under all the following conditions, EXCEPT if ________.
A. a sufficient number of buyers have a high current demand
B. the unit costs of producing a small volume are high enough to cancel the advantage of charging what the traffic will
bear
C. the high initial price does not attract more competitors to the market
D. consumers are likely to delay buying the product until its price drops
E. the high price communicates the image of a superior product
ANSWER: D
21) Companies that aim to ________ strive to be affordable luxuries.
A. survive in the market
B. partially recover their costs
C. maximize their market share
D. pursue value pricing
E. be product-quality leaders
ANSWER: E
22) Starbucks, Aveda, and BMW have been able to position themselves within their categories by combining quality,
luxury, and premium prices with an intensely loyal customer base. These companies are employing a ________ strategy.
A. market-skimming
B. market-penetration
C. survival
D. market share maximization
E. product-quality leadership
ANSWER: E
23) The first step in estimating demand is to ________.
A. analyze competitors' cost
B. select a pricing method

Downloaded by Lan Anh Tr?nh (lananh30303@gmail.com)


lOMoARcPSD|16636219

C. understand what affects price sensitivity


D. calculate fixed costs
E. decipher the experience curve
ANSWER: C
24) Consumers are less price sensitive ________.
A. to high cost items
B. when they frequently change their buying habits
C. when there are more substitutes
D. when there are more competitors
E. when they do not readily notice higher prices
ANSWER: E
25) Consumers are less price sensitive when ________.
A. price is only a small part of the total cost spent on the product over its lifetime
B. they perceive the higher prices to be unjustified
C. they change their buying habits regularly
D. there are many substitutes and competitors in the market
E. they are buying high-cost items
ANSWER: A
26) If demand hardly changes with a small change in price, the demand is said to be ________.
A. strained
B. marginal
C. inelastic
D. flexible
E. unit elastic
ANSWER: C
27) If demand changes considerably, with a small change in price, the demand is said to be ________.
A. unit elastic
B. elastic
C. inelastic
D. marginal
E. strained
ANSWER: B
28) If consumers were largely indifferent to a $0.05 increase in the price of a gallon of milk, the price rise is said to fall
within customers' ________.
A. price indifference band
B. experience curve
C. arm's-length price
D. learning curve
E. net price index
ANSWER: A
29) JJ pays overhead each month, including his company's bills for rent, heat, interest, and salaries, which are examples
of ________ costs.
A. total
B. average
C. activity-based
D. variable
E. fixed
ANSWER: E
30) Which of the following is true regarding price elasticity?
A. The higher the elasticity, the lesser is the volume growth resulting from a 1 percent price reduction.
B. Within the price indifference band, price changes have little or no effect on demand.
C. If demand is elastic, sellers will consider increasing the price.
D. Price elasticity does not depend on magnitude and direction of the contemplated price change.
E. When demand is inelastic, sellers should lower prices in order to increase total revenue.
ANSWER: B
31) Costs that do not vary with production levels or sales revenue are known as ________ costs.
A. overhead
B. variable
C. average

Downloaded by Lan Anh Tr?nh (lananh30303@gmail.com)


lOMoARcPSD|16636219

D. opportunity
E. total
ANSWER: A
32) Costs that differ directly with the level of production are known as ________ costs.
A. fixed
B. overhead
C. opportunity
D. target
E. variable
ANSWER: E
33) When ConAgra foods decided to cut $250 million in costs to return to a $1 price point (after sales dropped as a result
of raising prices $0.25 to cover higher commodity costs), it was using ________.
A. target costing
B. experience-curve pricing
C. ceiling pricing
D. the learning curve
E. promotional price elasticities
ANSWER: A
34) ________ cost is the cost per unit at that level of production; it equals total costs divided by production.
A. Target
B. Average
C. Marginal
D. Opportunity
E. Fixed
ANSWER: B
35) The decline in the average cost of production with accumulated production experience is called the ________.
A. demand curve
B. supply chain
C. learning curve
D. value chain
E. indifference curve
ANSWER: C
36) Experience-curve pricing ________.
A. assumes competitors are weak followers
B. allows products to project a high quality image
C. is applicable only to manufacturing costs
D. focuses on reducing fixed costs
E. is generally risk-free
ANSWER: A
37) Deducting the desired profit margin from the price at which a product will sell, given its appeal and competitors'
prices, is known as ________.
A. overhead costing
B. target costing
C. activity-based costing
D. benefit analysis
E. estimate costing
ANSWER: B
38) Competitors are most likely to react to a price change when ________.
A. the firm has a weak value proposition
B. the firm enjoys a monopoly
C. there are few competing firms
D. the product is heterogeneous
E. buyers have limited information
ANSWER: C
39) Which of the following is the most elementary pricing method?
A. value pricing
B. going-rate pricing
C. markup pricing
D. target-return pricing

Downloaded by Lan Anh Tr?nh (lananh30303@gmail.com)


lOMoARcPSD|16636219

E. perceived-value pricing
ANSWER: C
40) Despite its weaknesses, markup pricing remains popular for which of the following reasons?
A. Sellers can determine demand much more easily than they can estimate costs.
B. By tying the price to cost, the pricing task becomes more sophisticated.
C. When all firms in the industry use markup pricing, price competition flourishes.
D. Sellers take advantage of buyers when the latter's demand becomes acute.
E. Many people feel that cost-plus pricing is fairer to both buyers and sellers.
ANSWER: E
41) A manufacturer has invested $750,000 in a new product and wants to set a price to earn a 15 percent ROI. The cost
per unit is $18 and the company expects to sell 50,000 units in the first year. Calculate the company's target-return price
for this product.
A. $18.10
B. $18.23
C. $20.25
D. $20.70
E. $25.50
ANSWER: C
42) An umbrella manufacturing company's fixed costs are $275,000. The variable cost per unit is $5 and each umbrella is
sold at $10. How many units should the firm sell in order to break even?
A. 1,819
B. 5,500
C. 18,000
D. 27,500
E. 55,000
ANSWER: E
43) ________ pricing takes into account a host of inputs, such as the buyer's image of the product performance, the
channel deliverables, the warranty quality, customer support, and attributes such as the supplier's reputation,
trustworthiness, and esteem.
A. Perceived-value
B. Value
C. Going-rate
D. Auction-type
E. Markup
ANSWER: A
44) The key to perceived-value pricing is to ________.
A. reengineer the company's operations
B. deliver more unique value than competitors
C. adopt subtle marketing tactics compared to competitors
D. deliver more value but at a lower cost
E. invest heavily in advertising in order to convey superior value
ANSWER: B
45) ________ pricing is a matter of reengineering the company's operations to become a low-cost producer without
sacrificing quality.
A. Value
B. Going-rate
C. Auction-type
D. Markup
E. Perceived-value
ANSWER: A
46) A retailer who utilizes a(n) ________ policy charges a constant low price with little or no price promotions and special
sales.
A. everyday low pricing
B. high-low pricing
C. low cost
D. going-rate pricing
E. auction-type pricing
ANSWER: A

Downloaded by Lan Anh Tr?nh (lananh30303@gmail.com)


lOMoARcPSD|16636219

47) Matt's retail store offers all products at $2 less than its competitors. The store never runs promotional campaigns or
offers special discounts. Matt's retail store is following a(n) ________ pricing policy.
A. auction-type
B. target-plus
C. everyday low
D. high-low
E. going-rate
ANSWER: C
48) Everyday low pricing is most suitable if ________.
A. consumers are willing to perform activities such as clip coupons to avail of discounts
B. consumers tend to associate price with quality
C. customers are insensitive to changes in price
D. the cost of conducting frequent sales and promotions is high
E. consumers have sufficient time to find the best prices
ANSWER: D
49) In ________ pricing, the firm bases its price largely on competitor's prices.
A. going-rate
B. auction-type
C. markup
D. target-return
E. perceived-value
ANSWER: A
50) Which of the following auctions is characterized by one seller and many buyers?
A. Walrasian auctions
B. ascending bid auctions
C. closed auctions
D. sealed-bid auctions
E. reverse auctions
ANSWER: B
51) In which of the following auctions does the auctioneer first announce a high price for a product and then slowly
decreases the price until a bidder accepts?
A. a Dutch auction with one buyer and many sellers
B. an English auction with one seller and many buyers
C. an ascending bid auction
D. a sealed-bid auction
E. a Dutch auction with one seller and many buyers
ANSWER: E
52) In a(n) ________, the buyer announces something he or she wants to buy, and potential sellers compete to offer the
lowest price.
A. Dutch auction with one buyer and many sellers
B. English auction with one buyer and many sellers
C. English auction with one seller and many buyers
D. sealed-bid auction
E. ascending auction
ANSWER: A
53) ________ auctions let would-be suppliers submit only one bid; they cannot know the other bids.
A. Descending bid
B. Sealed-bid
C. English
D. Dutch
E. Reverse
ANSWER: B
54) In which of the following forms of countertrade do buyers and sellers directly exchange goods, with no money and no
third party is involved?
A. buyback arrangements
B. offsets
C. barter
D. sealed bids
E. compensation deals

Downloaded by Lan Anh Tr?nh (lananh30303@gmail.com)


lOMoARcPSD|16636219

ANSWER: C
55) A Japanese firm is ready to sell its recent technological innovation to the US government. But it has asked for 80
percent in cash and the rest in mica. The Japanese firm is looking to enter into a(n) ________ with the US government.
A. functional discount
B. compensation deal
C. buyback arrangement
D. offset agreement
E. barter deal
ANSWER: B
56) Armac Ltd. is a sluice-box manufacturer based in China. A sluice-box is used for gold prospecting. Armac is interested
in selling a few of its machines to an American mining company, but it wants 95 percent of the machines' price in gold
and the rest in ores recovered by using the machines. This is an example of a ________.
A. buyback arrangement
B. functional discount
C. barter deal
D. compensation deal
E. sealed bid
ANSWER: A
57) ROC Engineering, a Chinese shipbuilding company, agrees to build a fleet of submarines for the Sri Lankan navy, for
which it will be paid in the local Sri Lankan currency. As per the agreement, ROC must also spend a substantial amount of
the money it generates through this deal within the country. In accordance with the contract, ROC buys Sri Lankan tea at
a reduced rate. This is an example of which of the following forms of countertrade?
A. descending bid
B. offset
C. barter
D. compensation deal
E. buyback arrangement
ANSWER: B
58) ________ are offered by a manufacturer to trade-channel members if they will perform certain functions, such as
selling, storing, and recordkeeping.
A. Consumer promotions
B. Quantity discounts
C. Functional discounts
D. Seasonal discounts
E. Trade-in allowances
ANSWER: C
59) When hotels, motels, and airlines offer discounts in slow selling periods, they are said to be offering ________.
A. trade discounts
B. quantity discounts
C. functional discounts
D. seasonal discounts
E. trade-in allowances
ANSWER: D
60) A(n) ________ is an extra payment designed to gain reseller participation in special programs.
A. seasonal discount
B. allowance
C. discount
D. quantity discount
E. functional discount
ANSWER: B
61) ________ reward dealers for participating in advertising and sales support programs.
A. Functional discounts
B. Trade discounts
C. Promotional allowances
D. Rebates
E. Quantity discounts
ANSWER: C
62) When supermarkets and department stores drop the price on well-known brands to stimulate store traffic, they are
said to be following ________ pricing.

Downloaded by Lan Anh Tr?nh (lananh30303@gmail.com)


lOMoARcPSD|16636219

A. value
B. loss-leader
C. special event
D. high-low
E. everyday low
ANSWER: B
63) In ________, the seller charges a separate price to each customer depending on the intensity of his or her demand.
A. second-degree price discrimination
B. third-degree price discrimination
C. psychological discounting
D. special-customer pricing
E. first-degree price discrimination
ANSWER: E
64) In second-degree price discrimination, the seller charges ________.
A. less to buyers of larger volumes
B. different prices depending on the season, day, or hour
C. a separate price to each customer depending on the intensity of his or her demand
D. different prices for different versions of the same product
E. different prices for the same product depending on the channel through which it is sold
ANSWER: A
65) In ________, the seller charges different amounts to different classes of buyers.
A. perceived value pricing
B. third-degree price discrimination
C. first-degree price discrimination
D. second-degree price discrimination
E. psychological discounting
ANSWER: B
66) When museums charge a lower admission fee to students and senior citizens, this form of price discrimination is
known as ________ pricing.
A. location
B. channel
C. customer-segment
D. special-customer
E. loss-leader
ANSWER: C
67) Madame Tussaud's wax museum is a popular tourist attraction in London. The museum charges higher entry rates for
tourists compared to locals. This form of price discrimination is known as ________ pricing.
A. customer-segment
B. image
C. location
D. special customer
E. special event
ANSWER: A
68) When Coca-Cola carries a different price depending on whether the consumer purchases it in a fine restaurant, a fast-
food restaurant, or a vending machine, then this form of price discrimination is known as ________ pricing.
A. product-form
B. loss-leader
C. special event
D. channel
E. location
ANSWER: D
69) The prices of tickets to the opera vary depending on where the person would like to be seated — in the gallery or in
the stalls. This is an example of ________ pricing.
A. channel
B. time
C. image
D. product-form
E. location
ANSWER: E

Downloaded by Lan Anh Tr?nh (lananh30303@gmail.com)


lOMoARcPSD|16636219

70) When hotels drop their rates on the weekends, this form of price discrimination is known as ________ pricing.
A. channel
B. image
C. product-form
D. time
E. location
ANSWER: D
71) The airline and hospitality industries use ________, by which they offer discounted but limited early purchases,
higher-priced late purchases, and the lowest rates on unsold inventory just before it expires.
A. special-customer pricing
B. yield pricing
C. cash rebates
D. location pricing
E. customer-segment pricing
ANSWER: B
72) ________ refers to selling below cost with the intention of destroying competition.
A. Bid rigging
B. Loss-leader pricing
C. Predatory pricing
D. Price discrimination
E. Price penetration
ANSWER: C
73) For price discrimination to work ________.
A. the market must be segmentable and the segments must show similar intensities of demand
B. members in the lower-price segment must be able to resell the product to the higher- price segment
C. competitors must be able to undersell the firm in the higher-price segment
D. the practice must not breed customer resentment and ill will
E. the extra revenue derived from price discrimination must not exceed the cost of segmenting and policing the market
ANSWER: D
74) When a movie theater charges a lower ticket fee for children and senior citizens, it is engaging in ________ pricing.
A. product-form
B. image
C. customer-segment
D. location
E. time
ANSWER: C
75) A low price buys market share but not market loyalty. The same customers will shift to any lower-priced product that
may come along. This is called the ________ trap.
A. low-price
B. relative-market-share
C. shallow-pockets
D. target-market-share
E. fragile-market-share
ANSWER: E
76) When higher-priced competitors match lower prices of their competitors but have longer staying power because of
deeper cash reserves, it leads to a(n) ________ trap.
A. low-quality
B. fragile-market-share
C. price war
D. escalator
E. shallow-pockets
ANSWER: E
77) A company does not set a final price until the product is finished or delivered. This is known as ________.
A. delayed quotation pricing
B. an escalator clause
C. special-event pricing
D. time pricing
E. the shallow-pockets trap
ANSWER: A

Downloaded by Lan Anh Tr?nh (lananh30303@gmail.com)


lOMoARcPSD|16636219

78) When a company requires customers to pay today's price and all or part of any inflation increase that takes place
before delivery, it is known as ________.
A. special-customer pricing
B. an escalator clause
C. delayed quotation pricing
D. unbundling
E. time pricing
ANSWER: B
79) When a company maintains its price but removes or prices separately one or more elements that were part of the
former offer, such as free delivery or installation, it is known as ________.
A. escalating
B. differentiation
C. unbundling
D. reverse discounting
E. delayed quotation pricing
ANSWER: C
80) In markets that are characterized by products that are highly homogeneous, how should a firm react to a competitor's
reduction in price?
A. shrink the amount of the product available
B. substitute expensive materials or ingredients
C. reduce product features
D. reduce product services
E. augment the product
ANSWER: E

Downloaded by Lan Anh Tr?nh (lananh30303@gmail.com)


lOMoARcPSD|16636219

Chapter 17 Designing and Managing Integrated Marketing Channels


1) ________ are sets of interdependent organizations participating in the process of making a product or service available
for use or consumption.
A. Marketing channels
B. Interstitials
C. Communication channels
D. Sales territories
E. Marketing terrains
ANSWER: A
2) A merchant is a(n) ________ in the marketing channel.
A. wholesaler
B. broker
C. sales agent
D. warehouse
E. advertising agency
ANSWER: A
3) A manufacturer uses the company's sales force and trade promotions to carry, promote, and sell products to end users.
Which of the following strategies is this manufacturer using?
A. personalization strategy
B. tailoring strategy
C. push strategy
D. pull strategy
E. consumer promotion strategy
ANSWER: C
4) Total Beverages, a maker of fruit juices and health drinks, recently launched a new brand of packaged drinking water
called AquaPure. In order to induce distributors to carry the product, Total offers all its intermediaries a free refrigerator
to store bottles of AquaPure. This is an example of a ________.
A. consumer promotion
B. push strategy
C. backward flow
D. reverse flow
E. pull strategy
ANSWER: B
5) Spike Inc. is a sportswear manufacturer that recently launched its new line of customizable running shoes. The shoes
come with a digital component that allows them to adapt to the runner's biomechanics. To promote this new product,
Spike launches an advertising campaign and entices a famous athlete to endorse the product. This is an example of a
________.
A. trade promotion
B. reverse flow
C. push strategy
D. pull strategy
E. backward flow
ANSWER: D
6) When is a pull strategy appropriate?
A. when there is low brand loyalty
B. when consumers are able to perceive differences between brands
C. when brand choice is made in the store
D. when it is a low involvement purchase
E. when the product is an impulse item
ANSWER: B
7) Using the push strategy is most appropriate when ________.
A. consumers are able to perceive differences between brands
B. the product being sold is an impulse item
C. there is high brand loyalty for the product
D. the product is a high involvement purchase
E. consumers choose the brand before they go to the store
ANSWER: B
8) A firm uses its sales force to sell to large accounts and outbound telemarketing to sell to medium-sized accounts. The
firm is using ________ marketing.

Downloaded by Lan Anh Tr?nh (lananh30303@gmail.com)


lOMoARcPSD|16636219

A. hybrid
B. pull
C. personalized
D. vertical
E. internal
ANSWER: A
9) A computer manufacturing company allows customers to place orders online, which they can later pick up from a
convenient retail location. Which of the following terms best represents this practice?
A. channel integration
B. mass customization
C. online personalization
D. push strategy
E. internal marketing
ANSWER: A
10) Which of the following is the most accurate description of a value network?
A. a system of partnerships and alliances that a firm creates to source, augment, and deliver its offerings
B. a system of organizations and resources involved in moving a product from supplier to customer
C. an arrangement whereby an organization transforms inputs into finished goods
D. a network that allows an organization take the finished products to the end-users
E. a communication network that allows an organization to transfer information to end- customers
ANSWER: A
11) Companies should first think about the target market and then design the supply chain backward from that point.
This strategy is called ________ planning.
A. demand chain
B. resource
C. external channel
D. materials
E. strategic business
ANSWER: A
12) Which of the following channel functions constitute only a backward flow?
A. movement of physical goods
B. placing orders with manufacturers
C. persuasive communication
D. storage of physical goods
E. overseeing actual transfer of ownership
ANSWER: B
13) Identify the channel function that constitutes both backward and forward flow.
A. obtaining funds for financing
B. storage of physical goods
C. persuasive communication
D. movement of physical goods
E. overseeing actual transfer of ownership
ANSWER: A
14) Producers often shift some functions to intermediaries. Which of the following is the most significant benefit of doing
this?
A. It increases customer loyalty.
B. It provides the producer with greater control over operations.
C. It reduces the amount of direct customer interaction.
D. It lowers the producer's costs and prices.
E. It ensures greater information security.
ANSWER: D
15) Which of the following is an example of a zero-level channel?
A. A company takes online orders from customers and ships the products to them.
B. An organization uses a combination of direct salespeople and sales agencies to increase sales.
C. A company sells its products through wholesalers and retailers.
D. A company sells its products through chains of supermarkets and other large sellers.
E. A large company forms alliances with smaller companies to increase sales coverage.
ANSWER: A
16) Which of the following entities is present in a zero-level marketing channel?

Downloaded by Lan Anh Tr?nh (lananh30303@gmail.com)


lOMoARcPSD|16636219

A. consumers
B. retailers
C. brokers
D. jobbers
E. wholesalers
ANSWER: A
17) A direct marketing channel is a ________ channel.
A. one-level
B. two-level
C. zero-level
D. three-level
E. reverse-flow
ANSWER: C
18) A jobber in a three-level marketing channel is a(n) ________.
A. small-scale wholesaler
B. external broker
C. advertising agent
D. independent evaluator
E. communication channel
ANSWER: A
19) Door-to-door sales, home parties, mail order, telemarketing, TV selling, Internet selling, and manufacturer-owned
stores are examples of ________.
A. zero-level channels
B. jobbers
C. wholesalers
D. manufacturer's representatives
E. consumers
ANSWER: A
20) Which of the following activities is a reverse-flow channel of marketing?
A. raw materials movement
B. product recycling
C. materials ordering
D. finished goods storage
E. customer order placement
ANSWER: B
21) Toyota has an advantage over Lexus due to the fact that there are more Toyota dealers, which helps customers save
on transportation and search costs in buying and repairing an automobile. Which of the following service outputs relates
to this competitive advantage?
A. spatial convenience
B. service backup
C. lot size
D. waiting time
E. delivery time
ANSWER: A
22) As a service output produced by marketing channels, product variety refers to the ________.
A. units the channel permits a customer to purchase at once
B. assortment provided by the marketing channel
C. add-on services provided by the channel
D. ability of a product to provide incremental value
E. degree to which the channel makes it easy for customers to purchase a product
ANSWER: B
23) Which of the following terms refers to add-on services such as credit, delivery, installation, and repairs, and is
provided by a marketing channel?
A. service backup
B. product accessories
C. external products
D. product variety
E. service extensions
ANSWER: A

Downloaded by Lan Anh Tr?nh (lananh30303@gmail.com)


lOMoARcPSD|16636219

24) Atburex is a furniture manufacturing company in the United States. The company provides a 60-day credit period and
also offers on-site delivery and installation. These special benefits refer to which of the following service outputs?
A. service backup
B. large product variety
C. spatial convenience
D. large lot size
E. short waiting time
ANSWER: A
25) Which of the following types of distribution involves severely limiting the number of channel intermediaries?
A. exclusive
B. selective
C. intensive
D. aggressive
E. retail
ANSWER: A
26) Which of the following allows a firm to maintain control over service level and obtain more dedicated and
knowledgeable selling?
A. selective distribution
B. intensive distribution
C. push strategy
D. exclusive distribution
E. pull strategy
ANSWER: D
27) Which of the following channel alternatives is most suited to handle complex products and transactions?
A. sales forces
B. the Internet
C. dealers
D. telemarketers
E. direct mails
ANSWER: A
28) Which of the following is a major disadvantage of using the Internet as a marketing channel?
A. It is less effective for complex products.
B. It lacks convenience and practicality.
C. It cannot be used to reach a wide audience.
D. It is considered expensive.
E. It causes the company to lose direct contact with customers.
ANSWER: A
29) Armon Apparels designs, manufactures, and distributes athletic apparel and accessories for men and women. The
company has only nine distributors across the United States. These distributors control a nationwide network of 600
retailers. The company does not sell its products through other channels. This is an example of ________ distribution.
A. selective
B. intensive
C. exclusive
D. internal
E. passive
ANSWER: A
30) Exclusive dealing arrangements are mainly used by companies looking for an edge in markets increasingly driven by
________.
A. price
B. efficiency
C. product variety
D. add-on services
E. spatial convenience
ANSWER: A
31) An intensive distribution strategy serves well for ________.
A. premium cars
B. commercial trucks
C. private label products
D. industrial equipment

Downloaded by Lan Anh Tr?nh (lananh30303@gmail.com)


lOMoARcPSD|16636219

E. newspapers
ANSWER: E
32) Which of the following products is most likely to be sold using an exclusive distribution strategy?
A. designer luggage
B. cigarettes
C. alcoholic beverages
D. car fuel
E. medicine
ANSWER: A
33) Which of the following covers payment terms and producer guarantees?
A. conditions of sale
B. pricing policies
C. exclusive dealings
D. mutual services
E. territorial rights
ANSWER: A
34) ________ call(s) for the producer to establish a schedule of discounts and allowances that intermediaries see as
equitable and sufficient.
A. Exclusive dealings
B. Mutual services
C. Territorial rights
D. Price policy
E. Tying agreements
ANSWER: D
35) Electrobar, a European manufacturer of industrial kitchenware, sells to industrial canteens, restaurants, hotels, and so
forth. The company provides a one-year warranty on all products and also allows customers to pay in installments — they
pay 50 percent on delivery and the rest as equal installments. This refers to which element in the "trade-relations mix"?
A. price policy
B. conditions of sale
C. distributors' territorial rights
D. exclusive dealings
E. mutual services and responsibilities
ANSWER: B
36) Which of the following channels is associated with the lowest cost per transaction?
A. Internet
B. telemarketing
C. retail stores
D. distributor
E. sales force
ANSWER: A
37) Which marketing channel is associated with the highest value added per sale?
A. retail stores
B. sales force
C. distributors
D. Internet
E. telemarketing
ANSWER: B
38) Which of the following problems is most likely when a company chooses to use a sales agency instead of company
salespeople?
A. The company will find it difficult to control the sales process.
B. They do not take title to goods or negotiate purchases or sales.
C. The value-added per sale is the lowest for sales agencies.
D. Agencies will pay less attention to customers who buy the most or in large volumes.
E. Sales agencies are often difficult to access due to strict contractual obligations.
ANSWER: A
39) In an effort to boost sales, Broomer offers its retailers a higher margin for promoting and selling products from the
"Inducer" line to customers. This is an example of ________ power.
A. coercive
B. reward

Downloaded by Lan Anh Tr?nh (lananh30303@gmail.com)


lOMoARcPSD|16636219

C. passive
D. expert
E. referent
ANSWER: B
40) Because of the acceptance that the other Broomer products have in the market, retailers are willing to stock items
from the new "Inducer" line of clothing. This is an example of ________ power.
A. referent
B. passive
C. legitimate
D. coercive
E. reward
ANSWER: A
41) Broomer threatens to withdraw all its other products from the retailers' stores if they are unwilling to push products
from the "Inducer" line. This is an example of ________ power.
A. reward
B. coercive
C. legitimate
D. expert
E. referent
ANSWER: B
42) ________ power can be effective, but its exercise produces resentment and can lead the intermediaries to organize
countervailing power.
A. Coercive
B. Reward
C. Legitimate
D. Expert
E. Referent
ANSWER: A
43) A manufacturer offers its intermediaries an extra benefit for performing a promotional activity. This is an example of
the use of ________ power.
A. reward
B. coercive
C. functional
D. expert
E. referent
ANSWER: A
44) A manufacturer is using legitimate power when it ________.
A. requests a behavior that is warranted under the selling contract
B. threatens to withdraw a resource or terminate a relationship
C. offers intermediaries an extra benefit for performing specific acts or functions
D. makes the intermediaries sell more of a particular product by offering rewards
E. sells more products by making use of its reputation in the market
ANSWER: A
45) Hewlett-Packard is a highly respected brand. Many retailers want to be associated with the brand because of this
reputation. What kind of power does Hewlett-Packard obtain due to this reputation?
A. referent
B. functional
C. legitimate
D. coercive
E. reward
ANSWER: A
46) Which of the following types of power is objectively observable?
A. coercive power
B. legitimate power
C. group power
D. expert power
E. referent power
ANSWER: A
47) A new firm typically starts as a local operation selling in a fairly circumscribed market by ________.

Downloaded by Lan Anh Tr?nh (lananh30303@gmail.com)


lOMoARcPSD|16636219

A. finding and developing new intermediaries


B. using a few existing intermediaries
C. forming partnerships with the market leader
D. creating a special channel
E. forming partnerships with other firms
ANSWER: B
48) Who will most likely be willing to pay for high-value-added channels?
A. early buyers of a product
B. internal customers of a company
C. small and matured buyers of an industry
D. consumers of low involvement products
E. repeat customers of a product
ANSWER: A
49) A producer must modify its channel design and arrangements if ________.
A. consumer buying patterns change
B. the competition in the market stabilizes
C. the product is in the growth stage of its life cycle
D. the market size remains unchanged for a particular period
E. the firm's profits stabilize
ANSWER: A
50) What is the major difference between a conventional marketing channel and a vertical marketing system (VMS)?
A. Elements in a conventional marketing channel act as separate businesses whereas the elements in a VMS act as a
unified system.
B. A VMS has many intermediaries whereas a conventional marketing channel has a limited number of intermediaries.
C. A VMS is characterized by an independent producer whereas a conventional marketing channel is characterized by
multiple producers.
D. A conventional marketing channel has elements such as retailers and wholesalers whereas these elements are not
present in a VMS.
E. Producers have complete control over the other members in a conventional marketing channel whereas this control is
minimal in a VMS.
ANSWER: A
51) A(n) ________ includes the producer, wholesaler(s), and retailer(s) acting as a unified system.
A. parallel marketing channel
B. vertical marketing system
C. extensive marketing channel
D. internal marketing system
E. conventional marketing channel
ANSWER: B
52) A(n) ________ vertical marketing system combines successive stages of production and distribution under single
ownership.
A. corporate
B. administered
C. contractual
D. regulatory
E. controlled
ANSWER: A
53) The most advanced supply-distributor arrangements for ________ vertical marketing systems rely on distribution
programming.
A. corporate
B. administered
C. contractual
D. regulatory
E. controlled
ANSWER: B
54) Rotter Garder Inc. is a large-scale paint manufacturer and is known for its wide range of decorative paint products
and industrial coatings. In addition to making paints, the company also owns and operates the retail stores that sell its
products. This is an example of a(n) ________ vertical marketing system.
A. administered
B. contractual

Downloaded by Lan Anh Tr?nh (lananh30303@gmail.com)


lOMoARcPSD|16636219

C. referent
D. corporate
E. regulated
ANSWER: D
55) An administered VMS coordinates successive stages of production and distribution through ________.
A. an automated central control unit
B. single ownership
C. the combined efforts of all its members
D. the establishment of contractual obligations
E. the size and power of one of the members
ANSWER: E
56) A(n) ________ VMS consists of independent firms at different levels of production and distribution, integrating their
programs on a contractual basis to obtain more economies or sales impact than they could achieve alone.
A. administered
B. contractual
C. corporate
D. regulated
E. referent
ANSWER: B
57) A group of small sellers takes the initiative and organizes a new business entity to carry on wholesaling and possibly
some production. This entity is called a(n) ________.
A. retailer cooperative
B. franchise organization
C. area-based cartel
D. sponsored voluntary chain
E. alternate selling channel
ANSWER: A
58) In a retailer cooperative, ________.
A. profits are equally divided among members
B. members plan their advertising jointly
C. nonmembers cannot buy through the co-op
D. members rely on distribution programming
E. members standardize their selling practices
ANSWER: B
59) A group of small grocery shops forms a new business entity to buy products directly from manufacturers. The group
buys products in bulk that are then distributed among members. This helps the shops obtain better profit margins. Which
of the following types of vertical marketing systems can be observed here?
A. contractual
B. corporate
C. administered
D. controlled
E. regulatory
ANSWER: A
60) A franchise organization is an example of a(n) ________ vertical marketing system.
A. corporate
B. administered
C. contractual
D. regulatory
E. controlled
ANSWER: C
61) In a ________ marketing system, two or more unrelated companies put together resources or programs to exploit an
emerging marketing opportunity.
A. reverse flow
B. vertical
C. horizontal
D. lateral
E. forward flow
ANSWER: C
62) Which of the following is a major advantage of adding more channels for selling?

Downloaded by Lan Anh Tr?nh (lananh30303@gmail.com)


lOMoARcPSD|16636219

A. It helps the company increase its market coverage.


B. It helps the company reduce its fixed costs.
C. It reduces the likelihood of channel conflict.
D. It is the best strategy for selling low-involvement consumer products.
E. It results in economies of scale.
ANSWER: A
63) Which of the following is the most complete and accurate description of a pure-click company?
A. These are companies that launch a Web site without any previous existence as a firm.
B. These firms' business models are based on advertising revenue.
C. These are established companies that have an online site for selling products.
D. These companies get paid every time a user visits their Web site.
E. These companies sell online space to other Web sites and are paid for the service.
ANSWER: A
64) A large retail chain in the United States decides to expand its operations by adding an online site for e-commerce.
This is called a(n) ________ company.
A. B2B
B. brick-and-mortar
C. m-commerce
D. pure-click
E. brick-and-click
ANSWER: E
65) An Internet service provider (ISP) is a(n) ________ company.
A. pure-click
B. brick and click
C. brick-and-mortar
D. m-commerce
E. one-level
ANSWER: A
66) JSE Securities Exchange is the largest stock exchange in Africa. The JSE provides a market where securities can be
traded freely under a regulated procedure. The company acts as an intermediary between the traders. JSE is an example
of a(n) ________.
A. internal broker
B. infomediary
C. customer community
D. market maker
E. third party arbitrator
ANSWER: D
67) Exxon Consulting works as an agent on behalf of business consumers to collect information on various industrial
products. When faced with buying decisions, businesses can approach Exxon to obtain detailed information on the various
options available to them. The company earns revenue by selling such information to various customers. Exxon
Consulting can be called a(n) ________.
A. infomediary
B. market maker
C. customer community
D. third party arbitrator
E. informant
ANSWER: A
68) Consumer surveys suggest that one of the most significant inhibitors of online shopping is the absence of ________.
A. pleasurable experiences
B. competitive prices
C. adequate technical information
D. after-sales service
E. facilities to compare offerings
ANSWER: A
69) Which of the following is an example of a brick-and-click company?
A. IFB Industries, a company that sells products using various channels such as Internet, retailers, direct outlets, and
franchisees.
B. Opera, a Web browser and Internet suite developed by Opera Software, performs common tasks such as displaying
Web sites and sending and receiving e-mail messages.

Downloaded by Lan Anh Tr?nh (lananh30303@gmail.com)


lOMoARcPSD|16636219

C. Atrutron, a company that offers its customers access to the Internet using data transmission technology.
D. eBay Inc., an American company, facilitates online auctions and shopping to people and businesses across the globe to
buy and sell a broad variety of goods and services.
E. Yahoo! Inc., an American corporation, provides services via the Internet such as directories, e-mail, news, advertising,
online mapping, and so on.
ANSWER: A
70) M-commerce refers to ________.
A. conducting business using mobile channels
B. the use of mass media communications to attract customers
C. providing mobile and on-site services to customers
D. the use of a specific medium to communicate with prospects
E. using the Internet as a medium for doing business
ANSWER: A
71) When Japanese teenagers carry DOCOMO phones from NTT and use them to order goods, they are engaged in
________.
A. B2B e-commerce
B. brick-and-click commerce
C. infomediation
D. dilution
E. m-commerce
ANSWER: E
72) ________ channel conflict occurs between channel members at the same level.
A. Horizontal
B. Vertical
C. Multichannel
D. Administrative
E. Contractual
ANSWER: A
73) Alcart Solutions is a large distributor of Aldor phones in Canada. The company distributes products to various retailers
in the New Brunswick province. Recently Aldor received several complaints from its retailers that their orders are not
delivered on time. They also complain that Alcart offers preferential treatment to some of the other retailers in the region.
This is an example of ________ conflict.
A. multichannel
B. horizontal
C. vertical
D. intermediate
E. parallel
ANSWER: C
74) When Goodyear began selling its popular tire brands through Sears, Walmart, and Discount Tire, independent dealers
that sold the same tires at higher prices were angry. This is an example of a(n) ________ conflict.
A. horizontal
B. vertical
C. intermediate
D. multichannel
E. parallel
ANSWER: D
75) A manufacturer wants to achieve rapid market penetration through a low-price policy. However, the manufacturer's
dealers prefer to work with high margins and pursue short-run profitability. The major reason for this conflict is ________.
A. goal incompatibility
B. unclear roles
C. ambiguous rights
D. differences in perception
E. dependence on the manufacturer
ANSWER: A
76) When Estee Lauder set up a Web site to sell its Clinique and Bobbi Brown brands, Dayton Hudson reduced space for
Estee Lauder products in its department stores in response to the ________ conflict.
A. horizontal channel
B. multichannel
C. vertical channel

Downloaded by Lan Anh Tr?nh (lananh30303@gmail.com)


lOMoARcPSD|16636219

D. grid channel
E. end-customer
ANSWER: C
77) General Motors' executives work for a short time in some dealerships, and some dealership owners work in GM's
dealer policy department. This strategy helps the company avoid conflicts with its dealers. This is an example of the
________ strategy.
A. employee exchange
B. dual compensation
C. joint membership
D. co-optation
E. diplomacy
ANSWER: A
78) RX Corp. is a large manufacturer of electronic goods and sells its products through distributors and retailers. In order
to keep pace with the growing use of the Internet, the company decides to start selling online. The company faces stiff
opposition from its retailers as they believe that this will significantly reduce their profits. The company attempts to
eliminate this resistance by offering its retailers commissions for processing and delivering orders received via the Web.
This is an example of which of the following conflict resolution strategies?
A. dual compensation
B. joint membership
C. arbitration
D. co-optation
E. strategic pricing
ANSWER: A
79) Co-optation is an effort by one organization to win the support of the leaders of another by ________.
A. including them in advisory councils
B. engaging in mediation and arbitration
C. encouraging joint memberships in trade associations
D. encouraging employee exchanges
E. offering strategic justifications
ANSWER: A
80) Winstar is a large-scale manufacturer with more than a hundred partners across the globe. When making decisions
concerning distribution and channel optimization, the company invites members from its channel partners to be part of its
advisory committee. This helps the company maintain harmony with its partners. Which of the following conflict resolution
techniques is Winstar using?
A. diplomatic counselling
B. mediation
C. arbitration
D. co-optation
E. joint membership
ANSWER: D
81) Which of the following channel conflict resolution techniques is used only if everything else proves ineffective?
A. legal recourse
B. mediation
C. arbitration
D. co-optation
E. superordinate goals
ANSWER: A
82) What is the trade-off in return for the benefits associated with an integrated multichannel marketing system?
A. increased market coverage
B. customers who buy in one channel are more profitable than customers who buy across different channels
C. lower channel costs
D. two or more channels may end up competing for the same customer
E. customized selling
ANSWER: D
83) If a producer wants to achieve rapid market penetration through a low-price policy, while a dealer wants to work with
high margins to pursue short-run profitability, the source of the channel conflict is ________.
A. unclear roles and rights
B. goal incompatibility
C. differences in perception

Downloaded by Lan Anh Tr?nh (lananh30303@gmail.com)


lOMoARcPSD|16636219

D. intermediaries' dependence on the manufacturer


E. strategic justification
ANSWER: B
84) Jaycee was using ________ when his organization tried to win the support of the leaders of one of his distributors by
including them in advisory councils, boards of directors, and the like.
A. dual compensation
B. employee exchange
C. co-optation
D. joint membership
E. strategic justification
ANSWER: C

Downloaded by Lan Anh Tr?nh (lananh30303@gmail.com)


lOMoARcPSD|16636219

Chapter 18 Managing Retailing, Wholesaling, and Logistics


1) ________ includes all the activities in selling goods or services directly to final consumers for personal, non-business
use.
A. Wholesaling
B. Retailing
C. Procurement
D. Promoting
E. Warehousing
ANSWER: B
2) Which of the following is true for retailing?
A. Manufacturers are not considered to be retailers as they are engaged in producing the product.
B. Vending machines are considered to be retailing only if they are located within stores.
C. Retailing deals only with goods; it does not include services.
D. Selling from a consumer's home is direct selling, but not retailing.
E. Wholesalers are only considered to be retailers if they are selling to final consumers.
ANSWER: E
3) Which of the following is an example of retailing?
A. Dylan's sends catalogs to retail, industrial, and institutional customers.
B. SEZ U Inc. sells a limited line of fast-moving goods to small retailers for cash.
C. BEL Inc. sells FMCG goods to merchant wholesalers and distributors.
D. Praxis International sells products to consumers directly through the Internet.
E. Hub Styles procures its raw materials directly from farmers in the region.
ANSWER: D
4) Discount stores that try to keep prices as low as possible are more likely to function using ________ operations.
A. limited service
B. self-selection
C. full-service
D. self-service
E. limited-selection
ANSWER: D
5) Reynold's is a grocery chain that has always catered to mid-market customers. However, the owner, Mal, has noticed
that an influx of new residents are buying mostly the lower-cost and discounted products. To attract customers, Mal
decides to make a gradual switch to the discount store format, but to do this, he will have to cut costs wherever possible.
Which of the following types of services should Mal avoid in order to lower costs?
A. limited service
B. self-selection
C. full-service
D. self-service
E. limited-selection
ANSWER: C
6) A large staff, along with a higher proportion of specialty goods and slower-moving items and many services, are usually
features of ________ retailing.
A. self-service
B. self-selection
C. limited service
D. full-service
E. limited-selection
ANSWER: D
7) In ________ retailing, salespeople are ready to assist in every phase of the "locate- compare-select" process.
A. self-service
B. self-selection
C. full-service
D. limited service
E. limited-selection
ANSWER: C
8) In the ________ type of retailing, customers usually find their own goods, although they can ask salespeople for
assistance.
A. self-service
B. self-selection

Downloaded by Lan Anh Tr?nh (lananh30303@gmail.com)


lOMoARcPSD|16636219

C. full-service
D. limited service
E. limited-selection
ANSWER: B
9) Which of the following is true for self-service retailing?
A. They carry more shopping goods and services such as credit and merchandise-return privileges.
B. This service model is favored by discount stores and customers who want to save money.
C. It results in high staffing costs compared to other forms of retailing.
D. Salespeople are ready to assist in every phase of the "locate-compare-select" process.
E. Customers need more information and assistance than in other forms of retailing.
ANSWER: B
10) Which of the following types of retailing generally entails the highest costs?
A. full-service
B. self-service
C. limited-selection
D. limited service
E. self-selection
ANSWER: A
11) Which of the following is an example of a limited-service retailer?
A. Customers at TAB pick out the products they want and pay at the checkout counter.
B. Salespeople at Eli are always on hand to offer shoppers advice and assistance in choosing products.
C. Mina's allows customers to checkout their own purchases in order to reduce the staff count.
D. As CLO's offers a number of merchandise-return privileges, customers need information and assistance while shopping.
E. Alison's stocks a number of specialty products and the store's salespeople generally help shoppers with all their needs.
ANSWER: D
12) Bill and Josh are considering opening a retail store. They have identified their target market and location and are
finalizing the details of the merchandise they will carry. Since the neighborhood is rundown and the customers in the area
are very price-conscious, Bill and Josh want to offer goods from well-known brands, but at lower rates than the full retail
prices of the products. They choose to stock excess production from manufacturers or goods that have remained unsold
at other retailers. This is a description of a(n) ________ retailer.
A. off-price
B. specialty
C. discount
D. department
E. catalog
ANSWER: A
13) H&A is a retail chain that specializes in selling goods at very low prices. To achieve this, it stocks a very narrow
assortment of basic necessities and offers customers a "no-frills" shopping experience. H&A is an example of a(n)
________ store.
A. off-price
B. specialty
C. hard-discount
D. superstore
E. convenience
ANSWER: C
14) An example of a restaurant with a narrow and deep assortment is a ________.
A. small lunch counter
B. cafeteria
C. large restaurant
D. casual dining restaurant chain
E. delicatessen
ANSWER: E
15) E&OE produces and markets its own brand of skincare products using herbal remedies and natural ingredients
through standalone stores as well as an online portal. E&OE is a(n) ________ retailer.
A. off-price
B. specialty
C. discount
D. department
E. extreme value

Downloaded by Lan Anh Tr?nh (lananh30303@gmail.com)


lOMoARcPSD|16636219

ANSWER: B
16) Aldi, Lidl, Dollar General, and Family Dollar are examples of ________, as they carry a more restricted merchandise
mix than discount stores at even lower prices.
A. off-price retailers
B. extreme value stores
C. superstores
D. convenience stores
E. specialty stores
ANSWER: B
17) A factory outlet is an example of a(n) ________ retailer.
A. off-price
B. specialty
C. discount
D. department
E. catalog
ANSWER: A
18) Avon, Tupperware, and Southwestern Company of Nashville are among companies in the multibillion-dollar ________
industry, which involves selling door-to-door or at home sales parties.
A. direct marketing
B. catalog showroom
C. direct selling
D. automatic vending
E. buying services
ANSWER: C
19) Companies that sell products door-to-door or at home sales parties are engaging in ________.
A. franchising
B. network marketing
C. direct-response marketing
D. corporate selling
E. direct marketing
ANSWER: B
20) Which of the following is an example of direct selling?
A. E&OE sells its herbal skincare products exclusively through its standalone stores.
B. TCJ is a telemarketing firm that sells products from a number of different suppliers.
C. Jayne's sells most of its products to customers through home sales parties.
D. J3 is an online shopping portal where customers can buy directly from manufacturers.
E. Reynold's tries to minimize its staff costs by installing vending machines in its stores.
ANSWER: C
21) Which of the following is an example of direct marketing?
A. E&OE sells its herbal skincare products exclusively through its standalone stores.
B. TCJ is a telemarketing firm that sells products from a number of different suppliers.
C. Jayne's sells most of its products to customers through home sales parties.
D. J3 is a storeless retailer that organizes the retail activity of the employees of four firms.
E. Reynold's tries to minimize its staff costs by installing vending machines in its stores.
ANSWER: B
22) The ________ marketing sales system works by recruiting independent businesspeople who act as distributors.
A. catalog
B. multilevel
C. direct-response
D. corporate
E. direct
ANSWER: B
23) In ________, a salesperson goes to the home of a host who has invited friends, demonstrates the products, and
takes orders.
A. catalog marketing
B. franchising
C. direct-response selling
D. network marketing
E. direct marketing

Downloaded by Lan Anh Tr?nh (lananh30303@gmail.com)


lOMoARcPSD|16636219

ANSWER: D
24) Telemarketing is a type of ________.
A. direct selling
B. network marketing
C. multilevel selling
D. close-range marketing
E. direct marketing
ANSWER: E
25) Electronic shopping is a type of ________.
A. direct selling
B. network marketing
C. multilevel selling
D. corporate selling
E. direct marketing
ANSWER: E
26) A(n) ________ is a storeless retailer serving a specific clientele — usually employees of large organizations — who are
authorized to buy from a list of retailers that have agreed to give discounts in return for inclusion on the list.
A. direct-selling vendor
B. direct marketing vendor
C. buying service
D. automatic vendor
E. corporate retailer
ANSWER: C
27) An independent retailer using a central buying organization and joint promotion efforts with other retailers is part of a
________.
A. corporate chain store
B. voluntary chain
C. retailer cooperative
D. merchandising conglomerate
E. franchise organization
ANSWER: C
28) A ________ is a retail firm owned by its customers. Members contribute money to open their own store, vote on its
policies, elect a group to manage it, and receive dividends.
A. retailer cooperative
B. voluntary chain
C. consumer cooperative
D. merchandising conglomerate
E. franchise organization
ANSWER: C
29) A ________ is a wholesaler-sponsored group of independent retailers engaged in bulk buying and common
merchandising.
A. retailer cooperative
B. voluntary chain
C. consumer cooperative
D. merchandising conglomerate
E. franchise organization
ANSWER: B
30) Which of the following is true for franchisors?
A. The franchisor has to pay the franchisee to be part of the franchise system.
B. The franchisor licenses the trademark from the franchisee.
C. The franchisor must change its operations to suit those of the franchisee.
D. The franchisor collects royalty payments from the franchisee.
E. The franchisor pays startup costs for the franchisee.
ANSWER: D
31) Which of the following is true for franchisees?
A. The franchisee is paid by the franchisor to be part of the franchise system.
B. The franchisee licenses a trademark to the franchisor.
C. The franchisee must change its operations to suit those of the franchisor.
D. The franchisee collects royalty payments from the franchisor.

Downloaded by Lan Anh Tr?nh (lananh30303@gmail.com)


lOMoARcPSD|16636219

E. The franchisee owns the trade or service mark.


ANSWER: C
32) Which of the following is a benefit of franchising for franchisees?
A. The franchisee finds it easier to borrow money from financial institutions.
B. The franchisee receives ownership of the franchisor's trademark.
C. The franchisee must change its operations to suit those of the franchisor.
D. The franchisee collects royalty payments from the franchisor.
E. The franchisee is paid by the franchisor for being part of the system.
ANSWER: A
33) Jake wants to open a Subway franchise in his small town. To do this, he must pay the company a ________ fee.
A. slotting
B. title
C. royalty
D. merchandising
E. residual
ANSWER: C
34) Which of the following wholesaler functions reduces inventory costs and risks to suppliers and customers?
A. selling and promoting
B. warehousing
C. transportation
D. market information
E. assortment building
ANSWER: B
35) ________ includes all the activities in selling goods or services to those who buy for resale or business use.
A. Retailing
B. Wholesaling
C. Procurement
D. Promoting
E. Warehousing
ANSWER: B
36) Which of the following is true for the retail industry?
A. Discount stores and catalog showrooms are competing for the same customers.
B. Upscale retailers see a decline in sales as middle-market retailers thrive.
C. Small, specialized retailers are crowding out larger, more diverse retailers.
D. Store retailing sees no competition from nonstore retailing.
E. Discount stores are not doing as well as middle-market retailers.
ANSWER: A
37) Electronic shelf labeling allows retailers to ________.
A. check inventory levels instantaneously
B. order electronically from suppliers
C. run continual promotional messages
D. advertise sales and special offers
E. change price levels instantaneously
ANSWER: E
38) Staples is a giant retailer that concentrates on selling office supplies. Staples is an example of a(n) ________.
A. ambush marketer
B. supercenter
C. megamarketer
D. category killer
E. guerilla marketer
ANSWER: D
39) Which of the following is an example of a category killer?
A. PETCO
B. Walmart
C. Kohl's
D. The Limited
E. Tesco
ANSWER: A

Downloaded by Lan Anh Tr?nh (lananh30303@gmail.com)


lOMoARcPSD|16636219

40) In the corporate headquarters of a supermarket chain, ________ are responsible for developing brand assortments
and listening to salespersons' presentations.
A. central buyers
B. brokers
C. specialist buyers
D. agents
E. specialized wholesalers
ANSWER: C
41) In supermarkets and other retail outlets, RFID is used to ________.
A. change prices instantaneously
B. check for spoilage or damage to goods
C. advertise special offers and discounts
D. run continual promotional messages
E. monitor inventory and track goods
ANSWER: E
42) Which of the following is true for direct product profitability?
A. It is highly correlated with the gross margin on a product.
B. It is negligible compared to the gross margin on a product.
C. It bears little relation to the gross margin on a product.
D. It is significantly lower than the gross margin on a product.
E. It is exactly the same as the gross margin on a product.
ANSWER: C
43) Mass merchandisers and discount stores typically fall into the ________ group with respect to margins and volume.
A. mixed markup, high-volume
B. low-volume, mixed markup
C. low-volume, low-markup
D. high-volume, low-markup
E. high-markup, low-volume
ANSWER: D
44) A store selling expensive artwork and luxury goods typically falls into the ________ group with respect to margins and
volume.
A. mixed markup, high-volume
B. low-volume, mixed markup
C. low-volume, low-markup
D. high-volume, low-markup
E. high-markup, low-volume
ANSWER: E
45) Most retailers will put low prices on some items in order to increase traffic to the store. These low-priced products are
known as ________.
A. loss leaders
B. price ceilings
C. price skimmers
D. price floors
E. cold calls
ANSWER: A
46) American businessman King Gillette pioneered the sales model in which razor handles were given away for free or
sold at a loss, but sales of disposable razor blades were very profitable. This is known as the ________ model.
A. two-tiered pricing
B. predatory pricing
C. cross selling
D. loss leading
E. product churning
ANSWER: D
47) Which of the following is a prepurchase service offered by retailers?
A. accepting orders over the telephone
B. shipping the product
C. delivery to the customer's doorstep
D. general information
E. interior decoration of the retail outlet

Downloaded by Lan Anh Tr?nh (lananh30303@gmail.com)


lOMoARcPSD|16636219

ANSWER: A
48) Which of the following is a postpurchase service offered by retailers?
A. accepting orders over the telephone
B. advertising and window displays
C. delivery to the customer's doorstep
D. general information
E. interior decoration of the retail outlet
ANSWER: C
49) Which of the following is an ancillary service offered by retailers?
A. accepting orders over the telephone
B. advertising and window displays
C. delivery to the customer's doorstep
D. general information
E. alterations and tailoring
ANSWER: D
50) Stores are using ________ to measure a product's handling costs from the time it reaches the warehouse until a
customer buys it in the retail store.
A. electronic data interchange (EDI)
B. direct product profitability (DPP)
C. radio-frequency identification (RFID)
D. global positioning systems (GPS)
E. compounded annual grown rate (CAGR)
ANSWER: B
51) In your neighborhood there is a small men's store that has a limited selection of clothing, but the selection that is
carried is of very high quality and price. Services include free alterations and tailoring, personalized recordkeeping, and
free dry cleaning. Inside the store are deep leather chairs and couches and thick pile carpet. Upon entering the store, a
customer feels "special and rich." In terms of differentiation, what is this retailer trying to communicate by its decorations
and service level?
A. differentiation based on services mix and atmosphere
B. differentiation based on prepurchase services
C. differentiation based on postpurchase services and atmosphere
D. differentiation based on ancillary services and atmosphere
E. differentiation based on prepurchase and postpurchase services
ANSWER: A
52) Which of the following is the strongest differentiator for brick-and-mortar stores who want to emphasize their
superiority over online retailers?
A. product quality
B. the shopping experience
C. product range
D. pricing
E. the retailer's reputation
ANSWER: B
53) A brand developed by a retailer and/or wholesaler that is available only in selected retail outlets is called a ________
brand.
A. generic
B. franchisee
C. marque
D. national
E. private-label
ANSWER: E
54) Though it is sold only in Walmart stores, Walmart's Ol'Roy dog food has surpassed Nestlé's Purina brand as the top-
selling dog food. Ol'Roy is an example of a ________.
A. generic product
B. national brand
C. franchise
D. copy-cat brand
E. private label
ANSWER: E
55) Which of the following is true for distributor brands?

Downloaded by Lan Anh Tr?nh (lananh30303@gmail.com)


lOMoARcPSD|16636219

A. They sell at higher volumes than national brands and are also known as generics.
B. They are usually sold at higher prices than national brands because production costs are higher.
C. They are always of better quality than national brands as production is strictly monitored.
D. Distributor brands can be sold at lower prices yet generate a higher profit margin because of their lower cost structure.
E. Advertising and sales promotion costs for distributor brands are much higher than those for national brands.
ANSWER: D
56) ________ are unbranded, plainly packaged, less expensive versions of common products such as spaghetti, paper
towels, and canned peaches.
A. Common carriers
B. Shills
C. Generics
D. Private labels
E. Marques
ANSWER: C
57) In addition to its store brands and nationally well-known brands of detergents, Reynold's also carries much cheaper
varieties of detergents that are not advertised and have little-known names. They are often manufactured from lower-
quality ingredients and save on packaging and advertising costs. These are known as ________.
A. common carriers
B. shills
C. generics
D. private labels
E. marques
ANSWER: C
58) Because shelf space is scarce, many supermarkets now charge a ________ for accepting a new brand, to cover the
cost of listing and stocking it.
A. retainer
B. slotting fee
C. residual fee
D. contingent fee
E. royalty
ANSWER: B
59) Which of the following is considered a wholesaler?
A. retailers
B. brokers
C. producers
D. manufacturers
E. farmers
ANSWER: B
60) ________ are independently owned businesses that take title to the merchandise they handle. They are full-service
and limited-service jobbers, distributors, and mill supply houses.
A. Brokers
B. Agents
C. Merchant wholesalers
D. Specialized wholesalers
E. Retailers' branches
ANSWER: C
61) ________ serve bulk industries such as coal, lumber, and heavy equipment, assuming title and risk from the time an
order is accepted to its delivery.
A. Producers' cooperatives
B. Cash and carry wholesalers
C. Truck wholesalers
D. Drop shippers
E. Rack jobbers
ANSWER: D
62) ________ sell and deliver a limited line of semiperishable goods to supermarkets, grocery stores, hospitals,
restaurants, and hotels.
A. Producers' cooperatives
B. Cash and carry wholesalers
C. Truck wholesalers

Downloaded by Lan Anh Tr?nh (lananh30303@gmail.com)


lOMoARcPSD|16636219

D. Drop shippers
E. Rack jobbers
ANSWER: C
63) The owner of supermarket chain Reynold's has realized that customers want a wider variety of goods than is currently
available. However, Reynold's cannot afford the costs of storing excess inventory. Additionally, the owner is not willing to
take the risk that the new products will remain unsold. Which of the following types of wholesalers can help Reynold's
meet customer demand while minimizing costs?
A. producers' cooperatives
B. cash and carry wholesalers
C. truck wholesalers
D. drop shippers
E. rack jobbers
ANSWER: E
64) Agricultural assemblers, petroleum bulk plants and terminals, and auction companies are examples of ________.
A. full-service wholesalers
B. specialized wholesalers
C. limited-service wholesalers
D. merchant wholesalers
E. brokers
ANSWER: B
65) Which of the following is true of brokers?
A. Brokers represent buyers or sellers on a semipermanent basis.
B. Most brokers are small businesses with a few skilled salespeople.
C. Brokers bring buyers and sellers together and assist in negotiation.
D. Selling brokers have contractual authority to sell a manufacturer's entire output.
E. Purchasing brokers make purchases for buyers and often receive, inspect, warehouse, and ship merchandise.
ANSWER: C
66) ________ refers to buying large carload lots and dividing them into smaller units before shipping them out to
consumers.
A. Bulk breaking
B. Containerization
C. Wholesaling
D. Warehousing
E. Broking
ANSWER: A
67) ________ encompass sales forecasting, production planning, and inbound materials transportation.
A. Market logistics
B. Containerization
C. Transportation
D. Nonstore retailing
E. Wholesaling
ANSWER: A
68) The elapsed time between an order's receipt, delivery, and payment is called the ________ cycle.
A. variable-costs-to-payment
B. product-to-payment
C. inventory-to-sale
D. order-to-inventory
E. order-to-payment
ANSWER: E
69) ________ costs for a manufacturer consist of setup costs and running costs.
A. Inventory-carrying
B. Containerization
C. Wholesaling
D. Order-processing
E. Transportation
ANSWER: D
70) ________ was originally pioneered by Japanese firms such as Toyota to produce goods with minimal waste of time,
materials, and money.
A. Electronic funds transfer (EFT)

Downloaded by Lan Anh Tr?nh (lananh30303@gmail.com)


lOMoARcPSD|16636219

B. Market logistics
C. Electronic data interchange (EDI)
D. Lean manufacturing
E. Supply chain management
ANSWER: D
71) As inventory draws down, management must know at what stock level to request additional stock. This stock level is
called the ________.
A. reorder point
B. least fixed point
C. point of divergence
D. inflection point
E. critical point
ANSWER: A
72) A stock order point of 10 means ordering the product ________.
A. every 10 days
B. when stock falls to 10 units
C. every 10 units
D. when stock falls to 9 units
E. in batches of 10 items
ANSWER: B
73) Optimal order quantities exist when the curves for the order-processing cost per unit and inventory-carrying cost per
unit ________.
A. are collinear
B. are diagonal to each other
C. intersect
D. are parallel to each other
E. equal zero
ANSWER: C
74) E&OE is trying to minimize its inventory costs, which are extremely high. The company has realized that it can achieve
this by maintaining a near-zero inventory and producing more products only once it is ordered. Which of the following is
true for E&OE?
A. Inventory costs are lower than order-processing costs.
B. Running costs are higher than inventory-carrying costs.
C. Setup costs for the products are low.
D. Order-processing costs are high.
E. Order-processing costs are lower than setup costs.
ANSWER: C
75) E&OE is trying to minimize its inventory costs, which are extremely high. The company has realized that it can achieve
this by maintaining a near-zero inventory and producing more products only once it is ordered. Which of the following will
be true for E&OE?
A. Short production runs will be more expensive than longer ones.
B. Setup and order-processing costs will be high.
C. The order point will be high.
D. Order-processing costs will be lower than the inventory-carrying costs.
E. E&OE can reduce the average cost per unit by producing a long run.
ANSWER: D
76) Beyond the optimal order quantity, total cost per unit increases because ________.
A. inventory-carrying cost per unit increases
B. inventory-carrying cost per unit decreases
C. order-processing cost per unit increases
D. order-processing cost per unit increases though inventory cost decreases
E. inventory-processing cost per unit falls slowly
ANSWER: A
77) An item described as low-risk and low-opportunity is a ________.
A. nuisance item
B. bottleneck item
C. variable item
D. critical item
E. commodity

Downloaded by Lan Anh Tr?nh (lananh30303@gmail.com)


lOMoARcPSD|16636219

ANSWER: A
78) ________ consists of putting goods in boxes or trailers that are easy to transfer between two transportation modes.
A. Containerization
B. Haulage
C. Inventory carrying
D. Order processing
E. Warehousing
ANSWER: A
79) Which of the following is an example of a private carrier?
A. MET is a transporter that operates only in the Chicago area and charges fixed prices.
B. BCL is a family firm that owns only three trucks, but takes small orders for transport.
C. VTV owns a fleet of trucks and transports goods for any client for a fee.
D. COM is a shipping firm that transports goods by road and rail across the United States.
E. BEL Inc. manufactures parts for automobiles and transports its products to customers itself.
ANSWER: E
80) The trucking firm hired by your transportation manager provides a trucking service between the city of Sacramento
and the surrounding towns on a regular schedule and at fixed prices. The trucking firm saves transportation costs by
transporting the goods using trains as well as trucks, instead of trucks alone. Your transportation manager has hired a(n)
________ carrier.
A. airship
B. airtruck
C. trainship
D. fishyback
E. piggyback
ANSWER: E
81) If the shipper owns its own truck or air fleet, it becomes a ________ carrier.
A. containerized
B. private
C. contract
D. common
E. diversified
ANSWER: B

Downloaded by Lan Anh Tr?nh (lananh30303@gmail.com)


lOMoARcPSD|16636219

Chapter 19: Disigning and Managing Integrated Maketing Communications


1) ________ refers to the means by which firms attempt to inform, persuade, and remind consumers — directly or
indirectly — about the products and brands they sell.
A. Human resource development
B. Marketing communications
C. Financial management
D. Operations management
E. Planning
ANSWER: B
2) Which of the following elements of the marketing communications mix refers to any paid form of nonpersonal
presentation and promotion of ideas, goods, or services by an identified sponsor via print, broadcast, network, electronic,
and display media?
A. advertising
B. personal selling
C. sales promotion
D. direct marketing
E. public relations
ANSWER: A
3) Tracy works for a cellular phone company that offers trade-in allowances toward the upgrade of new phones, which is
an example of ________.
A. a sales promotion
B. interactive marketing
C. word-of-mouth marketing
D. personal selling
E. events and experiences
ANSWER: A
4) Which of the following elements of the marketing communications mix involves a variety of short-term incentives to
encourage trial or purchase of a product or service?
A. advertising
B. direct marketing
C. public relations
D. personal selling
E. sales promotion
ANSWER: E
5) Which of the following is an example of a trade promotion?
A. free samples
B. discount coupons
C. display allowances
D. contests for sales reps
E. premiums
ANSWER: C
6) Which of the following is an example of a business and sales force promotion?
A. advertising allowance
B. free samples
C. contests for sales reps
D. display allowance
E. discount coupons
ANSWER: C
7) ________ refer to the marketing communications element that involves company- sponsored activities and programs
designed to create daily or special brand-related interactions with consumers.
A. Publicity campaigns
B. Trade promotions
C. Advertisements
D. Public relations
E. Events and experiences
ANSWER: E
8) Which of the following elements of the marketing communications mix includes a variety of programs directed
internally to employees of the company or externally to consumers, other firms, the government, and media to promote
or protect a company's image or its individual product communications?

Downloaded by Lan Anh Tr?nh (lananh30303@gmail.com)


lOMoARcPSD|16636219

A. direct marketing
B. public relations and publicity
C. personal selling
D. advertising
E. sales promotion
ANSWER: B
9) Which of the following elements of the marketing communication mix involves use of mail, telephone, fax, e-mail, or
Internet to communicate with or solicit response or dialogue from specific customers and prospects?
A. advertising
B. personal selling
C. direct marketing
D. public relations
E. sales promotion
ANSWER: C
10) ________ is an element of the marketing communications mix that involves online activities and programs designed
to engage customers or prospects and directly or indirectly raise awareness, improve image, or elicit sales of products and
services.
A. Personal selling
B. Direct marketing
C. Sales promotion
D. Interactive marketing
E. Public relations
ANSWER: D
11) ________ is an element of the marketing communications mix that involves people-to-people oral, written, or
electronic communications that relate to the merits or experiences of purchasing or using products or services.
A. Personal selling
B. Sales promotion
C. Word-of-mouth marketing
D. Public relations
E. Advertising
ANSWER: C
12) Which of the following elements of the marketing communications mix involves face-to- face interaction with one or
more prospective purchasers for the purpose of making presentations, answering questions, and procuring orders?
A. advertising
B. sales promotion
C. word-of-mouth marketing
D. public relations
E. personal selling
ANSWER: E
13) Which of the following is an example of an advertising platform?
A. posters and leaflets
B. company magazines
C. fairs and trade shows
D. sales presentations
E. continuity programs
ANSWER: A
14) Which of the following is an example of an events and experiences platform?
A. fairs and trade shows
B. continuity programs
C. factory tours
D. sales presentations
E. community relations
ANSWER: C
15) Which of the following is an example of a public relations and publicity communication platform?
A. lobbying
B. company museums
C. street activities
D. company blogs
E. incentive programs

Downloaded by Lan Anh Tr?nh (lananh30303@gmail.com)


lOMoARcPSD|16636219

ANSWER: A
16) Which of the following is an example of an interactive marketing communication platform?
A. product demonstrations
B. factory tours
C. company museums
D. TV shopping
E. community relations
ANSWER: D
17) Which of the following is an example of a word-of-mouth marketing communication platform?
A. chat rooms
B. billboards
C. factory tours
D. incentive programs
E. trade shows
ANSWER: A
18) Which of the following is an example of a personal selling communication platform?
A. sales presentations
B. company blogs
C. telemarketing
D. TV shopping
E. press kits
ANSWER: A
19) The starting point in planning marketing communications is a ________ that profiles all interactions customers in the
target market may have with the company and all its products and services.
A. marketing budget
B. communications audit
C. market research program
D. publicity campaign
E. product launch
ANSWER: B
20) Along which of the following parameters should marketers evaluate communication options when building brand
equity?
A. popularity
B. innovativeness
C. technological sophistication
D. efficiency
E. novelty
ANSWER: D
21) Which of the following benefits is offered by sales promotion tools?
A. Sales promotion tools are more authentic and credible to buyers than others such as advertising, public relations, and
personal selling.
B. Sales promotion tools can reach prospects who prefer to avoid mass media and targeted promotions.
C. Sales promotion tools are typically an indirect form of "soft-sell" and hence, better received by customers.
D. Sales promotion tools incorporate some concession, inducement, or contribution that gives value to the consumer.
E. Sales promotion tools allow buyers personal choices and encourage them to respond directly.
ANSWER: D
22) Which of the following statements correctly reflects a characteristic of public relations as a marketing communications
tool?
A. Public relations can reach prospects who prefer to avoid mass media and targeted promotions.
B. They incorporate some concession, inducement, or contribution that gives value to the consumer.
C. Given their live, real-time quality, public relations tools are more actively engaging for consumers.
D. Public relations communications can be prepared to appeal to the addressed individual.
E. Public relations tools create an immediate and interactive episode between two or more persons.
ANSWER: A
23) Aisha is a marketing manager with Injoos, a company that manufactures packaged fruit juices. Knowing that several
other companies exist in the market that offer similar products, Aisha decides to build a customer base from among those
who prefer to avoid mass media and other targeted promotions. Which of the following marketing communications tools
would be her best option to build a favorable impression among the prospective customers?
A. advertising

Downloaded by Lan Anh Tr?nh (lananh30303@gmail.com)


lOMoARcPSD|16636219

B. personal selling
C. sales promotions
D. direct marketing
E. public relations
ANSWER: E
24) Which of the following marketing communications tools is most effective at the later stages of the buying process?
A. personal selling
B. public relations
C. advertising
D. sales promotions
E. direct marketing
ANSWER: A
25) Which of the following statements is true of the role of advertising in business markets?
A. Advertising is unsuitable for explaining any new features that a product might have.
B. Sales calls are more economical than reminder advertisements.
C. Sales calls are more effective than advertisements at reminding customers how to use a product and reassure them
about their purchase.
D. Sales representatives can use copies of the company's ads to legitimize their company and products.
E. Advertisements are the least preferred tools when intended to generate leads for sales representatives.
ANSWER: D
26) Which of the following circumstances are best suited for personal selling?
A. when the products used are simple and easy-to-use
B. when there is minimal risk involved in buying or using the products
C. when the market has fewer and larger sellers
D. when the products being marketed are inexpensive and easily available
E. when prospective customers are spread across a wide geographic area
ANSWER: C
27) Which of the following marketing communications tools is most effective at influencing customers at the conviction
stage of buyer readiness?
A. advertising
B. publicity
C. sales promotion
D. personal selling
E. events and experiences
ANSWER: D
28) Which element of the marketing communications mix allows for amplified expressiveness?
A. sales promotion
B. public relations
C. advertising
D. direct and interactive marketing
E. personal selling
ANSWER: C
29) Advertising and publicity tools play the most important roles in influencing buying decisions at the ________ stage of
buyer readiness.
A. comprehension
B. conviction
C. ordering
D. reordering
E. awareness-building
ANSWER: E
30) Which of the following tools or combinations of tools is most influential at the comprehension stage of buyer
readiness?
A. sales promotion and advertising
B. advertising and personal selling
C. publicity and personal selling
D. reminder advertising and publicity
E. sales promotion and personal selling
ANSWER: B

Downloaded by Lan Anh Tr?nh (lananh30303@gmail.com)


lOMoARcPSD|16636219

31) Which of the following marketing communications tools has the highest cost- effectiveness in the introduction stage of
the product life cycle?
A. personal selling
B. sales promotion
C. interactive marketing
D. direct marketing
E. events and experiences
ANSWER: E
32) Which of the following marketing communications tools is most influential at the maturity stage of a product's life
cycle?
A. sales promotions
B. direct marketing
C. advertising
D. publicity
E. interactive marketing
ANSWER: C
33) Luke is considering the various options available to him to promote an energy drink, Turbozade, that has decreasing
sales volumes after having peaked some time back. Which of the following marketing communications tools should Luke
focus marketing efforts on to keep the sales volume up?
A. advertising
B. direct marketing
C. events and experiences
D. sales promotions
E. publicity
ANSWER: D
34) Which of the following marketing communications tools is most influential at the reordering stage of buyer readiness?
A. events and experiences
B. publicity
C. direct marketing
D. sales promotion
E. interactive marketing
ANSWER: D
35) ________ is a measure of communications effectiveness that describes the percentage of target market exposed to a
communication.
A. Frequency
B. Reach
C. Width
D. Depth
E. Range
ANSWER: B
36) Which of the following factors found in the macromodel of the communications process refers to random and
competing messages that may interfere with the intended communication?
A. negative feedback
B. noise
C. attenuation
D. phase lag
E. selective distortion
ANSWER: B
37) Which of the following is the correct order of stages that a buyer is assumed to pass through, by the four classic
response hierarchy models?
A. cognitive stage — affective stage — behavioral stage
B. affective stage — cognitive stage — behavioral stage
C. behavioral stage — affective stage — cognitive stage
D. cognitive stage — behavioral stage — affective stage
E. affective stage — behavioral stage — cognitive stage
ANSWER: A
38) LCH is a leading electronics company that produces and markets its own brand of desktop and laptop computers for
both individual consumers and businesses. Which of the following sequences of consumer responses is relevant as a
marketing communications model for LCH's products?

Downloaded by Lan Anh Tr?nh (lananh30303@gmail.com)


lOMoARcPSD|16636219

A. learn-do-feel
B. feel-learn-do
C. do-feel-learn
D. feel-do-learn
E. do-learn-feel
ANSWER: C
39) LCH is a leading electronics company that produces and markets its own brand of desktop and laptop computers, for
both individual consumers and businesses. During the planning process, LCH conducts a ________, which profiles all
interactions customers in the target market may have with LCH, its computers, and its services.
A. sales promotion
B. brand association
C. brand contact
D. response hierarchy model
E. communication audit
ANSWER: E
40) All response hierarchy models of the communication process assume the buyer passes through cognitive, affective,
and behavioral stages, in that order. Which of the following product categories lends itself most appropriately to such a
"learn-feel-do" sequence?
A. clothes
B. dishwashers
C. real estate
D. personal computer
E. air tickets
ANSWER: C
41) When planning communications for a detergent brand, which of the following sequences of buyer responses should
the marketer choose on which to base the communications model?
A. feel-do-learn
B. do-feel-learn
C. feel-learn-do
D. learn-do-feel
E. learn-feel-do
ANSWER: D
42) Which of the following steps in the innovation-adoption model of marketing communications corresponds to the
cognitive stage that a buyer passes through?
A. interest
B. evaluation
C. trial
D. awareness
E. adoption
ANSWER: D
43) According to the hierarchy-of-effects model, which of the following corresponds to the behavioral stage that a buyer
passes through?
A. awareness
B. knowledge
C. purchase
D. preference
E. conviction
ANSWER: C
44) According to the hierarchy-of-effects model, which of the following corresponds to the affective stage that a buyer
passes through?
A. attention
B. exposure
C. reception
D. adoption
E. conviction
ANSWER: E
45) Which of the following sequences accurately represents the hierarchy-of-effects model of marketing communications?
A. attention-interest-desire-action
B. awareness-interest-evaluation-trial-adoption

Downloaded by Lan Anh Tr?nh (lananh30303@gmail.com)


lOMoARcPSD|16636219

C. awareness-knowledge-liking-preference-conviction-purchase
D. exposure-reception-cognitive response-attitude-intention-behavior
E. knowledge-persuasion-decision-implementation-confirmation
ANSWER: C
46) Marketing communication strategy can be decided by conducting an image analysis by profiling the target audience in
terms of ________.
A. brand knowledge
B. purchase patterns
C. demographic characteristics
D. income levels
E. psychographic characteristics
ANSWER: A
47) Which of the following is the marketing communications objective for a new-to-the- world product, such as electric
cars?
A. enhancing brand awareness
B. developing brand attitude
C. increasing brand purchase intention
D. encouraging repeat purchases
E. establishing category need
ANSWER: E
48) When Apple introduced the iPod in October, 2001, it was the first-of-its-kind product that offered sizable storage
capacity for songs and a portable device that was not seen before in the market. Which of the following is most likely to
have been the marketing communications objective for the iPod at the time of its introduction?
A. developing brand awareness
B. building customer traffic
C. enhancing purchase actions
D. establishing product category
E. enhancing firm image
ANSWER: D
49) Which method for establishing the total marketing communications budget sets communication budgets to achieve
the same amount of share-of-voice as competitors?
A. comparative-parity method
B. objective-and-task method
C. affordable method
D. competitive-parity method
E. percentage-of-sales method
ANSWER: D
50) One of the possible objectives of marketing communications is helping consumers evaluate a brand's perceived ability
to meet a currently relevant need. Which of the following is a negatively oriented relevant brand need?
A. normal depletion
B. sensory gratification
C. social approval
D. intellectual stimulation
E. value enhancement
ANSWER: A
51) One of the possible objectives of marketing communications is helping consumers evaluate a brand's perceived ability
to meet a currently relevant need. Which of the following is a positively oriented relevant brand need?
A. problem removal
B. social approval
C. normal depletion
D. problem avoidance
E. incomplete satisfaction
ANSWER: B
52) One of the possible objectives of marketing communications is helping consumers evaluate a brand's perceived ability
to meet a currently relevant need. Which of the following relevant brand needs is most likely emphasized by an
advertisement for a luxury car?
A. problem removal
B. sensory gratification
C. normal depletion

Downloaded by Lan Anh Tr?nh (lananh30303@gmail.com)


lOMoARcPSD|16636219

D. intellectual stimulation
E. problem avoidance
ANSWER: B
53) Creative strategies refer to the ________.
A. way marketers translate their messages into a specific communication
B. amount of creative content in a communications message
C. degree of innovation involved in the marketing of a product
D. novelty of a marketing communication
E. type of medium used to deliver a marketing communication
ANSWER: A
54) A(n) ________ appeal is a creative strategy that elaborates on product or service attributes or benefits.
A. aesthetic
B. informational
C. bandwagon
D. emotional
E. transformational
ANSWER: B
55) A(n) ________ appeal is a creative strategy that elaborates on a nonproduct-related benefit or image.
A. logical
B. transformational
C. reasonable
D. informational
E. rational
ANSWER: B
56) Which of the following ads depict a transformational appeal?
A. Thompson Water Seal can withstand intense rain, snow, and heat.
B. DIRECTV offers better HD options than cable or other satellite operators.
C. NBA phenomenon LeBron James pitching Nike, Sprite, and McDonald's.
D. Pringles advertised "Once You Pop, the Fun Don't Stop" for years.
E. Excedrin stops the toughest headache pain.
ANSWER: D
57) Which of the following ads depict an informational appeal?
A. Thompson Water Seal can withstand intense rain, snow, and heat.
B. The California Milk Processor Board ran the successful "Got Milk?" ad to boost declining sales.
C. VW advertised to active, youthful people with its famed "Drivers Wanted" campaign.
D. Pringles advertised "Once You Pop, the Fun Don't Stop" for years.
E. KFC has marketed its fast-food products under the slogan, "Finger Lickin' Good."
ANSWER: A
58) ________ is one of the sources of a spokesperson's credibility that refers to the specialized knowledge that he or she
claims to possess.
A. Trustworthiness
B. Expertise
C. Acquaintance
D. Likability
E. Professionalism
ANSWER: B
59) ________ is a source of a spokesperson's credibility that describes how objective and honest the spokesperson is
perceived to be.
A. Likability
B. Expertise
C. Experience
D. Trustworthiness
E. Compassion
ANSWER: D
60) Which of the following sources of a spokesperson's credibility describes his or her attractiveness?
A. expertise
B. trustworthiness
C. likability
D. integrity

Downloaded by Lan Anh Tr?nh (lananh30303@gmail.com)


lOMoARcPSD|16636219

E. experience
ANSWER: C
61) Which of the following marketing communications principles implies that communicators can use their good image to
reduce some negative feelings toward a brand but in the process might lose some esteem with the audience?
A. principle of closure
B. principle of duality
C. principle of delegation
D. principle of congruity
E. principle of neutrality
ANSWER: D
62) Which of the following personal communication channels consist of company salespeople contacting buyers in the
target market?
A. expert channels
B. social channels
C. advocate channels
D. independent channels
E. informal channels
ANSWER: C
63) Which of the following personal communications channels consist of family members, neighbors, friends, and
associates talking to target buyers?
A. expert channels
B. advocate channels
C. social channels
D. formal channels
E. sponsored channels
ANSWER: C
64) Top Gear is an award-winning British television series about motor vehicles, mainly cars. It is presented by hosts who
test drive new cars and provide reviews on the cars' performance, their prices, and other factors. Which of the following
personal communications channels is Top Gear closest to in description?
A. expert channel
B. formal channel
C. social channel
D. sponsored channel
E. advocate channel
ANSWER: A
65) Personal influence in marketing communications carries great weight when the ________.
A. product being marketed is a convenience item
B. purchase of the product is considered to be safe and risk-free
C. product suggests something about the user's status or taste
D. product being marketed is purchased on a frequent basis
E. product or service in questions is used without being recommended by others
ANSWER: C
66) Which of the following is a form of mass communications channel?
A. interactive marketing
B. personal selling
C. public relations
D. word-of-mouth marketing
E. sales presentations
ANSWER: C
67) Which of the following statements is true of the two-step approach to mass communications?
A. The influence of mass media on public opinion is more direct, powerful, and automatic than marketers have supposed.
B. Communications through mass media bypasses opinion leaders and reaches the individual buyers.
C. The two-step flow supports the notion that consumption styles are primarily influenced by a "trickle-down" or "trickle-
up" effect from mass media.
D. According to the two-step flow, people interact primarily within their own social groups and acquire ideas from opinion
leaders in their groups.
E. Two-step communication suggests that mass communicators should direct messages to groups of buyers who interpret
the message and act accordingly.
ANSWER: D

Downloaded by Lan Anh Tr?nh (lananh30303@gmail.com)


lOMoARcPSD|16636219

68) Which of the following is a characteristic of the affordable method of establishing a marketing communications
budget?
A. fixed annual budget
B. suitable for long-range planning
C. priority given to role of promotion as an investment
D. calculated to reflect what the company can spare for marketing communications
E. based on the immediate impact of promotion on sales volume
ANSWER: D
69) Which of the following is NOT true about the two-step process by which mass communications affect personal
attitudes?
A. The influence of mass media is mediated by opinion leaders, people whose opinions others seek or who carry their
opinions to others.
B. The influence of mass media on public opinion is not as direct, powerful, and automatic as marketers have supposed.
C. The two-step flow supports the notion that consumption styles are primarily influenced by a trickle-up or trickle-down
effect from mass media.
D. People interact primarily within their own social groups and acquire ideas from opinion leaders in their groups.
E. Two-step communication suggests that mass communicators should direct messages specifically to opinion leaders and
let them carry the message to others.
ANSWER: C
70) Gabrielle is the chief marketing officer of Boyd Pharmaceuticals. She is meeting with Trent, the chief financial officer
to decide on the company's marketing communications budget. After extensive discussions, they decide that the size of
the budget will be calculated as a fraction of the overall turnover. What method did Gabrielle and Trent use to arrive at
the marketing communications budget?
A. affordable method
B. objective-and-task method
C. competitive-parity method
D. activity-based method
E. percentage-of-sales method
ANSWER: E
71) Which of the following is an advantage of using the percentage-of-sales method to determine the marketing
communications budget?
A. The percentage-of-sales method encourages stability when competing firms spend approximately the same portion of
their sales on communications.
B. The percentage-of-sales method views sales as the determiner of communications rather than as the result.
C. The percentage-of-sales method leads to a budget set by market opportunities rather than the availability of funds.
D. The percentage-of-sales method encourages experimentation with countercyclical communication or aggressive
spending.
E. The percentage-of-sales method encourages building the communication budget by determining what each product
and territory deserves.
ANSWER: A
72) Which of the following is a disadvantage of using a percentage-of-sales method to determine the marketing
communications budget?
A. It discourages stability when competing firms spend approximately the same percentage of their sales on
communications.
B. By using a percentage-of-sales method, communication expenditures tend to be extremely high irrespective of what a
company can afford.
C. It discourages management from thinking of the relationship among communication cost, selling price, and profit per
unit.
D. Dependence of the percentage-of-sales method on year-to-year sales fluctuations interferes with long-range planning.
E. The percentage-of-sales method views sales as the result in itself rather than the determiner of communications.
ANSWER: D
73) Gabrielle is the chief marketing officer of Boyd Pharmaceuticals. She is meeting with Trent, the chief financial officer
to decide on the company's marketing communications budget. They pore over the sales reports and other financial
records and determine the amount of resources they can spare for marketing communications, after resources have been
allocated to other functions, such as R&D, logistics, etc. What method did Gabrielle and Trent use to arrive at the
marketing communications budget?
A. objective-and-task method
B. affordable method
C. competitive-parity method

Downloaded by Lan Anh Tr?nh (lananh30303@gmail.com)


lOMoARcPSD|16636219

D. activity-based method
E. percentage-of-sales method
ANSWER: B
74) Gabrielle is the chief marketing officer of Boyd Pharmaceuticals. She is meeting with Trent, the chief financial officer
to decide on the company's marketing communications budget. They decide to trust in the prevailing collective wisdom of
the industry as a whole, and not wanting to instigate a communications war, settle on spending only as much as their
nearest market rival does on marketing communications. What method did Gabrielle and Trent use to arrive at the
marketing communications budget?
A. objective-and-task method
B. affordable method
C. competitive-parity method
D. activity-based method
E. percentage-of-sales method
ANSWER: C
75) Marketing communications budgets tend to be higher when there ________.
A. is high channel support
B. exists hardly any change in the marketing program over time
C. are infrequent product purchases in large quantities
D. are differentiated products and nonhomogeneous customer needs
E. are many easily reachable customers spread over small geographic territories
ANSWER: D
76) Qualities such as candor, humor, and naturalness make a spokesperson ________, which is an important component
of spokesperson credibility.
A. likable
B. an expert
C. trustworthy
D. transformational
E. informational
ANSWER: A
77) As Ben manages communications for his company's watch brand, which has reached the decline stage in the product
life cycle, which of the following marketing communications mix tools is he most likely to continue?
A. interactive marketing
B. advertising
C. personal selling
D. direct marketing
E. sales promotion
ANSWER: E
78) Stacey wants to use a highly relevant, engaging, and implicit communications mix mode to use a "soft sell" approach
for her new makeup line. The communications mix mode that has these characteristics is ________.
A. sales promotions
B. events and experiences
C. advertising
D. direct and interactive marketing
E. personal selling
ANSWER: B
79) Which of the following is NOT one of the four important contributions an effectively trained company sales force can
make to consumer marketing?
A. remind end-consumers about the product
B. increase stock position
C. build enthusiasm
D. conduct missionary selling
E. manage key accounts
ANSWER: A
80) The most effective communications mix tool at later stages of the buying process is ________ because it is
particularly effective at building buyer preference, conviction, and action.
A. sales promotions
B. personal selling
C. advertising
D. direct and interactive marketing

Downloaded by Lan Anh Tr?nh (lananh30303@gmail.com)


lOMoARcPSD|16636219

E. events
ANSWER: B
81) ________ refers to a planning process designed to assure that all brand contacts received by a customer or prospect
for a product, service, or organization are relevant to that person and consistent over time.
A. Brand engagement
B. Integrated marketing communications
C. Market research
D. Customerization
E. Marketing research
ANSWER: B

Downloaded by Lan Anh Tr?nh (lananh30303@gmail.com)


lOMoARcPSD|16636219

Chapter 23: Managing a Holistic Marketing Organization for the Long Run
1) Appointing teams to manage customer-value-building processes and break down walls between departments is part of
which of the following shifts in business practices?
A. reengineering
B. outsourcing
C. benchmarking
D. supplier partnering
E. customer partnering
ANSWER: A
2) ________ involves buying more goods and services from external domestic or foreign vendors.
A. Merging
B. Broadening
C. Outsourcing
D. Globalizing
E. Accelerating
ANSWER: C
3) ________ involves studying "best practice companies" to improve performance.
A. Empowering
B. Globalizing
C. Flattening
D. Benchmarking
E. Focusing
ANSWER: D
4) Merging involves ________.
A. reducing the number of organizational levels to get closer to the customer
B. removing barriers that separate organizational departments
C. partnering with fewer but better value-adding suppliers
D. working more closely with customers to add value to their operations
E. acquiring firms in the same or complementary industries
ANSWER: E
5) Which of the following business practices involves reducing the number of organizational levels to get closer to the
customer?
A. flattening
B. globalizing
C. decentralization
D. merging
E. justifying
ANSWER: A
6) Which of the following business practices focuses specifically on designing the organization and setting up processes to
respond quickly to changes in the environment?
A. benchmarking
B. outsourcing
C. focusing
D. accelerating
E. globalizing
ANSWER: D
7) ________, or reducing the number of organizational levels to get closer to the customer, is one of the important shifts
in marketing and business practices.
A. Merging
B. Flattening
C. Globalizing
D. Focusing
E. Justifying
ANSWER: B
8) As a business practice, broadening involves ________.
A. acquiring or merging with firms in the same or complementary industries to gain economies of scale and scope
B. factoring the interests of customers, employees, shareholders, and other stakeholders into
the activities of the enterprise
C. buying more goods and services from outside domestic or foreign vendors

Downloaded by Lan Anh Tr?nh (lananh30303@gmail.com)


lOMoARcPSD|16636219

D. appointing teams to manage customer-value-building processes and break down walls between departments
E. becoming more accountable by measuring, analyzing, and documenting the effects of marketing actions
ANSWER: B
9) As a marketing practice, monitoring involves ________.
A. designing the organization and setting up processes to respond quickly to changes in the environment
B. becoming more accountable by measuring, analyzing, and documenting the effects of marketing actions
C. tracking what is said online and elsewhere and studying customers, competitors, and others to improve business
practices
D. determining the most profitable businesses and customers and expending greater organizational resources to capitalize
on them
E. factoring the interests of customers, employees, shareholders, and other stakeholders into the activities of the
enterprise
ANSWER: C
10) The role of marketing in the organization is changing. Traditionally marketers have played the roles of ________,
charged with understanding customers' needs and transmitting their voice to various functional areas.
A. middlemen
B. opinion leaders
C. clients
D. influencers
E. end consumers
ANSWER: A
11) Which of the following observations is true?
A. In today's business environment, marketers are mainly middlemen.
B. Marketing has the sole ownership of customer interactions.
C. Only when all employees realize their job is to create, serve, and satisfy customers does the company become an
effective marketer.
D. Marketers must clearly differentiate all customer-facing processes, such that every customer receives a personalized
marketing message.
E. Marketers must avoid collaborating with other organizational departments, as it blurs functional responsibilities.
ANSWER: C
12) ________ requires that everyone in the organization accept the concepts and goals of marketing and engage in
choosing, providing, and communicating customer value.
A. Internal marketing
B. Corporate communications
C. Integrated marketing communications
D. Supply chain management
E. Employee engagement
ANSWER: A
13) The most common form of marketing organization consists of ________ reporting to a marketing vice president who
coordinates their activities.
A. zonal managers
B. functional specialists
C. product managers
D. area market specialists
E. brand managers
ANSWER: B
14) Which of the following is the main advantage of a functional marketing organization?
A. easy coordination
B. lower staff requirements
C. administrative simplicity
D. reduced competition between functional groups
E. smooth working relationships
ANSWER: C
15) A company selling in a national market often organizes its sales force along ________.
A. functional groups
B. geographic lines
C. product teams
D. brand groups
E. product categories

Downloaded by Lan Anh Tr?nh (lananh30303@gmail.com)


lOMoARcPSD|16636219

ANSWER: B
16) A product-management organization makes sense if ________.
A. the company's products are quite varied
B. there are very few products in the company's portfolio
C. the company is pursuing a low cost strategy
D. the company's products satisfy similar customer needs
E. the company is vertically integrated
ANSWER: A
17) A product- or brand-management organization is characterized as a(n) ________ system.
A. hub-and-spoke
B. waterfall
C. internal marketing
D. top-down
E. lateral
ANSWER: A
18) A disadvantage of the product-management organization is that ________.
A. it marginalizes a company's smaller brands
B. it reduces organizational responsiveness to new products in the marketplace
C. product managers generally exercise authority outside their areas of responsibility
D. it prevents product managers from gaining sufficient expertise in their product areas
E. it fails to build long-term strengths as brand managers normally manage brands for only short periods
ANSWER: E
19) A product-management organization ________.
A. often proves to be cost-effective
B. simplifies the process of developing a national strategy
C. focuses on building market share rather than customer relationships
D. reduces an organization's staffing requirements
E. allows product managers to achieve functional expertise
ANSWER: C
20) There are three types of product-team structures. These are ________.
A. vertical, triangular, and horizontal product teams
B. vertical, horizontal, and circular product teams
C. horizontal, vertical, and rectangular product teams
D. horizontal, vertical, and flattened product teams
E. vertical, rectangular, and circular product teams
ANSWER: A
21) Which of the following is true about a brand-asset management team (BAMT)?
A. A BAMT is part of the triangular and vertical product team structures.
B. BAMTs often report directly to the organization's chief branding officer.
C. BAMTs are a traditional, tried and tested means of managing brands.
D. Companies with a product-management organization often have only one BAMT.
E. The BAMT consists of key representatives from functions affecting the brand's performance.
ANSWER: E
22) One of the options in a product-management organization is to eliminate product manager positions for minor
products and assign two or more products to each remaining manager. Under what conditions is this alternative feasible?
A. when the product mix is highly diverse
B. when there are very few products in the company's portfolio
C. when two or more products serve a similar set of needs
D. when customers fall into different user groups
E. when the company produces many products for many markets
ANSWER: C
23) When customers fall into different user groups with distinct buying preferences and practices, a ________
organization is desirable.
A. market-management
B. product-management
C. brand-management
D. geographic
E. functional
ANSWER: A

Downloaded by Lan Anh Tr?nh (lananh30303@gmail.com)


lOMoARcPSD|16636219

24) Companies that produce many products for many markets may adopt a ________ marketing organization.
A. flat
B. brand
C. product
D. matrix
E. top-down
ANSWER: D
25) Which of the following is true regarding a matrix-management organization?
A. It proves to be cost-effective in the long run.
B. It often creates conflicts regarding authority and responsibility for marketing activities.
C. It hampers the flow of information among marketing personnel.
D. It fosters a strong sense of accountability for performance among product and market managers.
E. It is best suited to companies that offer a small range of products to niche markets.
ANSWER: B
26) Transforming into a true market-driven company requires organizing around ________.
A. sales
B. customer segments
C. products
D. functions
E. brands
ANSWER: B
27) Which of the following is true of building a creative marketing organization?
A. It is enough if firms are customer-oriented.
B. Companies must watch trends and be ready to capitalize on them.
C. Firms should focus more on efficiency rather than innovation.
D. Companies should attempt to minimize risks as much as possible.
E. Firms should focus on protecting their existing markets and physical resources.
ANSWER: B
28) Rising customer expectations, evolving employee goals and ambitions, and tighter government legislation and
pressure are driving companies to ________.
A. operate leaner manufacturing facilities
B. manage shorter supply chains
C. operate flatter organizations
D. practice a higher level of corporate social responsibility
E. vertically integrate
ANSWER: D
29) In order to promote ethical cultures, companies should do all of the following EXCEPT ________.
A. disseminate a written code of ethics
B. build a company tradition of ethical behavior
C. hold people responsible for observing ethical and legal guidelines
D. ensure every employee knows and observes relevant laws
E. encourage business practices that are not clearly ethical or unethical
ANSWER: E
30) One opportunity for designing a cause program so the sponsoring company is not overlooked among other corporate
sponsors is to support a(n) ________, or a disease that afflicts fewer than 200,000 people.
A. orphan cause
B. overlooked calamity
C. behavioral campaign
D. social marketing plan
E. value campaign
ANSWER: A
31) ________ gives products the appearance of being more environmentally friendly without living up to that promise.
A. Ambush marketing
B. Greenwashing
C. Astroturfing
D. Viral marketing
E. Green politics
ANSWER: B
32) ________ refers to the ability to meet humanity's needs without harming future generations.

Downloaded by Lan Anh Tr?nh (lananh30303@gmail.com)


lOMoARcPSD|16636219

A. Greenwashing
B. Sustainability
C. Ecological footprinting
D. Scalability
E. Legal practice
ANSWER: B
33) Cause-related marketing is part of ________.
A. sustainability
B. corporate societal marketing
C. social media
D. global marketing
E. community marketing
ANSWER: B
34) Cause marketing has a particularly interested audience in civic-minded ________.
A. Millennial consumers
B. baby boomers
C. Gen Xers
D. Gen Yers
E. shadow boomers
ANSWER: A
35) Cadbury's "Sports for Schools" promotion offered sports and fitness equipment for schools in exchange for vouchers.
The problem was that the public and media saw a perverse incentive for children to eat more chocolate, a product
associated with obesity. Which of the following best summarizes Cadbury's problem?
A. Customers felt that the cause was not in sync with the company's brand image.
B. Consumers did not value the cause Cadbury was promoting.
C. Customers questioned the link between the product and the cause and saw the firm as self-serving and exploitive.
D. Consumers resented being sold an inferior product on the back of a cause-marketing program.
E. Consumers felt that the campaign did not make a sufficient attempt to change the target audience's behavior.
ANSWER: C
36) Which of the following is true regarding cause-related marketing?
A. The positive impact of cause-related marketing can be increased through sporadic involvement with numerous causes.
B. Many companies focus on multiple causes to simplify execution and maximize impact.
C. Limiting support to a single cause increases the pool of stakeholders who can transfer positive feelings from the cause
to the firm.
D. Most firms choose causes that fit their corporate or brand image and matter to their employees and shareholders.
E. In order to avoid public backlash, firms are advised to adopt a hard-sell approach to their cause efforts.
ANSWER: D
37) ________ marketing by nonprofit or government organizations furthers a cause.
A. Corporate societal
B. Brand
C. Causal
D. Social
E. Place
ANSWER: D
38) Social marketing programs designed to discourage cigarette smoking or excessive consumption of alcohol are
examples of ________ campaigns.
A. cognitive
B. active
C. behavioral
D. value
E. normative
ANSWER: C
39) A social marketing program which aims to alter ideas about abortion is an example of a(n) ________ campaign.
A. cognitive
B. active
C. behavioral
D. value
E. normative
ANSWER: D

Downloaded by Lan Anh Tr?nh (lananh30303@gmail.com)


lOMoARcPSD|16636219

40) Which of the following represents the objective of a cognitive social marketing campaign?
A. Motivate people with obesity to eat healthy and exercise more often.
B. Change public attitudes and stereotypes associated with people who are obese.
C. Explain the different causes of obesity and how it can be prevented.
D. Encourage people to participate in a walkathon aimed at promoting awareness about obesity.
E. Help people with obesity to implement lifestyle changes.
ANSWER: C
41) Which of the following represents the objective of a social marketing campaign aimed at changing people's actions?
A. Motivate people with obesity to eat healthy and exercise more often.
B. Change public attitudes and stereotypes associated with people who are obese.
C. Explain the different causes of obesity and how it can be prevented.
D. Encourage people to participate in a walkathon aimed at promoting awareness about obesity.
E. Help people with obesity to implement lifestyle changes.
ANSWER: D
42) Which of the following represents the objective of a social marketing campaign aimed at changing people's values?
A. Motivate people with obesity to eat healthy and exercise more often.
B. Change public attitudes and stereotypes associated with people who are obese.
C. Explain the different causes of obesity and how it can be prevented.
D. Encourage people to participate in a walkathon aimed at promoting awareness about obesity.
E. Help people with obesity to implement lifestyle changes.
ANSWER: B
43) Which of the following represents the objective of a social marketing campaign aimed at changing people's behavior?
A. Motivate people with obesity to eat healthy and exercise more often.
B. Change public attitudes and stereotypes associated with people who are obese.
C. Explain the different causes of obesity and how it can be prevented.
D. Encourage people to participate in a walkathon aimed at promoting awareness about obesity.
E. Attract people with obesity to sign up for a one-time free medical check up.
ANSWER: A
44) Which of the following should be done to increase the likelihood that social marketing programs will be successful?
A. Choose target markets that are the least ready to respond.
B. Promote multiple, doable behaviors in clear, simple terms.
C. Make it easy to adopt the behavior.
D. Adopt a soft sell approach, as opposed to attention grabbing messages.
E. Explain the benefits in an exaggerated manner.
ANSWER: C
45) The first step in the social marketing planning process is ________.
A. determining the focus of the program
B. selecting the target audience
C. setting objectives and goals
D. designing the market offering
E. finding a source of funding
ANSWER: A
46) ________ software provides a set of Web-based applications that automate and integrate project management,
campaign management, budget management, asset management, brand management, customer relationship
management, and knowledge management.
A. Marketing dashboard
B. Enterprise resource planning
C. Supply chain management
D. Marketing resource management
E. Enterprise campaign management
ANSWER: D
47) ________ is the process that turns marketing plans into action assignments and ensures
that such assignments are executed in a manner that accomplishes the plan's stated objectives.
A. Marketing implementation
B. Marketing research
C. Marketing analysis
D. Brand management
E. Product management
ANSWER: A

Downloaded by Lan Anh Tr?nh (lananh30303@gmail.com)


lOMoARcPSD|16636219

48) ________ is the process by which firms assess the effects of their marketing activities and programs and make
necessary changes and adjustments.
A. Marketing control
B. Marketing implementation
C. Test marketing
D. Market watch
E. Market analysis
ANSWER: A
49) As long as finance focuses on short-term profit, it will oppose major investments designed to build satisfied, loyal
customers. Which of the following steps that a marketing CEO can take to create a market- and customer-focused
company can help a CEO overcome this challenge?
A. Empower the employees.
B. Get outside help and guidance.
C. Install a modern marketing planning system.
D. Change the company's reward measurement and system.
E. Develop strong in-house marketing training programs.
ANSWER: D
50) IBM lets frontline employees spend up to $5,000 to solve a customer problem on the spot, which is an example of
which of the following steps that a marketing CEO can take to create a market- and customer-focused company?
A. Empower the employees.
B. Hire strong marketing talent.
C. Get outside help and guidance.
D. Install a modern marketing planning system.
E. Develop strong in-house marketing training programs.
ANSWER: A
51) Top and middle management are primarily responsible for ________ control.
A. annual-plan
B. efficiency
C. profitability
D. technological
E. innovation
ANSWER: A
52) ________ software provides a set of Web-based applications that automate and integrate project management,
campaign management, budget management, asset management, brand management, customer relationship
management, and knowledge management.
A. Annual plan control
B. Profitability control
C. Efficiency control
D. Strategic control
E. Marketing resource management (MRM)
ANSWER: E
53) Who is primarily responsible for efficiency control?
A. BAMT
B. marketing auditor
C. middle management
D. line and staff management
E. top management
ANSWER: D
54) The purpose of profitability control is to ________.
A. examine whether the planned results are being achieved
B. examine where the company is making and losing money
C. evaluate and improve the spending efficiency and impact of marketing expenditures
D. examine whether the company is pursuing its best opportunities with respect to markets, products, and channels
E. understand the efficiency of the sales force, advertising, sales promotion, and distribution
ANSWER: B
55) The ________ is primarily responsible for strategic control.
A. line and staff management
B. marketing auditor
C. marketing controller

Downloaded by Lan Anh Tr?nh (lananh30303@gmail.com)


lOMoARcPSD|16636219

D. middle management
E. BAMT
ANSWER: B
56) The purpose of strategic control is to ________.
A. examine whether the planned results are being achieved
B. examine where the company is making and losing money
C. evaluate and improve the spending efficiency and impact of marketing expenditures
D. examine whether the company is pursuing its best opportunities with respect to markets, products, and channels
E. understand the efficiency of the sales force, advertising, sales promotion, and distribution
ANSWER: D
57) Marketing effectiveness rating instruments and marketing audits are approaches to ________ control.
A. annual-plan
B. profitability
C. efficiency
D. strategic
E. statistical
ANSWER: D
58) Which of the following is true regarding annual-plan control?
A. This control process begins by identifying the causes of serious performance deviations.
B. The tools used for this purpose are sales analysis, market share analysis, marketing expense-to-sales analysis, and
financial analysis.
C. The marketing controller has the primary responsibility for annual-plan control.
D. Its purpose is to evaluate and improve the spending efficiency and impact of marketing expenditures.
E. It measures profitability by product, territory, customer, segment, trade channel, and order size.
ANSWER: B
59) Which of the following is an example of a distribution metric used for measuring the performance of marketing plans?
A. effective reach
B. customer acquisition costs
C. market share
D. stocks cover in days
E. response rate
ANSWER: D
60) Your firm has experienced a decline in sales over the last three quarters. You have traced the problems to distribution
inefficiencies. Which of the following should you track to ensure that the firm's distribution efficiency is maximized?
A. average sales per point of sale
B. sales from new products
C. trial rate
D. repurchase rate
E. new customer gains
ANSWER: A
61) A ________ is a comprehensive, systematic, independent, and periodic examination of a company's or business unit's
marketing environment, objectives, strategies, and activities, with a view to determining problem areas and opportunities
and recommending a plan of action to improve the company's marketing performance.
A. marketing plan
B. test market
C. marketing audit
D. market-based scorecard analysis
E. marketing metric
ANSWER: C
62) Which of the following is true regarding a marketing audit?
A. It focuses primarily on a firm's macromarketing environment.
B. It identifies the most-needed improvements and incorporates them into a corrective- action plan with short- and long-
run steps.
C. It is less effective at locating the real source of a problem than a functional audit.
D. It analyzes only those marketing activities that have failed to produce adequate results.
E. It relies solely on company managers for data and opinions.
ANSWER: B
63) To succeed in the future, marketing must ________.
A. become more holistic

Downloaded by Lan Anh Tr?nh (lananh30303@gmail.com)


lOMoARcPSD|16636219

B. build brands through promotion rather than performance


C. be more departmental
D. focus on free-spending
E. rely more on mass marketing
ANSWER: A
64) Which of the following is likely to be an important trend in marketing in the future?
A. marketing intuition
B. free-spending marketing
C. manual marketing
D. marketing science
E. mass marketing
ANSWER: D
65) Which of the following is a best practice in business and marketing?
A. end-product orientation
B. reacting to competitors
C. vertical integration
D. stockholder driven
E. teamwork
66) Which of the four characteristics of a marketing audit is described by the fact that outside consultants can bring
necessary objectivity to the process?
A. comprehensive
B. systematic
C. independent
D. periodic
E. opinion-based
67) ________ analysis looks at specific products, territories, and so forth that failed to produce expected sales.
A. Microsales
B. Sales variance
C. Expense-to-sales
D. Full costing
E. Financial
68) Served market share ________.
A. expresses the company's sales as a percentage of total market sales
B. is sales as a percentage of the total sales to the market
C. is market share in relationship to the largest competitor
D. is always smaller than overall market share
E. measures the relative contribution of different factors to a gap in sales performance
69) ________ is market share in comparison to the largest competitor.
A. Relative market share
B. Served market share
C. Overall market share
D. Market value
E. Target market share
70) How much of the sales performance gap is due to price decline?
A. 0.4 percent
B. 2.5 percent
C. 28.5 percent
D. 37 percent
E. 71.4 percent
70) The sales gap due to reduced volume is ________ percent.
A. 0.4
B. 2.5
C. 28.5
D. 63
E. 71.4
71) If Ming was interested in capturing distribution metrics for her retail organization, which of the following might she
consider?
A. customer complaints
B. share of shelf

Downloaded by Lan Anh Tr?nh (lananh30303@gmail.com)


lOMoARcPSD|16636219

C. effective reach
D. trial rate
E. market share
72) Which of the following is true about market share?
A. Outside forces affect all companies in the same way.
B. A company's performance should be judged against the average performance of all companies.
C. A decline in market share does not necessarily mean the company is performing worse than are other companies.
D. A decline in market share cannot be deliberately engineered.
E. All shifts in market share have marketing significance.
73) A(n) ________ market share of exactly 100 percent means that a company is tied for the market lead. A rise in
relative market share means the company is gaining on its leading competitor.
A. overall
B. served
C. potential
D. relative
E. actual
74) Annual-plan control requires making sure the company isn't overspending to achieve sales goals. The key ratio to
watch is ________.
A. stock turnover
B. gross margin
C. return on capital
D. cash flow return on investment
E. marketing expense-to-sales
75) ________ approach advocates argue that all costs must ultimately be imputed in order to determine true profitability.
A. Direct-cost
B. Full-cost
C. Traceable-cost
D. Activity-based costing
E. Fixed cost
76) Cabot's CEO's annual compensation is an example of a ________ cost.
A. direct
B. variable
C. traceable common
D. nontraceable common
E. manufacturing
77) The cost of land where a plant that manufactures common parts is set up falls under ________ costs.
A. traceable common
B. nontraceable common
C. variable
D. manufacturing
E. material
78) Suppose the manufacturer pays a commission on every car sold. Then, the salesperson's commission is classified as
a(n) ________.
A. cost of labor
B. traceable common cost
C. nontraceable common cost
D. advertising cost
E. direct cost
79) The cost of operating the common manufacturing facility is a(n) ________ cost.
A. opportunity
B. traceable
C. nontraceable
D. sunk
E. differential
80) The manufacturer launches a brand building advertising campaign. The campaign does not promote any one specific
car but is aimed at promoting the company as a whole. Which type of cost does this fall under?
A. direct costs
B. material costs
C. nontraceable costs

Downloaded by Lan Anh Tr?nh (lananh30303@gmail.com)


lOMoARcPSD|16636219

D. traceable costs
E. labor costs
81) Which of the following is NOT a predicted direction for marketing in the coming years?
A. the demise of precision marketing and rise of mass marketing
B. the demise of the marketing department and the rise of holistic marketing
C. the demise of marketing intuition and the rise of marketing science
D. the demise of manual marketing and the rise of both automated and creative marketing
E. the demise of free-spending marketing and the rise of ROI marketing

Downloaded by Lan Anh Tr?nh (lananh30303@gmail.com)

You might also like